100
Volume 10 Number 1 January 2011 Geriatric Nephrology Guest Co-Editors: Dimitrios G. Oreopoulos, MD, PhD, Richard J. Glassock, MD, Sarbjit Vanita Jassal, MD, Ann M. O'Hare, MD, Mitchell H. Rosner, MD, Mark A. Swidler, MD, Mark E. Williams, MD, and Jocelyn Wiggins, BM, BCh Editor-in-Chief: Stanley Goldfarb, MD Deputy Editor: Raymond R. Townsend, MD NephSAP Nephrology Self-Assessment Program ®

2011.01 - Geriatric Nephrology

  • Upload
    hhmmee

  • View
    173

  • Download
    4

Embed Size (px)

Citation preview

Page 1: 2011.01 - Geriatric Nephrology

Volume 10 • Number 1 • January 2011

Geriatric NephrologyGuest Co-Editors: Dimitrios G. Oreopoulos, MD, PhD,Richard J. Glassock, MD,Sarbjit Vanita Jassal, MD,Ann M. O'Hare, MD,Mitchell H. Rosner, MD,Mark A. Swidler, MD,Mark E. Williams, MD,and Jocelyn Wiggins, BM, BCh

� Editor-in-Chief: Stanley Goldfarb, MD

� Deputy Editor: Raymond R. Townsend, MD

NephSAPNephrology Self-Assessment Program

®

Page 2: 2011.01 - Geriatric Nephrology

EDITOR-IN-CHIEFStanley Goldfarb, MDUniversity of Pennsylvania Medical SchoolPhiladelphia, PA

DEPUTY EDITORRaymond R. Townsend, MDUniversity of Pennsylvania Medical SchoolPhiladelphia, PA

MANAGING EDITORGisela Deuter, BSN, MSAWashington, DC

ASSOCIATE EDITORSRajiv Agarwal, MDIndiana University School of MedicineIndianapolis, IN

David J. Cohen, MDColumbia UniversityNew York, NY

Michael J. Choi, MDJohns Hopkins University School of MedicineBaltimore, MD

Michael Emmett, MDBaylor UniversityDallas, TX

Linda F. Fried, MD, MPHUniversity of PittsburghPittsburgh, PA

Richard J. Glassock, MDProfessor Emeritus, The David Geffen Schoolof Medicine at the University of CaliforniaLos Angeles, CA

Kathleen D. Liu, MDUniversity of California San FranciscoSan Francisco, CA

Kevin J. Martin, MBBChSt. Louis University School of MedicineSt. Louis, MO

Rajnish Mehrotra, MDHarbor UCLA Research and Education InstituteTorrance, CA

Patrick T. Murray, MDUniversity College DublinDublin, Ireland

Patrick H. Nachman, MDUniversity of North CarolinaChapel Hill, NC

Aldo J. Peixoto, MDYale UniversityWest Haven, CT

Richard H. Sterns, MDUniversity of Rochester School of Medicineand DentistryRochester, NY

John P. Vella, MDMaine Medical CenterPortland, ME

FOUNDING EDITORSRichard J. Glassock, MD, MACPEditor-in-Chief Emeritus

Robert G. Narins, MD, MACP

PrefaceNephSAP® is one of the three major publications of the American Society of Nephrology(ASN). Its primary goals are self-assessment, education, and the provision of ContinuingMedical Education (CME) credits and Maintenance of Certification (MOC) credits forindividuals certified by the American Board of Internal Medicine. Members of the ASNautomatically receive NephSAP with their monthly issue of The Journal of the AmericanSociety of Nephrology (JASN).

EDUCATION: Medical and Nephrologic information continually accrues at a rapid pace.Bombarded from all sides with demands on their time, busy practitioners, academicians, andtrainees at all levels are increasingly challenged to review and understand all this new material.

Each bimonthly issue of NephSAP is dedicated to a specific theme, i.e., to a specific areaof clinical nephrology, hypertension, dialysis, and transplantation, and consists of an Editorial,a Syllabus, a Commentary on the Syllabus, and self-assessment questions. Over the course of24 months, all clinically relevant and key elements of nephrology will be reviewed and updated.The authors of each issue digest, assimilate, and interpret key publications from the previousissues of other years and integrate this new material with the body of existing information.

SELF-ASSESSMENT: Twenty-five single-best-answer questions will follow the 50 to 75 pagesof Syllabus text. The examination is available online with immediate feedback. Those answer-ing �75% correctly will receive CME credit, and receive the answers to all the questions alongwith brief discussions and an updated bibliography. To help answer the questions, readers maygo to the ASN web site, where relevant material from UpToDate in nephrology will be posted.Thus, members will find a new area reviewed every 2 months, and they will be able to test theirunderstanding with our quiz. This format will help readers stay abreast of developing areas ofclinical nephrology, hypertension, dialysis, and transplantation, and the review and update willsupport those taking certification and recertification examinations.

CONTINUING MEDICAL EDUCATION: Most state and local medical agencies as well ashospitals are demanding documentation of requisite CME credits for licensure and for staffappointments. A maximum of 48 credits annually can be obtained by successfully completingthe NephSAP examination. In addition, individuals certified by the American Board of InternalMedicine may obtain credits towards Maintenance of Certification (MOC) by successfullycompleting the self-assessment portion of NephSAP.

BOARD CERTIFICATION AND INSERVICE EXAMINATION PREPARATION: Each issuewill also contain 5 questions and answers examining core topics in the particular disciplinereviewed in the Syllabus. These questions are designed to provide trainees with challengingquestions to test their knowledge of key areas of nephrology.

� This paper meets the requirements of ANSI/NISO Z39.48-1921 (Permanence of Paper),effective with July 2002, Vol. 1, No. 1.

GUEST CO-EDITORSRichard J. Glassock, MDProfessor Emeritus, The David Geffen School ofMedicine at the University of California, LosAngeles, CA

Sarbjit Vanita Jassal, MDToronto General Hospital Toronto, Ontario, Canada

Ann M. O’Hare, MDUniversity of Washigton, Seattle, WA

Dimitrois G. Orepoulos, MD, PhDUniversity of Toronto, Toronto, Ontario, Canada

Mitchell H. Rosner, MDUniversity of Virginia Health System,Charlottesville, VA

Mark A. Swidler, MDMount Sinai Medical Center, New York, NY

Jocelyn Wiggins, BM, BChUnivesrity of Michigan, Ann Arbor, MI

Mark E. Williams, MDHarvard Medical School, Boston, MA

NephSAP®

©2011 by The American Society of Nephrology

NephSAP®

Page 3: 2011.01 - Geriatric Nephrology

Geriatric Nephrology

Editorial 1Geriatric Nephrology: A Missing Area of Nephrology’s

Expertise—Lynn E. Schlanger, MD, James L. Bailey, MD, andJeff M. Sands, MD

Syllabus 6Geriatric Nephrology: Another Milestone in a 25-Year Jour-

ney—Dimitrios G. Oreopoulos, MD, PhD, Richard J. Glassock,MD, Sarbjit Vanita Jassal, MD, Ann M. O’Hare, MD, MitchellH. Rosner, MD, Mark A. Swidler, MD, Mark E. Williams, MD,and Jocelyn Wiggins, BM, BCh

Early Efforts . . . . . . . . . . . . . . . . . . . . . . . . . . . . . . . . . . . . . . .6

Declining Interest . . . . . . . . . . . . . . . . . . . . . . . . . . . . . . . . .6

T. Franklin Williams Scholars Program . . . . . . . . . . . . . . . .6

Geriatric Nephrology and the National Institutes ofHealth . . . . . . . . . . . . . . . . . . . . . . . . . . . . . . . . . . . . . . . . .7

Mandated Training of Geriatric Nephrology and the Roleof the ASN: A New Era . . . . . . . . . . . . . . . . . . . . . . . . . .7

Geriatric Nephrology Courses during the ASN RenalWeek . . . . . . . . . . . . . . . . . . . . . . . . . . . . . . . . . . . . . . . . .7

Outreach to Other Societies . . . . . . . . . . . . . . . . . . . . . . . . .7

Geriatric Nephrology in NephSAP . . . . . . . . . . . . . . . . . . . .7

Introduction to the Biology of Aging and the Kidney . . . .8

Cellular Processes that Change with Age . . . . . . . . . . . . . .8

Epigenetics . . . . . . . . . . . . . . . . . . . . . . . . . . . . . . . . . . . . .8

Micro RNAs . . . . . . . . . . . . . . . . . . . . . . . . . . . . . . . . . . .9

Mitochondria . . . . . . . . . . . . . . . . . . . . . . . . . . . . . . . . . . .9

NephSAP®

Volume 10, Number 1, January 2011

Page 4: 2011.01 - Geriatric Nephrology

Oxidative Stress . . . . . . . . . . . . . . . . . . . . . . . . . . . . . . . .10

Autophagy . . . . . . . . . . . . . . . . . . . . . . . . . . . . . . . . . . . . .10

Telomeres . . . . . . . . . . . . . . . . . . . . . . . . . . . . . . . . . . . . .10

Calorie Restriction . . . . . . . . . . . . . . . . . . . . . . . . . . . . . .11

DNA Damage and Progeria . . . . . . . . . . . . . . . . . . . . . .11

Individual Aging Genes . . . . . . . . . . . . . . . . . . . . . . . . . . .12

IGF-1 . . . . . . . . . . . . . . . . . . . . . . . . . . . . . . . . . . . . . . . .12

TOR Signaling . . . . . . . . . . . . . . . . . . . . . . . . . . . . . . . . .12

Sirtuins . . . . . . . . . . . . . . . . . . . . . . . . . . . . . . . . . . . . . . .12

Klotho . . . . . . . . . . . . . . . . . . . . . . . . . . . . . . . . . . . . . . .12

Animal Models of Extended Lifespan . . . . . . . . . . . . . . . .13

Age, Estimated GFR Formulas, and Assessment of Riskfor Adverse Cardiovascular and Renal Outcomes inChronic Kidney Disease . . . . . . . . . . . . . . . . . . . . . . .15

Cross-Sectional Relationships among Age, Estimated GFR,and Proteinuria . . . . . . . . . . . . . . . . . . . . . . . . . . . . . . . . .15

Longitudinal Changes in Renal Function with Aging . . . .15

Validity of Methods for Estimating GFR in the Elderly . .16

Relationship between eGFR and Proteinuria and Risk forDeath in Older Adults . . . . . . . . . . . . . . . . . . . . . . . . . . .17

Association of eGFR with Mortality . . . . . . . . . . . . . . .17

Association of Proteinuria with Mortality . . . . . . . . . . .18

Combined Association of eGFR and Proteinuria withMortality . . . . . . . . . . . . . . . . . . . . . . . . . . . . . . . . . . .20

Relationship between eGFR and Proteinuria and Risk forProgression of Kidney Disease in Older Adults . . . . . .20

Competing Risks for Death and Progression to ESRD . . .21

Risk Factors for Progression to ESRD in the Elderly . . . .21

Geriatric Hypertension . . . . . . . . . . . . . . . . . . . . . . . . . . . . . .23

Mechanisms . . . . . . . . . . . . . . . . . . . . . . . . . . . . . . . . . . . . .24

NephSAP®

Volume 10, Number 1, January 2011

Page 5: 2011.01 - Geriatric Nephrology

Central Nervous System Complications . . . . . . . . . . . . . . .26

Treatment . . . . . . . . . . . . . . . . . . . . . . . . . . . . . . . . . . . . . . .27

Hypertension in the Very Elderly Trial . . . . . . . . . . . . . . .28

Lifestyle Modification . . . . . . . . . . . . . . . . . . . . . . . . . . . . .29

Renin-Angiotensin-Aldosterone Blockade . . . . . . . . . . . . .31

Adverse Effects and Adherence . . . . . . . . . . . . . . . . . . . . .32

Diabetic Kidney Disease in the Elderly . . . . . . . . . . . . . . . . .35

Epidemiology . . . . . . . . . . . . . . . . . . . . . . . . . . . . . . . . . . . .35

Pathophysiology . . . . . . . . . . . . . . . . . . . . . . . . . . . . . . . . . .36

Role of Advanced Glycation End Products . . . . . . . . . . . .37

Kidney Function . . . . . . . . . . . . . . . . . . . . . . . . . . . . . . . . .39

Treatment . . . . . . . . . . . . . . . . . . . . . . . . . . . . . . . . . . . . . . .39

Glycemic Control . . . . . . . . . . . . . . . . . . . . . . . . . . . . . .39

Renin-Angiotensin-Aldosterone System Blockade . . . . .40

Glomerular Diseases . . . . . . . . . . . . . . . . . . . . . . . . . . . . . . . .43

Epidemiology of Glomerular Disease in the Elderly . . . . .43

MCD in the Elderly . . . . . . . . . . . . . . . . . . . . . . . . . . . . . . .43

MN in the Elderly . . . . . . . . . . . . . . . . . . . . . . . . . . . . . . . .44

Crescentic GN and Small Vessel Vasculitis in theElderly . . . . . . . . . . . . . . . . . . . . . . . . . . . . . . . . . . . . . . .45

IgAN in the Elderly . . . . . . . . . . . . . . . . . . . . . . . . . . . . . . .46

Membranoproliferative GN in the Elderly . . . . . . . . . . . . .46

Postinfectious GN in the Elderly . . . . . . . . . . . . . . . . . . . .47

LN in the Elderly . . . . . . . . . . . . . . . . . . . . . . . . . . . . . . . .47

Monoclonal Ig Deposition Diseases in the Elderly . . . . . .48

Amyloidosis . . . . . . . . . . . . . . . . . . . . . . . . . . . . . . . . . . .48

Non-Amyloid MIDD . . . . . . . . . . . . . . . . . . . . . . . . . . . .49

Drug-Associated Glomerular Disease in the Elderly . . . . .49

NephSAP®

Volume 10, Number 1, January 2011

Page 6: 2011.01 - Geriatric Nephrology

Other Glomerular Diseases in the Elderly . . . . . . . . . . . . .49

Acute Kidney Injury . . . . . . . . . . . . . . . . . . . . . . . . . . . . . . . .52

Incidence and Risk Factors for Acute Kidney Injury . . . .52

Structural and Cellular Changes in the Aging Kidney . . .53

Calorie Restriction and SIRT1 . . . . . . . . . . . . . . . . . . . . . .53

Diagnosis of AKI in the Elderly . . . . . . . . . . . . . . . . . . . . .54

Impact of AKI in the Elderly . . . . . . . . . . . . . . . . . . . . . . .54

Mortality . . . . . . . . . . . . . . . . . . . . . . . . . . . . . . . . . . . . .54

Risk for CKD and ESRD . . . . . . . . . . . . . . . . . . . . . . . .55

Quality of Life . . . . . . . . . . . . . . . . . . . . . . . . . . . . . . . . .55

Management of AKI in the Elderly . . . . . . . . . . . . . . . . . .55

Therapeutic Options for Older Individuals with CKD . . . . .57

Geriatric Syndromes in Elderly Patients with CKD . . . . .58

Renal Replacement Therapies for Older Individuals . . . . .59

Home-Based Therapies: Home HD, PD, and NocturnalDialysis . . . . . . . . . . . . . . . . . . . . . . . . . . . . . . . . . . . .59

Hospital- or Center-Based Therapy: HD and NocturnalDialysis . . . . . . . . . . . . . . . . . . . . . . . . . . . . . . . . . . . .59

Assistive Programs and Dialysis in Nursing Homes . . .60

Nondialysis Care as an Active Treatment Strategy . . . .60

Transplantation . . . . . . . . . . . . . . . . . . . . . . . . . . . . . . . .61

Outcomes with and without Renal Replacement Therapies . . .61

Survival . . . . . . . . . . . . . . . . . . . . . . . . . . . . . . . . . . . . . .61

Quality of Life . . . . . . . . . . . . . . . . . . . . . . . . . . . . . . . . .62

Associated Morbidity . . . . . . . . . . . . . . . . . . . . . . . . . . .63

Miscellaneous Issues Relating to the TherapeuticManagement of CKD in Older Patients . . . . . . . . . . . . .64

Palliative Care and Geriatric Treatment of Patients withAdvanced Chronic Kidney Disease . . . . . . . . . . . . . . . .67

NephSAP®

Volume 10, Number 1, January 2011

Page 7: 2011.01 - Geriatric Nephrology

Palliative Care . . . . . . . . . . . . . . . . . . . . . . . . . . . . . . . . . . .68

Palliative Care in Medical Practice . . . . . . . . . . . . . . . . . .69

Geriatric Palliative Care . . . . . . . . . . . . . . . . . . . . . . . . . . .70

Palliative Care in Advanced CKD and Dialysis . . . . . . . .70

Geriatric Medical Decision Making . . . . . . . . . . . . . . . . . .71

Nondialysis Medical Renal Therapy . . . . . . . . . . . . . . . . . .74

Symptom Burden . . . . . . . . . . . . . . . . . . . . . . . . . . . . . . . . .75

Summary and Conclusion . . . . . . . . . . . . . . . . . . . . . . . . . .78

CME Self-Assessment Questions . . . . . . . . . . . . . . . . . . . . . 82Questions Linked to UpToDate in Green

Upcoming Issues

Fluid, Electrolyte, and Acid-Base Disturbances—

Richard H. Sterns, MD, and Michael Emmett, MD . . . . . . .March 2011

Acute Kidney Injury and Critical Care Nephrology—

Patrick T. Murray, MD, and Kathleen D. Liu, MD . . . . . . . . .May 2011

Renal Pathology—

Glen S. Markowitz, MD, Barry Stokes, MD, Neeraja Kambham, MD, Leal

C. Herlitz, MD, and Vivette D. D’Agati, MD . . . . . . . . . . . . .July 2011

Chronic Kidney Disease and Progression—

Linda F. Fried, MD, and Michael J. Choi, MD . . . . . .September 2011

Transplantation—

John P. Vella, MD, and David J. Cohen, MD. . . . . . . .November 2011

NephSAP®

Volume 10, Number 1, January 2011

Page 8: 2011.01 - Geriatric Nephrology

The Editorial Board of NephSAP extends its sincere appreciation to the following reviewers. Their efforts and insights have helped toimprove the quality of this postgraduate education offering.

NephSAP Review PanelNihal Y. Abosaif, MBBChSt. James University HospitalLeeds, United Kingdom

Georgi Abraham, MBBSSri Ramachandra UniversityMedical CenterChennai, India

Pablo H. Abrego, MD, FASNMarshfield ClinicWausau, WI

Anil K. Agarwal, MD, FASNOhio State University Medical CenterColumbus, OH

Mustafa Ahmad, MDKing Fahad Medical CityRiyadh, Saudi Arabia

Jafar Al-Said, MD, FASNBahrain Specialist HospitalManama, Bahrain

Dante Amato-Martinez, MD, PhDUniversidad Nacional Autonoma de MexicoTlalnepantla, Mexico

Anis U. Ansari, MDMedical AssociatesClinton, IA

Akhtar Ashfaq, MD, FASNNorth Shore University HospitalGreat Neck, NY

Azra Bihorac, MD, FASNUniversity of FloridaGainesville, FL

Mona B. Brake, MDRobert J. Dole VA Medical CenterWichita, KS

Mauro Braun, MDCleveland Clinic FloridaWeston, FL

Chokchai Chareandee, MD, FASNRegions HospitalSaint Paul, MN

W. James Chon, MD, FASNUniversity of Chicago Medical CenterChicago, IL

Devasmita Choudhury, MDUniversity of Texas SouthwesternMedical SchoolDallas, TX

Bulent Cuhaci, MD, FASNDrexel University College of MedicinePhiladelphia, PA

Rajiv Dhamija, MDWalk in Medical CareArtesia, CA

Susan R. DiGiovanni, MDVirginia Commonwealth UniversityRichmond, VA

Francis Dumler, MDWilliam Beaumont HospitalRoyal Oak, MI

Mahmoud T. El-Khatib, MD, PhD, FASNUniversity of Cincinnati Medical CenterCincinnati, OH

Lynda A. Frassetto, MD, FASNUniversity of California at San FranciscoSan Francisco, CA

Duvuru Geetha, MDJohns Hopkins UniversityBaltimore, MD

Carl S. Goldstein, MDRobert Wood Johnson Medical SchoolNew Brunswick, NJ

Nabil G. Guirguis, MDKidney Dialysis and Transplant GroupBridgeport, WV

Pawan K. Gupta, MDAltoona Regional Health SystemAltoona, PA

Carsten Hafer, MDUniversity of HannoverHannover, Germany

Richard N. Hellman, MDIndiana University School of MedicineIndianapolis, IN

Ekambaram M. Ilamathi, MD, FASNSuffolk Nephrology ConsultantsStony Brook, NY

Viswanathan S. Iyer, MD, FASNAKD-HTN LLCHarrisburg, PA

Bernard G. Jaar, MDJohns Hopkins Medical Institutions andNephrology Center of MarylandBaltimore, MD

Avanelle V. Jack, MDLouisiana State University HealthSciences CenterNew Orleans, LA

Sharon L. Karp, MDIndiana University School of MedicineIndianapolis, IN

Pranay Kathuria, MD, FASNUniversity of Oklahoma College of MedicineTulsa, OK

Quresh T. Khairullah, MD, FASNSt. Clair Specialty PhysiciansDetroit, MI

Apurv Khanna, MDSUNY Upstate Medical UniversitySyracuse, NY

Ramesh Khanna, MDUniversity of Missouri at ColumbiaSchool of MedicineColumbia, MO

Edgar V. Lerma, MD, FASNUniversity of Illinois at ChicagoCollege of MedicineChicago, IL

Meyer D. Lifschitz, MDShaare Zedek Medical CenterJerusalem, Israel

Philippe S. Madhoun, MDChu CharleroiCharleroi, Belgium

Jolanta Malyszko, MD, PhD, FASNMedical UniversityBialystok, Poland

Naveed N. Masani, MDWinthrop University HospitalMineola, NY

Hanna W. Mawad, MD, FASNUniversity of Kentucky Medical CenterLexington, KY

Pascal Meier, MD, FASNCentre Hospitalier Universitaire VaudoisLausanne, Switzerland

Beckie Michael, DO, FASNMarlton Nephrology and HypertensionMarlton, NJ

Shahriar Moossavi, MD, PhDWake Forest University BaptistMedical CenterWinston-Salem, NC

Scott R. Mullaney, MDUniversity of California at San DiegoSan Diego, CA

Quaid J. Nadri, MD, FASNKing Faisal Specialist Hospital andResearch CenterRiiyadh, Saudi Arabia

NephSAP®

Volume 10, Number 1, January 2011

Page 9: 2011.01 - Geriatric Nephrology

Suzanne M. Norby, MD, FASNMayo ClinicRochester, MN

Michal Nowicki, MDMedical University of ŁodzŁodz, Poland

Macaulay A. Onuigbo, MD, FASNMayo ClinicEau Claire, WI

Than N. Oo, MDNephrology CenterKalamazoo, MI

Kevin P. O’Reilly, MDColumbus Nephrology, Inc.Columbus, OH

Malvinder S. Parmar, MB, MS, FASNNorthern Ontario School of MedicineTimmins, ON, Canada

Pairach Pintavorn, MD, FASNEast Georgia Kidney and HypertensionAugusta, GA

Paul H. Pronovost, MD, FASNYale University School of MedicineWaterbury, CA

Mohammad A. Quasem, MD, FASNState University of New YorkBinghamton, NY

Wajeh Y. Qunibi, MDUniversity of Texas Health Sciences CenterSan Antonio, TX

Venkat Ramanathan, MD, FASNBaylor College of MedicineHouston, TX

Karthik M. Ranganna, MDDrexel University College of MedicinePhiladelphia, PA

Pawan K. Rao, MD, FASNSt. Joseph’s Hospital Health CenterSyracuse, NY

Joel C. Reynolds, MD, FASNBrooke Army Medical CenterSan Antonio, TX

Robert M.A. Richardson, MDUniversity of TorontoToronto, ON, Canada

Bijan Roshan, MDJoslin Diabetes CenterHarvard Medical SchoolBoston, MA

Abinash C. Roy, MDUniversity of Utah School of MedicineSaint George, UT

Mario F Rubin, MDMassachusetts General HospitalBoston, MA

Ehab R. Saad, MD, FASNMedical College of WisconsinMilwaukee, WI

Mohammad G. Saklayen, MDWright State University Medical SchoolDayton, OH

Ramesh Saxena, MD, PhDUniversity of Texas SouthwesternMedical CenterDallas, TX

Gaurang M. Shah, MDLong Beach VA Healthcare SystemLong Beach, CA

Robert J. Shay, MD, FASNEast Georgia Kidney andHypertension GroupAugusta, GA

Rolf A.K. Stahl, MDUniversity of HamburgHamburg, Germany

Harold M. Szerlip, MD, FASNMedical College of GeorgiaAugusta, GA

Bekir Tanriover, MDDialysis Nephrology AssociatesDallas, TX

Tushar J. Vachharajani, MD, FASNWake Forest UniversitySchool of MedicineWinston-Salem, NC

Allen W. Vander, MD, FASNKidney Center of South LouisianaThibodaux, LA

Luigi Vernaglione, MDM. Giannuzzi HospitalManduria, Italy

Shefali Vyas, MDSaint Barnabas Medical CenterLivingston, NJ

Alexander Woywodt, MD, FASNLancashire Teaching Hospitals NHSFoundation TrustPreston, United Kingdom

Page 10: 2011.01 - Geriatric Nephrology

Program Mission and ObjectivesThe mission of the Nephrology Self-Assessment Program (NephSAP) is to regularly provide a vehicle that will be useful for clinicalnephrologists who seek to renew and refresh their clinical knowledge and diagnostic and therapeutic skills. This Journal consists of aseries of challenging, clinically oriented questions based on case vignettes, a detailed Syllabus that reviews recent publications,and an Editorial on an important and evolving topic. Taken together, these parts should assist individual clinicians under-taking a rigorous self-assessment of their strengths and weaknesses in the broad domain of nephrology.

Accreditation and Credit DesignationThe American Society of Nephrology is accredited by the Accreditation Council for Continuing Medical Education to provide con-tinuing medical education for physicians.

The ASN designates this educational activity for a maximum of 8.0 AMA PRA Category 1 Credits™. Physicians should only claimcredit commensurate with the extent of their participation in the activity.

Continuing Medical Education (CME) Information

CME Credit: 8.0 AMA PRA Category 1 Credits™

Date of Original Release: January 2011Examination Available Online: on or before Monday, January 10, 2011Audio Files Available: No audio files for this issue.

CME Credit Eligible Through: December 31, 2011

Answers: Correct answers with explanations will be posted on the ASN website in January 2012 when the issue is archived.UpToDate Links Active: January and February 2011

Core Nephrology question links active: No core questions for this issue

Target Audience: Nephrology Board and recertification candidates, practicing nephrologists, and internists.

Method of Participation:● Read the syllabus that is supplemented by original articles in the reference lists, and complete the online self-assessment

examination.● Examinations are available online only after the first week of the publication month. There is no fee. Each participant is

allowed two attempts to pass the examination (�75% correct).● Upon completion, review your score and incorrect answers.● Your CME certificate can be printed immediately after completion.● Answers and explanations are provided with a passing score and/or after the second attempt.● CME Credit will be posted to your transcript within 48 hours after checking the attestation box.

Instructions to access the Online Examination and Evaluation:● Go to the ASN website: www.asn-online.org.● Click the CME tab at the top of the homepage.● Click the ASN CME Center button on the left side of the page.● Click on to the ASN CME Center icon.● Login to the ASN website.● Select Claim Credits for the NephSAP topic-activity you would like to complete.● Complete the NephSAP examination.● Complete the evaluation.● Enter the number of CME credits commensurate with your participation in the activity.● Check the box attesting that you have completed this activity.● You can print your CME certificate immediately.● CME credit will be posted to your transcript within 48 hours.● View or print your full transcript anytime at “My CME Center.”

NephSAP®

Volume 10, Number 1, January 2011

Page 11: 2011.01 - Geriatric Nephrology

Instructions to Obtain American Board of Internal Medicine (ABIM) Maintenance of Certification(MOC) Points:

Each issue of NephSAP provides 10 MOC points. Respondents must meet the following criteria:● Be certified by ABIM in internal medicine and/or nephrology and must be enrolled in the ABIM–MOC program

via the ABIM website (www.abim.org).● Take the self-assessment examination within the timeframe specified in this issue of NephSAP.● Designate the issue for MOC points by clicking on the MOC link on the CME certificate page after passing the examination.

You will be leaving the ASN site and transferring the information directly to the ABIM in real-time.● Provide your ABIM Certificate ID number and your date of birth.● You will receive a confirmation message from the ABIM indicating the receipt of your information.

MOC points will be applied to only those ABIM candidates who have enrolled in the program. It is your responsibility to completethe ABIM MOC enrollment process.

Instructions to access the ASN website, NephSAP, and the UpToDate link

Compatible Browser: The ASN website (asn-online.org) has been formatted for cross-browser functionality, and shoulddisplay correctly in all modern web browsers. We recommend that you use Internet Explorer.

Monitor Settings: The ASN website was designed to be viewed in a 1024 � 768 or higher resolution.

Technical Support: If you are having difficulty viewing any of the pages, please refer to the ASN technical support pagefor possible solutions. If you continue having problems, contact Hal Nesbitt at [email protected] provides an additional source of information that should help you answer up to 5 selected NephSAPquestions from each issue. The link is free and will remain active for the first 60 days after publication for eachissue.

● On the ASN home page, double click on the NephSAP link (NephSAP cover) on the bottom side of the page.● On the NephSAP page, click on the UpToDate button on the left hand side to access the current links.

NephSAP®

Volume 10, Number 1, January 2011

Page 12: 2011.01 - Geriatric Nephrology

Disclosure InformationThe ASN is responsible for identifying and resolving all conflicts of interest prior to presenting any educational activity to learners to ensure thatASN CME activities promote quality and safety, are effective in improving medical practice, are based on valid contents, and are independent of thecontrol from commercial interests and free of commercial bias. All faculty are instructed to provide balanced, scientifically rigorous and evidence-based presentations. In accordance with the disclosure policies of the Accreditation Council for Continuing Medical Education (ACCME) as well asguidelines of the Food and Drug Administration (FDA), individuals who are in a position to control the content of an educational activity are re-quired to disclose relationships with a commercial interest if (a) the relation is financial and occurred within the past 12 months; and (b) the individ-ual had the opportunity to affect the content of continuing medical education with regard to that commercial interest. For this purpose, ASN considerthe relationships of the person involved in the CME activity to include financial relationships of a spouse or partner. Peer reviewers are asked toabstain from reviewing topics if they have a conflict of interest. Disclosure information is made available to learners prior to the start of any ASNeducational activity.

Agarwal, Rajiv—Research funding: Abbott; Consultant/scientific advisor: Rockwell Medical, Watson Pharma; Honoraria: Abbott, Astra-Zeneca, MerckCohen, David J.—Research funding: Life Cycle Pharma, Novartis, Roche, Wyeth; Honoraria: Bristol-Myers-Squibb, Novartis, RocheEmmett, Michael—Honoraria: ABIM, Albert Einstein Medical Center, American Renal Associates, Best Doctors, Braintree Labs, Brown

University, Fresenius, Partners-Boston, Renal Ventures, Shire, Temple Medical, TIPS; Editorial board membership: American Journal ofCardiology, Baylor Proceedings

Fried, Linda F.—Research funding: Merck, Reata; Honoraria: PfizerFuchs, Elissa (Medical Editor)—noneGlassock, Richard J.—Consultant: Bio-Marin (inactive), Eli Lilly (active), FibroGen (inactive), Genentech (active), Lighthouse Learning (active),

Novartis (active), QuestCor (active), Wyeth (inactive); Ownership interests: LaJolla Pharmaceutical, Reata Inc.; Honoraria: American Society ofNephrology, various medical schools for lectures and/or visiting professor; Membership board of directors/scientific advisor: American RenalAssociates, Los Angeles Biomedical Institute, University Kidney Research Associates (UKRO), Wyeth; Editorial board: UpToDate, AmericanJournal of Nephrology; Royalties: Oxford University Press; Paid expert testimony: Various legal firms regarding product liability

Goldfarb, Stanley—Consultant: Bayer, GE Healthcare; Ownership interests: Polymedix; Honoraria: GE Healthcare, FreseniusLiu, Kathleen D.—Ownership interest: AmgenMartin, Kevin J.—Consultant: Abbott, Cytochroma, Kai, Shire; Honoraria: Abbott, Genzyme, Kai, Shire; Scientific advisor: Abbott,

Cytochroma, KaiMehrotra, Rajnish—Research funding: Amgen, Baxter, Shire; Consultant: Novartis; Honoraria: AMAG, Baxter, Healthcare ShireMurray, Patrick T.—Employment: spouse, Merck, Sharpe, & Dohme (Europe); Consultant/honoraria/research funding/

scientific advisor: Abbott Laboratories (USA), Argutus Medical (UK), FAST Diagnostics (USA); NxStage Medical (USA).Nachman, Patrick H.—Honoraria: QuestCor; Multicenter clinical trial participation: OtsukaPeixoto, Aldo J.—Consultant: Abbott, Sanofi-Aventis; Research funding: Pulsemetric; Honoraria: Boehringer-Ingelheim, Merck, Novartis,

Takeda; Scientific advisor/membership: Associate Editor–Blood Pressure Monitoring; Editorial Board: American Journal of Nephrology,Brazilian Journal of Nephrology

Sterns, Richard H.—Consultant: Astellas, Otsuka; Honoraria: Astellas, Otsuka; UpToDate; Scientific advisor: UpToDateTownsend, Raymond R.—Consultant: Daiichi-Sankyo, GlaxoSmithKline, Merck, Nicox, Novartis, Roche; Research funding: Novartis;

Honoraria: American Society of Hypertension, National Kidney FoundationVella, John P.—noneGuest Co-Editors:Glassock, Richard J.—see aboveJassal, Sarbjit Vanita—Consultant: Amgen; Honoraria: Baxter, Johnson and JohnsonO’Hare, Ann M.—Honoraria: Japanese Society for Footcare; Editorial board: Acquired Cystic Kidney Disease, American Journal of Kidney

Disease, Clinical Journal of the American Society of Nephrology; Royalties: UpToDateOreopoulos, Dimitrios G.—noneRosner, Mitchell H.—noneSwidler, Mark A.—noneWilliams, Mark E.—noneWiggins, Jocelyn—Honoraria: American Society of Nephrology, Association of Specialty ProfessorsEditorial authors:Bailey, James—noneSands, Jeff M.—Ownership interests: ATT, Chevron, Diageo, Exxon Mobil, Frontier Communications, GE, T Rowe Price, Verizon; Honoraria:

Boston University, Tulane University, University of Otago, University of Aarhus, University of Michigan, Washington University, Wayne StateUniversity; Educational Testing Consultants; NIDDK, Satellite Healthcare/Coplan Extramural Grant Program, Southern Societies for ClinicalInvestigation; Scientific advisor/memberships: American Heart Association Kidney Council Past-Chair, American Physiological Society FinanceCommittee Chair; Editorial board: American Journal of Physiology, Clinical and Translational Science, Hypertension-consulting editor,International Journal of Nephrology and Renovascular Disease-honorary editorial board, Journal of the American Society of Nephrology,NIDDK Board of Scientific Counselors, Satellite Healthcare/Coplan Extramural Grant Program Scientific Advisory Board

Schlanger, Lynn E.—none

Commercial SupportThere is no commercial support for this issue.

NephSAP®

Volume 10, Number 1, January 2011

Page 13: 2011.01 - Geriatric Nephrology

EditorialGeriatric Nephrology: A Missing Area of Nephrology’s Expertise

Lynn E. Schlanger, MD, James L. Bailey, MD, and Jeff M. Sands, MDRenal Division, Emory University, Atlanta, Georgia

The medical community is aware of the “agingcrisis” that is confronting society as it enters the 21stcentury. As early as the 1960s, the importance oftraining health care professionals in geriatrics wasrecognized (1). It was estimated that by 2000, thosewho were aged �65 years would account for 12.4% ofthe population (35 million). By 2030, it is estimatedthat there will be a 19.6% increase (71 million) inindividuals who are older than 65 years; and by 2050,one in five adults will be older than 65 years (2–4). InEurope, there is expected to be a similar growth rate inthe elderly population from 6.9% of the population(550 million) in 2000 to 12% (973 million) of thepopulation by 2030 (3,5). Those who are aged �80years have the fastest rate of growth, with an expectedincrease to 19.5 million by 2030. This is approxi-mately a sevenfold increase from 2000. This growth isattributed to the coming of age of the “baby boomers”and advancement in medical care and in technology. Asimilar trend is found in developing countries, with alag time of 20 years (6).

Parallel with this growth in the elderly popula-tion is an increase in the incidence and prevalence ofchronic kidney disease (CKD) and of end-stage kidneydisease (ESKD) (7–9). The National Health and Nu-tritional Examination Survey (NHANES) examinedthe prevalence of stages 1 throguh 4 CKD from 1988to 1994 to 1998 to 2004 in noninstitutionalized civil-ians. There was an increase in the prevalence of CKDfrom 10.3 to 13.1% of the population with the greatestpercentage increase observed in the group aged �70years, going from 37 to 47% (8). Although the growthtrend in the number of elderly contributed to theincreased prevalence of CKD, an even greater contri-bution came from comorbidities such as diabetes,hypertension, and obesity (8).

Regardless of practice setting, nephrologists willtake care of the elderly. It is estimated that they will

account for at least 50% of patient encounters (10). Alarge retrospective study from Australia evaluated thetrend in nephrology referrals after the implementationof the four-variable Modification of Diet in RenalDisease (MDRD) GFR formula (11). There was a 40%increase in nephrology referrals for CKD to tertiaryand regional renal services as well as private practices(11). Diabetes and CKD in older patients were themost common reasons for these referrals, and 69% ofthese referrals were from primary care physicians.This trend will continue as the aging population con-tinues to grow and the incidence of diabetes amongthem also increases. Accompanying this trend is therise of health care expenditures. Today, 80% of healthcare expenditures are on chronic diseases (12,13),including CKD and ESKD (14). Using the 1992 to1998 Medicare current beneficiary survey, the esti-mated cumulative health expenditure was slightly lessin a healthy 70-year-old individual as compared with a70-year-old institutionalized individual (15). The im-proved health in the aging and increased longevity willnot have an effect on these cumulative heath careexpenditures (15).

The emphasis of care for the elderly is differentfor nephrologists and geriatricians. Nephrologists areconfronted with both chronic and acute medical issuesin elderly patients, such as acute kidney injury (16),electrolyte abnormalities (see review in reference 17)cardiovascular disease (7,18), glomerulonephritis(19,20), and CKD (2,8). Within the nephrology sub-specialty is a divergence between the age groups in themode of clinical presentation, clinical course, andclinical outcome.

Age and stage of kidney disease seem to modifyprognosis. In an observational study of 206,622 vet-erans with stages 3 through 5 CKD, O’Hare et al. (7)found that 4.4% developed ESKD, 28% of whom were�75 years of age. At all CKD stages, the rate of death

Nephrology Self-Assessment Program - Vol 10, No 1, January 2011

1

Page 14: 2011.01 - Geriatric Nephrology

and ESKD was inversely related to estimated GFR.The elderly were more likely to die than progress toESKD as compared with a younger cohort. Moreover,the incidence of death at all stages of kidney diseasewas higher in the elderly as compared with theyounger cohort. The likelihood of progressing andsurviving to ESKD depended more on age of theindividual than other factors (7).

Kidney biopsies are being performed morereadily in the very elderly because of the likelihood ofdiscovering treatable kidney disease. Although a smallpercentage of kidney biopsies are being performed inthe very elderly (3.1%), most of them display abnor-mal findings (19). Fully forty-six percent of kidneybiopsies performed on those individuals 80 yr or olderrevealed acute kidney injury. A third of those individ-uals with AKI had pauci-immune glomerulonephritis(33%) (19). This finding has resulted in a paradigmshift in the way kidney disease is approached in theelderly and very elderly. More diagnostic proceduresare being performed; extended-donor criteria havebeen instituted; older donors are likely to be chosen forkidney transplant; and more bench and clinical re-search centers around aging.

The focus of geriatricians is different. They dealwith the “Giants of Geriatrics “(immobility, instabil-ity, incontinence, and impaired intellect/memory), aswell as socioeconomic and psychologic issues thataffect the patient’s daily activities and the subsequentimpact on family members. It has been shown thatelderly persons with CKD and ESKD have functionaland cognitive issues that are more likely to be ad-dressed by geriatricians than by nephrologists. Robertset al. (21) interviewed 47 elderly patients from a singledialysis unit with regard to the number of episodes ofsyncope, postural hypotension, and falls. A high per-centage of individuals reported having symptoms: 20reported pre-syncopal or syncopal episode, 34 re-ported dizziness, and 14 had a fall. All of the patientstested had abnormal autonomic function. The authorconcluded that caution should be taken in lowering apatient’s dry weight and with being overly aggressivein controlling BP.

Cognitive skills decrease with worsening renalfunction. Recently, a large study (22) evaluated thetrajectory changes in daily activities in the very elderlyfrom the nursing home by using data from theUSRDS. The Minimum Data Set-Activities of DailyLiving (MDS-ADLS) were evaluated three months

before initiation of dialysis and then 3 mo and 12 moafter dialysis initiation. There was a decline in dailyactivities in over 60% of the nursing home patients(22). By the 12th month, 58% of the patients had died.This was higher than the expected national average of16% and the functional status was maintained in only13.8% of the patients (22). A cross-sectional studylooked at cognitive ability in individuals on hemodi-alysis (23) and noted moderate (36.1%) and severeimpairment (37.3%) among patients on hemodialysis.Interestingly only 2.9% had a documented history ofcognitive impairment (23). Factors associated withsevere cognitive impairment included stroke (OR1.95; P � 0.03), equilibrated Kt/V � 1.2 (OR1.67;P � 0.05), and education � 12 yr (OR 0.32; P �0.01). In chronic kidney disease there appears to be adirect decline in cognitive ability in the elderly (24).The Rush Memory and Aging Project, which was alongitudinal study of a community of 866 elderlywithout dementia (24), showed that a decrease in thebaseline estimated GFR was associated with a declinein cognitive ability (P � 0.017) for episodic memory,semantic memory, and working memory (24). Therewas also a loss of independence in more than 30% ofoctogenarians within the first six month after initiatingdialysis (25).

Due to the medical complexity and the socioeco-nomic, psychologic, and functional issues involved intaking care of an elderly patient, familiarity withgeriatrics is required to provide optimal care. In 1968the American Board of Medicine (AMA) approvedGeriatrics as a medical subspecialty and the firstAmerican geriatric fellowship program was estab-lished in 1972 (26). Presently, there are 120 geriatricfellowship programs, but geriatric fellows make upjust a small percentage of the total number of medicalgraduates (1%) (27). Of these, an even smaller per-centage will pursue a career in academic medicine.This is an inadequate number of graduates, far lessthan are required to meet the demands for teaching andcaring for the anticipated increase in the elderly andvery elderly population. Such small numbers of geri-atricians necessarily limits the exposure of the subspe-cialty to medical trainees. Despite the AmericanBoards of Internal Medicine and Family Practice re-ducing the required number of years of geriatric fel-lowship to one year (28), shortages of geriatriciansexist. In 2001–2002 only 69% of the available geriat-ric fellowship positions were filled (29). If these trends

2 Nephrology Self-Assessment Program - Vol 10, No 1, January 2011

Page 15: 2011.01 - Geriatric Nephrology

continue, it will be an insurmountable task for geria-tricians to care for the aging population. Only a smallproportion of practicing health care providers have hadany formal training in geriatrics. Out of 650,000 prac-ticing physicians in the U.S., less than 9,000 aregeriatricians, or about 2.5 geriatricians per 10,000elderly patients (28). That number is expected to fall toabout 6,000 in the near future (28). Fewer than 3% ofcurrent medical students take any elective geriatriccourses (30). Because of the lack of geriatricianscaring for the aging population, there is a need forfamily practitioners, general internists, and subspecial-ists to participate in their care.

In 1985 the first symposium on Geriatric Ne-phrology was held in Toronto to discuss the medicalneeds of the growing elderly population (31). The FirstInternational Conference on Geriatric Nephrology andUrology commenced in 1990, which led to the forma-tion of the International Society of Geriatric Nephrol-ogy. This interest in geriatric nephrology began towane; member attendance decreased at meetings andsubscriptions declined to the International Journal ofGeriatric Nephrology and Urology. The latter resultedin this journal being consolidated with the Journal ofInternational Urology and Nephrology (31).

As a result of the concern for the decline in thenumbers of geriatricians, the John A. Hartford Foun-dation provided funding to the American GeriatricSociety (AGS) in 1994 for an initiative entitled “Inte-grating Geriatrics into the Subspecialties of InternalMedicine (IGSIM)” (32). The aim was to improve andincorporate geriatric learning in all subspecialties byallocating a portion of the funds toward geriatricretreats (GERs) where geriatricians and subspecialistsfrom other areas would meet to discuss commonissues. As a result, the T. Franklin Williams CareerDevelopment Award was instituted for junior facultywith specific interests in gerontology and geriatrics bythe ASN and the Association of Specialty Professors(ASP). This was funded by the Atlantic Philanthropiesand the Hartford Foundation. Since 2003 a total of 9scholarships have been awarded (32).

Geriatric retreats were created to strengthen ge-riatric education in all subspecialties of internal med-icine (33). The retreats included leaders in internalmedicine subspecialties and geriatricians; they havemet over a six year period. In 1998, the NephrologyGeriatric Retreat was held in Alberta, Canada (33).The goals of JAHF/AGS was to increase the aware-

ness of geriatrics in the subspecialties, improve geri-atric education, increase research in geriatrics, recruitjunior and midlevel faculty with an interest in geriat-rics, provide more informative material at national andregional subspecialty meetings, and develop a geriatricnephrology curriculum. In 2004, the ASP and IGSIMsent out surveys to 10 subspecialty societies to deter-mine the impact that the GERs had on their specificsubspecialty geriatric curriculum (33). They focusedon four areas: 1) topics at the 2003 and 2004 annualmeetings related to geriatrics; 2) society journal arti-cles related to geriatrics from 1995 to 2003 and in2004; 3) societal organizational structure relating togeriatrics; and 4) continuing medical education (CME)on the topic of geriatrics. Although the ASN hadgeriatric material at the 2003 and 2004 annual meet-ings, available geriatric CME credits (NephSAP), re-certification resources related to geriatrics, and theASP Franklin Williams Scholar Program, the numberof geriatric specific articles had not increased from1995 to 2004. This was based on a literature searchlimited to JASN. Moreover, the ASN had no specificcommittees focused on geriatrics or aging; in 2008, theASN established one. More recently, the number ofarticles on aging and nephrology has surpassed thenumber published between 1995 and 2004.

In 2005 the ACGME (Accreditation Council forGraduate Medical Education) recommended that “ge-riatric nephrology” be added to the core curriculum. Inpart, their treatise stated that “fellows must have for-mal instruction, clinical experience and demonstratecompetence in prevention, evaluation, and manage-ment of geriatric aspects of nephrology, includingdisorders of the aging kidney and urinary tract. Thefellows must receive formal instruction in geriatricmedicine, including physiology and pathology of theaging kidney; and drug dosing and renal toxicity in theelderly patient” (31).

In response to the ACGME mandate, Dr. DonaldKohan, Chair of the ASN Training Program Directors(TPD) Executive Committee, invited geriatric expertsto form a committee to design and establish a Nephrol-ogy Curriculum for Nephrology Trainees by 2008.This was to be used as a learning tool. The GeriatricTask Force was co-chaired by Drs. Dimitrios Oreo-poulos and Joycelyn Wiggins. The Nephrology Cur-riculum on geriatrics resulted from these efforts andcontains thirty seven 5 to 6 page chapters on topicsrelating to geriatric nephrology. It is available through

Nephrology Self-Assessment Program - Vol 10, No 1, January 2011 3

Page 16: 2011.01 - Geriatric Nephrology

the ASN website (www.asn-online.org). Each chapterhas questions touching on pertinent points. In addition,the ASN Renal Week held a 2-d course in 2008, 2009,and again in 2010 on Geriatric Nephrology. Thiscourse catered to both renal fellows and nephrologists.More recently, the ASN Geriatric Nephrology Advi-sory Committee was formed.

With all of these committees, grants, and orga-nizations created to improve geriatric educationamong trainees and faculty, and to increase interest ingeriatric research, has there been any improvement?The International Association of Gerontology and Ge-riatrics, and European Union of Medical Specialties-Europe Section sent out a survey to 47 Europeancountries in 2006 to study the changes in geriatriceducation that had occurred in undergraduate and postgraduate training since the original survey was done in1994 (34). Thirty three of 47 institutions reportedhaving a training program in geriatrics. Of these 31completed the survey. Following the initial surveythere was a major improvement in geriatric education.The number of medical schools with a chair in geri-atrics had increased by 44% going from 74 to 132.There was also an increase in undergraduate andpost-graduate education on the topic by 23% and 19%,respectively (34). Although there were improvements,the mandatory continuing medical education was onlyenforced in 10/31 countries. In the United States, theOffice of Geriatric Medicine and Institute for Study ofHealth at the University of Cincinnati conducted anational survey to evaluate ACGME accredited inter-nal medicine programs to determine what changes hadoccurred in the geriatric curriculum in 2005 (35). Sixtypercent of the internal medicine program directorsresponded to the survey. Changes from the originalsurvey in 2002 were compared with survey resultsfrom 2005. There was variability in geriatric clinicalinstruction ranging from 2 wk to 6 wk with themajority at least 4 wk (62%). There was no increase indidactics instruction in geriatrics between 2002 and2005. The number of full time faculty members teach-ing geriatrics rose from a mean of 2.2 in 2002 to meana 3.5 in 2005 (35). A similar national survey ofnephrology program directors on the extent of theincorporation of geriatric nephrology in the core cur-riculum remains to be completed. As awareness growsin Europe and the United States, efforts are increasingto improve the education in geriatrics in future years,but much work still needs to be done.

In 2003 medical students and residents in internalmedicine were interviewed by professional facilitators toevaluate the competencies and to explore the gaps in thegeriatric curriculum (36). Although the medical studentsand residents felt intimidated by the complexity of med-ical issues surrounding Geriatrics, were inhibited intrying to converse with patients with cognitive impair-ment, and were unfamiliar with availability of socialprograms for geriatric patients at the time of hospitaldischarge (36), all reported feeling uplifted and en-joyed talking with the elderly patient.

There have been a few national surveys that haveattempted to evaluate nephrology training programs’education on issues related to end-of-life (37–39). Asthe typical renal fellow cares for older patients withmuch co-morbidity, that fellow encounters death on aregular basis (38). When second year nephrology fel-lows were asked to grade how well various topics weretaught on a scale from 0 to 10 with 0 being the leastand 10 the most taught, end-of-life teaching received alow score of 3.8 � 2.6 while hemodialysis received ahigh score of 8.9 � 1.5 even though the majority(99%) felt it was important to receive instruction onend–of-life care (38). In an internet based survey ofrecent nephrology graduates who had completed theirtraining from 2004–2008 (39), nearly 49% felt thatthey were ill prepared and not well trained in end-of-life care (39).

On a local level it is extremely difficult to findinformation pertaining to the core geriatric curricu-lum in individual nephrology fellowship programs.Some information can be obtained on the internetbut it is sparse with minimal mention in a program’sdescription. Mount Sinai’s nephrology fellowshipprogram has established Geriatric Nephrology andGeriatric/Palliative Care Program. Other institutionssuch as the University of California at Davis HealthSystem offers training in geriatric nephrology but 25%of the nephrology fellowship programs in the UnitedStates offer no geriatric fellowship program. More-over, the depth of the training in geriatric nephrologyvaries greatly from one program to the next since thereare no strict guidelines as seen in internal medic-ine residency programs (35). Hopefully, there willbe uniform guidelines in the future to ensure adequategeriatric education of our trainees and more avail-able geriatric training grants for junior and midlevelfaculty.

4 Nephrology Self-Assessment Program - Vol 10, No 1, January 2011

Page 17: 2011.01 - Geriatric Nephrology

References1. Libow L: A fellowship in geriatric medicine. J Am Geriatr Soc 20:

580–584, 19722. Stevens LA, Coresh J, Levey AS: CKD in the elderly: Old questions

and new challenges—World kidney day 2008. Am J Kidney Dis 51:535–537, 2008

3. Centers for Disease Control and Prevention: Public health and aging:Trends in aging—United States and worldwide. MMWR 52: 101–106,2003

4. National Center for Health Statistics: Health, United States, 2003,Hyattsville, MD, US Department of Health and Human Services,Centers for Disease Control and Prevention, 2003

5. Hallan SI, Coresh J, Astor BC, Asberg A, Powe NR, Romundstad S,Hallan HA, Lydersen S, Holmen J: International comparison of therelationship of chronic kidney disease prevalence and ESRD risk.J Am Soc Nephrol 17: 2275–2284, 2006

6. Gutierrez-Robledo LM: Looking at the future of geriatrics care indeveloping countries. J Gerontol 57A: M162–M167, 2002

7. O’Hare AM, Cho AI, Bachetti P, Garg AX, Kaufman JS, Walter LC,Mehta MA, Steinman MA, Allon M, McClellan WM, Landefeld CS:Age affects outcomes in chronic kidney disease. J Am Soc Nephrol18: 2758–2765, 2007

8. Coresh J, Selvin E, Stevens LA, Manzi J, Kusck JW, Eggers P, VanLente F, Levey AS: Prevalence of chronic kidney disease in theUnited States. JAMA 298: 2038–2047, 2007

9. Whaley-Connell AT, Sowers JR, Stevens LA, McFarlane SI, ShlipakMG, Norris KC, Chen SC, Qiu Y, Wang C, Li S, Vassalotti JA,Collins AJ, Kidney Early Evaluation Program Investigators: CKD inthe United States: Kidney Early Evaluation Program (KEEP) andNational Health and Nutrition Examination Survey (NHANES)1999–2004. Am J Kidney Dis 51: S31–S20, 2008

10. Keith DS, Nichols GA Gullion CM, Brown JB, Smith DH: Longi-tudinal follow-up and outcomes among population with chronickidney disease in a large managed care organization. Arch Intern Med164: 659–663, 2004

11. Noble E, Johnson DW, Gray N, Hollett P, Hawley CM, Campbell SB,Mudge DW: The impact of automated eGFR reporting and educationon nephrology service referrals. Nephrol Dial Transplant 23: 3845–3850, 2008

12. Boult C, Christmas C, Durso S, Leff B, Boult LB, Fried LP:Perspective: Transforming chronic care for older persons. Acad Med83: 627–631, 2008

13. Medicare Payment Advisory Commission: A Data Book: HealthcareSpending and the Medicare Program, June 2003. Section 5: Na-tional health care and Medicare spending. Available at: http://www.medpac.gov/chapters/Jun10DataBookSec1.pdf. AccessedSeptember 2010

14. Smith DH, Gullion CM, Nichols G, Keith DS, Betz J: Cost of medicalcare for chronic kidney disease and comorbidity among enrollees ina large HMO. J Am Soc Nephrol 15: 1300–1306, 2004

15. Lubitz J, Beebe J, Baker C: Longevity and Medicare expenditures.N Engl J Med 332: 99–1003, 1995

16. Ishani A, Xue Jay L, Himmelfarb J, Eggers PW, Kimel PL, MolitorisBA, Collins AJ: Acute kidney injury increases risk of ESRD amongelderly. J Am Soc Nephrol 20: 223–228, 2009

17. Gregori J-AA, Nunez J-FM: Handling of water and electrolytes in thehealthy old. Rev Clin Gerontol 19: 1–12, 2009

18. O’Hare AM, Bertenthal D, Covinsky KE, Landefeld S, Sen S, MehtaK, Steinman MA, Borzecki A, Walter LC: Mortality risk stratifica-tion in chronic kidney disease: One size for all ages? J Am SocNephrol 17: 846–853, 2006

19. Moutzouris D-A, Herlitz L, Appel GB, Markiwitz GS, Freudetha B,Radhakrishnan J, D’Agati VD: Renal biopsy in the very elderly. ClinJ Am Soc Nephrol 4: 1073–1082, 2009

20. Nair R, Bell JM, Walker PD: Renal biopsy in patients aged 80 yearsand older. Am J Kidney Dis 44: 618–626, 2004

21. Roberts RG, Kenny RA, Brierley EJ: Are elderly hemodialysispatients at risk of falls and postural hypotension? Int Urol Nephrol35: 415–421, 2003

22. Kurella Tamura MK, Covinsky KE, Chertow GM, Yaffe K, Land-efeld CS, McCulloch CE: Functional status of elderly adults beforeand after initiation of dialysis. N Engl J Med 361: 1539–1547, 2009

23. Murray AM, Tupper DE, Knopman DS, Gilbertson DT, Pederson SL,Li S, Smith GE, Hochhalter AK, Collins AJ, Kane RL: Cognitiveimpairment in hemodialysis patients is common. Neurology 67:216–223, 2006

24. Buchman AS, Tanne D, Boyle PA, Shah RC, Leurgans SE, BennettDA: Kidney function is associated with the rate of cognitive declinein the elderly. Neurology 73: 920–927, 2009

25. Jassal SV, Chiu E, Hladunewich M: Loss of independence in patientsstaring dialysis at 80 years of age or older. N Eng J Med 361:1612–1613, 2009

26. Libow LS: Commentary: The first geriatric residency-fellowship inthe United States. J Gerontol Series A Biol Sci Med Sci 59: 1165–1166, 2004

27. Brotherton SE, Etzel SI: Graduate Medical Education, 2008–2009.JAMA 302: 1357–1372, 2009

28. Warshaw GA, Bragg EJ: The training of geriatricians in the UnitedStates: Three decades of progress. J Am Geriatr Soc 51[Suppl]:S338–S345, 2003

29. Warshaw GA, Bragg EJ, Shaull RW, Goldenbar LM, Lindsell CJ:Geriatric medicine fellowship programs: A national Study from theAssociation Directors of Geriatric Academic Program’sLongitudinalStudy of training and practice in geriatric medicine. J Am Geriatr Soc51: 1023–1030, 2003

30. Warshaw GA, Bragg EJ, Thomas DC, Ho ML, Brewer DE: Areinternal residency programs adequately preparing physicians to carefor the baby boomers: A national survey from the association ofDirectors of Geriatric Academic Programs Status of Geriatric Work-force Study. J Am Geriatr Soc 54: 1603–1609, 2006

31. Oreopoulus D, Wiggins J: Chapter 1: Geriatric nephrology has comeof age: At last. Available at: http://www.asn-online.org/education_and_meetings/geriatrics/OnlineGeriatricsCurriculum.pdf. Accessed Sep-tember 2010

32. Hazzard WR, Woolard N, Regenstreiff D: Integrating geriatrics intothe subspecialties of internal medicine: The Hartford Foundation/American Geriatrics Society/Wake Forest University Bowman GraySchool of Medicine Initiative. J Am Geriatr Soc 45: 638–640, 1997

33. Sonu IS, High KP, Clayton CP, Woolard NF, Hazzard WR: Anevaluation of geriatrics activities within internal medicine subspecial-ties. Am J Med 119: 995–1000, 2006

34. Michel J-P, Huber P, Cruz-Jentoft AJ: Europe-wide survey of teach-ing in geriatric medicine. J Am Geriatr Soc 56: 1536–1542, 2008

35. Warshaw GA, Thomas MD, Callahan EH, Bragg EJ, Shaull EW,Lindsell CJ, Goldenhar LM: A national survey on the current statusof general internal medicine residency education in geriatric medi-cine. J Gen Intern Med 18: 679–684, 2003

36. Drickamer MA, Levy B, Irwin KS, Rohrbaugh RM: Perceived needsfor geriatric by medical students, internal medicine residents, andfaculty. J Gen Intern Med 21: 1230–1234, 2006

37. Davison SN, Jhangri GS, Holley JL, Moss AH: Nephrologists’reported preparation for end of life decision making. Clin J Am SocNephrol 1: 1256–1262, 2006

38. Holley JL, Carmody SS, Moss AH, Sullivan AM, Cohen LM, BlockSD, Arnold RM: The need of end-of-life training in nephrology:National survey results of nephrology fellows. Am J Kidney Dis 42:813–820, 2003

39. Berns JS: A survey-based evaluation of self perceived competencyafter nephrology fellowship training. Clin J Am Soc Nephrol 5:490–496, 2010

Nephrology Self-Assessment Program - Vol 10, No 1, January 2011 5

Page 18: 2011.01 - Geriatric Nephrology

SyllabusGeriatric Nephrology: Another Milestone in a 25-Year Journey

Dimitrios G. Oreopoulos, MD, PhD, FRCPC, FACP,*† Richard J. Glassock, MD, MACP,‡

Sarbjit Vanita Jassal, MB, MD (UK), MRCP (UK), FRCP (C),*† Ann M. O’Hare, MD,§�

Mitchell H. Rosner, MD, FACP,¶ Mark A. Swidler, MD,**

Mark E. Williams, MD, FACP, FASN,††‡‡§§ and Jocelyn Wiggins, BM, BCh��

*Department of Medicine, University of Toronto, Toronto, Ontario, Canada; †Division ofNephrology, University Health Network, Toronto, Ontario, Canada; ‡Department ofMedicine, Geffen School of Medicine, University of California, Los Angeles, Laguna Niguel,California; §Department of Medicine, University of Washington, Seattle, Washington; �VAPuget Sound Healthcare System, Seattle, Washington; ¶Department of Medicine, Division ofNephrology, University of Virginia Health System, Charlottesville, Virginia; **Renal Division,Department of Medicine, and Department of Geriatrics and Palliative Medicine, Mount SinaiMedical Center, New York, New York; ††Department of Medicine, Harvard Medical School,Boston, Massachusetts; ‡‡Department of Medicine, Beth Israel Deaconess Medical Center,Boston, Massachusetts; §§Joslin Diabetes Center, Boston, Massachusetts; and ��Department ofMedicine, Medical Director Geriatric Center Clinics, University of Michigan, Ann Arbor,Michigan

The Co-Editors for this issue are members of the ASN Geriatric Nephrology Advisory Group.

Learning Objectives:1. To recognize the biology of the aging process

and how this influences kidney function2. To explain how aging influences the diagnosis of

chronic kidney disease (CKD) and susceptibilityto acute kidney injury (AKI)

3. To describe specific kidney diseases as they oc-cur in the elderly, including glomerular and vas-cular diseases and diabetic nephropathy

4. To summarize the medical and social issues re-lating to management of CKD and ESRD in theelderly, including palliative care and decisionmaking for dialysis therapy

Early EffortsIn May 1985, in response to the needs of the

increasing number of elderly patients who requireddialysis, the First International Conference on Geriat-ric Nephrology was held in Toronto, Ontario, Canada.This successful initial meeting was followed by thecreation of the International Society for Geriatric Ne-phrology with its own journal (International Journal

for Geriatric Nephrology and Urology) so that thoseconcerned about the elderly on dialysis would have aforum in which to exchange ideas and experiences.Five additional international meetings of this Societyfor Geriatric Nephrology were held in Salamanca,Spain; Lisbon, Portugal; Atlanta, Georgia; Thessal-oniki, Greece; and Antalya, Turkey.

Declining InterestDespite all these efforts and activities, the inter-

est among nephrologists concerning geriatric nephrol-ogy did not increase and, if anything, was decreasing.Membership in the society and participation in itsmeetings also were declining. Also, subscription toand submission of articles to the society’s journal werenot sufficient to sustain it, and, as a result, the pub-lisher now publishes these articles as a section of theInternational Urology and Nephrology Journal.

T. Franklin Williams Scholars ProgramIn 2002, The T. Franklin Williams Scholars Pro-

gram was launched to develop a new generation ofmedical subspecialists with expertise in aging and the

Nephrology Self-Assessment Program - Vol 10, No 1, January 2011

6

Page 19: 2011.01 - Geriatric Nephrology

care of the elderly within their area of specialization.This program was sponsored by the Association ofSpecialty Professors and funded by the John A. Hart-ford Foundation and Atlantic Philanthropies. Theyhave partnered with 11 professional societies, includ-ing the American Society of Nephrology (ASN) tofund 2-year career development awards for juniorfaculty who are willing to get a geriatrician mentor anddevote some of their research effort to aging-relatedproblems. Ten nephrologists have completed this pro-gram, and there are 2 more years of funding in theprogram. It is hoped that these young nephrologistswill be the leaders of geriatric nephrology in thefuture. Several of the Williams Scholars participatedin producing the ASN geriatric nephrology curriculumand will serve on the geriatric advisory board. At theend of the current cycle of funding, the NationalInstitute on Aging (NIA) has plans to continue thisprogram in a format that will be announced shortly.

Geriatric Nephrology and the National Institutesof Health

The National Institutes of Health has recognizedthe growing importance of aging in the kidney diseasepopulation. They have sponsored two workshops toaddress what is known about kidney disease in theelderly and to identify research gaps and make prior-ities for resources to address them. These workshopswere co-sponsored by the Association of SpecialtyProfessors, ASN, The American Geriatrics Society,NIA, and the National Institute of Diabetes and Di-gestive and Kidney Diseases. The first workshop fo-cused on CKD in the older adult and was held in May2008. A summary of their recommendations appearsin an article in the Journal of the American Society ofNephrology (JASN) (1). One result of this workshopwas an NIH program announcement for grants entitled“Renal Function and Chronic Kidney Disease in Ag-ing,” which is open until May 2012. A second work-shop took place in May 2010. It focused on AKI inolder adults. There will be a follow-up publication inJASN summarizing the group’s recommendations, andit is anticipated that there will be an additional pro-gram announcement linked to this topic.

Mandated Training of Geriatric Nephrology andthe Role of the ASN: A New Era

A spur to interest in geriatric nephrology wasstimulated by the exponential increase in the number

of patients who are older than 75 and require dialysisand the recognition after the introduction of estimatedGFR (eGFR) calculation that revealed the high per-centage of elderly who have CKD. Nephrologists arenow inundated by the elderly in their clinics anddialysis units and are forced to practice as amateurgeriatricians.

Recognizing that geriatric nephrology is essen-tial to nephrology training, the Accreditation Councilfor Graduate Medical Education (ACGME) mandatedin 2008 that “fellows must have formal instruction,clinical experience and demonstrate competence in theprevention, evaluation and management of geriatricaspects of nephrology.” In response to this require-ment, ASN invited a group of individuals to form acommittee to design a curriculum by identifying thetopics and authors to write the corresponding chapters.This curriculum is now available on line at the ASNweb site not only to the US nephrology community butalso to all interested individuals around the world.

Geriatric Nephrology Courses during the ASNRenal Week

In 2008, 2009, and now 2010, ASN has includeda 2-day course on geriatric nephrology. The first 2years, the authors of the curriculum presented theirchapters. Their presentations were audio taped and arenow available on the ASN web site (www.asn-online.org/education_and_meetings/geriatrics/). In additionto the formal course, the ASN Advisory Group pro-posed to the ASN Council suggestions for enhance-ments to the general meeting program.

Outreach to Other SocietiesThe ASN Geriatric Advisory Board is planning to

reach out to other specialty societies. We would like topartner with them at their annual professional societymeetings to raise awareness about the special challengesof older people with kidney disease. We plan to workwith the American Geriatric Society and the Society ofGeneral Internal Medicine and Family Medicine.

Geriatric Nephrology in NephSAPThis is the latest milestone in the development of

geriatric nephrology. The mission of NephSAP is toprovide a vehicle that is useful for clinical nephrolo-gists who seek to renew and refresh their clinicalknowledge and therapeutic skills. We are grateful tothe editors of NephSAP, who saw this as an appropri-ate time to present to the nephrology community a

Nephrology Self-Assessment Program - Vol 10, No 1, January 2011 7

Page 20: 2011.01 - Geriatric Nephrology

series of challenging clinically oriented questionsbased on vignettes and a detailed syllabus that reviewsrecent publications on geriatric nephrology. After aninitial meeting with the editor, we decided to cover thefollowing areas:

Y Biology of aging and its interaction with the kidneyY Decline of kidney function with age and the limitation

of various formulas in assessing eGFRY Hypertension and diabetic kidney disease in the elderlyY Glomerular disease in the elderlyY AKI in the elderlyY Kidney replacement therapies in the elderlyY Palliative care and geriatric advanced CKD manage-

ment

We expect that this issue will be an important mile-stone in the future promotion of geriatric nephrologyand will contribute to the appropriate care of manyelderly patients with CKD.

Introduction to the Biology of Aging and theKidney

The fountain of youth is a legendary spring thatcan restore the youth of anyone who drinks its waters.Tales of such a fountain have been recounted acrossthe world for thousands of years and appear in writingsby Herodotus, a Greek historian who lived in the 5thcentury BC. It also occurs in diverse European andearly Arabic literature. Despite the long and wide-spread interest in prolonging youth, serious researchinto the biology of aging is a very young field. Declinein tissue function and vitality was assumed to be theinevitable consequence of age and thus not worth study-ing. Aging is now known to be a complex biologicalprocess controlled by signaling pathways and transcrip-tion factors. In this section, we review what is knownabout the biology of aging and how it might be relevantto the kidney dysfunction that occurs with age in ourpatients.

The fastest growing group of people in theUnited States who have impaired kidney function isthe oldest age group. The population of those who areolder than 65 years in the United States is expected todouble over the next 20 years. Average life expectancyin 2004 was 75.2 years for men and 80.4 years forwomen; by 2015, it is expected to be 76.2 and 82.2years, respectively, and to continue growing. Duringthe 1990s population of those who were older than 85years was the fastest growing group at 38% growth peryear. This older age group is the largest consumer of

health care services (2). It has been known for morethan 50 years that renal function declines with age andthat glomerulosclerosis increases with age even in theabsence of comorbidities. The Baltimore LongitudinalStudy of Aging was started in 1958 and collected bothcross-sectional and longitudinal data on changes inrenal function with age (3). It was estimated that GFRdeclines approximately 10 ml/min per decade after age40, and this continues to be true. There is still debateabout whether this is truly due to aging or is caused byage-associated comorbidities such as hypertension.Patients with possible CKD are being referred tonephrologists in greater numbers since the introduc-tion of formulas for estimating GFR. Most clinicallaboratories supply an eGFR when a serum creatinineis ordered. National Health and Nutrition Survey(NHANES) data revealed estimates that 11% of theUS population has CKD, and this may be as high as30% in the older population (4). GFR declines withage in normal individuals; therefore, it can be difficultto distinguish age-related decrease in GFR from CKDin the elderly. Older patients with mildly decreasedGFR and a low risk for progressive decline in renalfunction need to be distinguished from those withprogressive disease because once identified, theyprobably do not need to be followed by a nephrologist.One of the most contentious issues in nephrologytoday is whether the decline in GFR is “normal pro-cess of aging” or “kidney disease” associated with thechronic diseases of aging (5,6). For older patients whotruly have CKD, age is an independent risk factor formortality of approximately 3% per year (7). Thissection briefly defines some of the issues related to“normal aging” biology and how they may contributeto the perceived epidemic of CKD. Cellular processesare addressed first followed by individual genes andpathways linked to aging.

Cellular Processes that Change with Age

Many cellular processes are involved inmaintaining normal cell phenotype and cellphysiology. Declines in efficiency of theseprocesses with age play a role in the de-cline of renal function with age.

Epigenetics Every cell in the body has the samecomplement of DNA. During nephrogenesis, regula-

8 Nephrology Self-Assessment Program - Vol 10, No 1, January 2011

Page 21: 2011.01 - Geriatric Nephrology

tory genes partition the genome into active and inac-tive domains, a process referred to as epigenetics. Thisallows the specification and maintenance of cellularphenotype. The body has two basic cell types: Stemcells, which are undifferentiated and capable of self-renewal, and pluripotency and somatic cells, whichmake up the distinctive individual cell types of eachorgan. As cell lineage and patterning occur under thecontrol of regulatory genes, chromatin domains aredefined through methylation and acetylation of his-tones. These domains may be activated or silenced todirect cell fate. These changes initiate differentiationand are maintained from one cell generation to thenext during cell division. An example of this is theWT1 regulatory gene, which is expressed early inembryogenesis and directs the development of theurogenital tract. Failure of WT1 transcriptional regu-lation results in unregulated cell division and causesthe Wilms’ tumor (nephroblastoma) of childhood. An-other major kidney regulatory DNA-binding protein isPax2, which plays a pivotal role in the development ofthe glomerulus and in directing the partitioning of thegenome into active and inactive domains. It is ex-pressed early in development and provides a linkbetween DNA and histone modification (8). In thisway, specific areas of DNA can be activated for celldifferentiation while other areas are silenced. Throughthe process of methylation and acetylation, thesechanges in the genome become both stable and heri-table. In this way, cell lineage becomes establishedand phenotype is maintained. Interestingly, once glo-merular development is complete, Pax2 is no longerexpressed. Once differentiation is established, it isself-maintaining. How do epigenetics affect aging inthe kidney? In cells that turn over on a regular basis,these epigenetic mechanisms are renewed at each celldivision. Podocytes probably do not replicate, sothey need to maintain their epigenetic characteristicsthroughout a lifetime. If their epigenetic marks areunstable over time, then we would expect to see loss ofcharacteristic phenotypic proteins and/or expression ofproteins that should have been silenced. In the Fischer344 rat model of the aging glomerulus, we do see bothof these processes (9). In our genetic screening processin aging Fischer 344 rats, we found 92 glomerulargenes that seemed to be silenced at 2 months but hadrobust expression by 24 months. The most strikingexample was prepronociceptin, a neurologic gene thathas been characterized as a modulator in pain signal-

ing pathways. It showed a �80-fold increase in ex-pression in the glomerulus between young rats and oldrats, suggesting that this area of chromatin had beensilenced during differentiation but had achieved ex-pression as a result of failure of the epigenetic regu-lation with age. In contrast, nephrin, a podocyte-specific protein, showed a decline in expression in thesame animals, again suggesting inefficiencies in epi-genetic patterning with age. Similar changes in geneexpression have been seen in aging brain studies (10).Neurones are also terminally differentiated, such aspodocytes, and depend on epigenetic partitioning oftheir genome to maintain phenotype.Micro RNAs Micro RNAs (MiRNAs) are one ofthe newest areas in regulatory biology. The firstmiRNA was discovered in 1993. They are single-stranded, noncoding (nontranslated) RNA moleculesof 22 to 25 nucleotides and are expressed in thenucleus as a 70-bp stem loop structure and cleavedinto its short active form in the cytoplasm. They act torepress mRNA and prevent translation and have beenshown to be extremely important in development,regulation of expression, and pathology in the kidney(11). Harvey et al. (12) showed that disruption ofmiRNA leads to rapid end-stage kidney disease. miR-195 has been shown to play an important role indiabetic nephropathy. The role of miRNAs in cellsenescence and aging is reviewed in detail by Grillariand Grillari-Voglauer (13). There are no specific dataon the role that they play in aging in the kidney, but asmore evidence accumulates about the role that theyplay in differentiation, cellular function, and disease, itis only a matter of time before their role in the agingkidney is defined.Mitochondria Mitochondria are intracellular or-ganelles that produce energy in the form of ATPthrough oxidative phosphorylation. They also seem toplay an important role in the cellular death, or apopto-sis, pathway. Although the majority of mitochondrialproteins come from nuclear DNA, mitochondria con-tain some of their own DNA (mtDNA). The mutationrate in mtDNA is 10 times higher than that of nuclearDNA because mitochondria lack protective histones orrepair mechanisms. Kidney cells participate in manyenergy-rich functions, such as ion pumps, and trans-portation of low molecular weight proteins and vita-mins. This makes the kidney particularly susceptibleto mitochondrial damage. There are no data on the rolethat mitochondria play in aging in the kidney. How-

Nephrology Self-Assessment Program - Vol 10, No 1, January 2011 9

Page 22: 2011.01 - Geriatric Nephrology

ever, it is likely that mitochondrial dysfunction con-tributes to the vulnerability of older patients to AKI.Mitochondrial mutations have been shown to bothincrease and decrease lifespan in Caenorhabditis elegans(a nematode), although at the expense of energy pro-duction (14). Severe reduction of mitochondrial func-tion in worms shortens lifespan. Mice that have beenengineered to accumulate mtDNA mutations showreduced electron transport chain function, signs ofaccelerated aging, and shortened lifespan (15). Declin-ing mitochondrial function seems likely to play a rolein aging in the kidney.Oxidative Stress There is a well-developed body ofliterature on the role of oxidants in the aging process(16–18). Advanced glycation end products (AGEs)form when sugars are attached to the amino groups ofproteins and nucleic acids. They are highly reactiveand generate reactive oxygen species. These canbe derived from cellular oxidative metabolism and areincreased in conditions such as diabetes, or they can beacquired through diet (19,20). High levels of oxidantstress are associated with all forms of age-relatedchronic disease: Cardiovascular disease (CVD), cere-brovascular disease, CKD, and diabetes (21). It is alsoassociated with increased markers of inflammation,such as fibrosis and macrophage infiltration. Elevatedlevels of both oxidant stress and inflammation areassociated with CKD (22). In addition to advancedglycation end products, oxidized lipids and lipopro-teins play a role in both aging and kidney disease.Animal models of hypercholesterolemia, high-fat di-ets, and scavenger receptor defects all show CKD aswell as atherosclerosis and CVD (23,24). In our ownstudies of glomerular aging, we have found significantchanges in the expression of ceruloplasmin by parietalepithelial cells lining Bowman’s capsule (25). Cerulo-plasmin is an antioxidant whose expression increasesfivefold in ad libitum–fed rats as they age. In calorie-restricted rats, ceruloplasmin protein expression in-creases less than twofold with age. Both the cell-associated alternately spliced variant and secretedvariants of ceruloplasmin were expressed, and wewere able to detect ceruloplasmin in urine. Ceruloplas-min expression by Bowman’s capsule epithelial cellstherefore occurred in direct proportion to known levelsof oxidant activity (older age and high-calorie diet)and is secreted in the urine. This may well represent aprotective mechanism within the kidney to reduceoxidant damage of the tubule by filtered oxidant load.

Autophagy Autophagy is the process by whichdamaged proteins and organelles are delivered to thelysosome for disassembly and recycling. This processallows postmitotic cells, such as podocytes, to dealwith damage caused by oxidation, misfolding, or anyother protein pathology. Failure to maintain cell integ-rity is associated with dysfunction and aging (26). Inmice, podocyte-specific knockouts of players in theautophagy pathway have been created (27). Thesemice show accelerated glomerular aging with accumu-lations of oxidized and ubiquitinated proteins, endo-plasmic reticulum stress, and proteinuria and ulti-mately to late-onset glomerulosclerosis. In this model,a 10-fold increase in urinary protein-to-creatinine ratiowas seen in 20-month-old mice with a fivefold in-crease in glomerulosclerosis. Damaged proteins andorganelles accumulated in the podocytes and had adetrimental effect on cellular homeostasis, and the au-thors showed a decline in podocyte numbers per glomer-ulus compared with controls by 22 months. Declines inautophagy proteins have been demonstrated in biopsiesof people with glomerular diseases (27).Telomeres Telomeres are specialized structures atthe ends of chromosomes. They are vital for chromo-some stability and maintenance of chromosome lengthduring cell division. Telomeres are nucleoproteincomplexes that contain several kilobases of TTAGGGdouble-stranded repeats and approximately 400 to 500bases of TTAGGG repeats in a 3� single-strandedoverhang. The complex that orchestrates replication ofDNA sits at the end of the chromosome sequence sothat this area does not get copied. Thus, telomerelength dictates cell lifespan because each round ofDNA replication results in successive telomere short-ening until the telomeres become critically shortened,resulting in cellular crisis. Stem cells overcome thislimitation using an enzyme called telomerase (a ribo-nucleoprotein with reverse transcriptase activity),which adds TTAGGG repeats back to the telomere andthereby prevents DNA shortening and subsequent cri-ses. The enzyme is made up of a telomerase RNAcomponent that provides the template to add theTTAGGG repeats; a second component of the telom-erase is telomerase reverse transcriptase. Telomeraseis expressed primarily in germ cells and stem cells, aswell as in most cancer cells. At least two barriers thatmay alter replicative senescence for diploid cell typesseem to exist: One is premature telomere shortening,and the other is an accumulation of environmental

10 Nephrology Self-Assessment Program - Vol 10, No 1, January 2011

Page 23: 2011.01 - Geriatric Nephrology

stress–imposed DNA damage, limiting the number ofcell divisions to 10 to 15 instead of �50 in vitro. Anexcellent review by Campisi (28) outlines the currentunderstanding of the relationship of cellular senes-cence, tumor suppression, and organism aging. How isthis complex cell biology relevant to the kidney?Westoff et al. (29) studied the role of telomeres inischemia-reperfusion injury in telomerase-deficientmice, comparing fourth-generation mice with first-and second-generation mice. As the telomeres short-ened through the generations, mice demonstrated re-duced proliferative capacity in tubular, glomerular,and interstitial cells. They were more vulnerable toAKI and less likely to regenerate tubular epithelium.They mimicked older patients who are significantlymore susceptible to acute renal injury and less likely torecover renal function after such an injury. Feest et al.(30) showed that there is a progressive age-dependentincrease in AKI after age 60. Mortality also increaseswith age, and older patients who require dialysis havea mortality rate of �80% (31). Rates of recovery ofrenal function are 28% lower in patients who are olderthan 65 years (32).Calorie Restriction Calorie restriction has beenknown to increase lifespan since 1935, when McCayet al. (33) published a rodent study that showed thatreducing calories without malnutrition extended bothaverage and maximum life expectancy. In the inter-vening years, work on calorie restriction and its mech-anism was limited mainly to short-lived organisms,such as yeast, worms, and flies. In the 1980s, the NIAset up a series of studies in rodents to examine mech-anisms of aging and the retardation of aging-associ-ated pathologies, not just extension of lifespan. TheNIA developed specific colonies of aging mammalsand systematically characterized the aging process inrodents, comparing ad libitum–fed animals with theircalorie-restricted littermates. They established guide-lines for the nutritional management of calorie restric-tion and continue to supply calorie-restricted animalsand appropriate food to funded projects (34–36). Mostrecently, Colman et al. (37) published the results of a20-year longitudinal study of calorie restriction inprimates. They observed the effect of calorie restric-tion on the resistance to age-associated illness as wellas the effect on mortality. They reported a threefoldincrease in the onset of age-associated diseases in adlibitum–fed animals as compared with the calorie-restricted animals at any given age and a significant

increase in lifespan. They specifically studied onset ofdiabetes, cancer, CVD, and brain atrophy becausethese are common diseases of aging in humans. In ourlaboratory, we have looked specifically at the effect ofcalorie restriction on rat glomerular aging and pathol-ogy. We have shown that calorie restriction reducesmesangial matrix expansion and proteinuria and pre-vents the development of age-associated glomerulo-sclerosis (38). Although this result is encouraging,calorie restriction itself is unlikely ever to be a realistictherapeutic option in humans.DNA Damage and Progeria Maintaining the in-tegrity of DNA is essential for healthy cellular well-being. Mutations in DNA are the cause of manycancers and also of many accelerated aging processes.Mutations that lead to cancer are typically due to lossof tumor suppressor genes or activation of prolifera-tive genes. Mutations that lead to accelerated aging areusually localized to the genes responsible for DNArepair, replication, and transcription. The best studiedof these premature aging syndromes, known as prog-eria, are due to mutations in helicases, which areresponsible for reading and checking and repairing theintegrity of DNA. Loss of function in helicases canresult in defective replication, inefficient transcription,deficient mismatch repair, and chromosome rearrange-ments. Although none of the progeroid syndromesexactly matches physiologic aging, studies of thesesyndromes are yielding clues to how DNA damagethat accumulates with age may play a role in the agingprocess (39). There is one report of kidney pathologyrelated to progeria. It describes renal histopathologyfrom two patients who died from progeria. Theyounger patient, who died at age 11 years, had noglomerulosclerosis, whereas the kidney from the 20-year-old patient showed focal renal scarring with focalglomerulosclerosis and associated tubular atrophy,similar to that seen in physiologic aging (40). Studiesof these genetic pathways and the cellular response toDNA instability have shown that the cell can switch onan antiaging response by suppressing metabolism andcell growth through the target of rapamycin (TOR)pathway (see the TOR Signaling section). Mouse ex-periments are ongoing and attempting to slow theaging process using rapamycin (sirolimus) to interferewith mammalian TOR signaling (41). Results fromthis study so far are modest, showing an increase inmaximum lifespan of 14% for female mice and 9% formale mice. This does show promise for pharmacologic

Nephrology Self-Assessment Program - Vol 10, No 1, January 2011 11

Page 24: 2011.01 - Geriatric Nephrology

interventions that prevent age-related diseases, but itremains to be shown that age-associated glomerulo-sclerosis can be slowed. There is also the consider-ation of significant adverse effect profile associatedwith rapamycin.

Individual Aging Genes

The expression of individual genes canhave a profound influence on the agingprocess. Some of these genes have beenshown to have a renal phenotype.

IGF-1 In 1993, Kenyon et al. (42) published thefirst article showing that mutations in individual genescould affect lifespan and the rate of aging. In theirarticle, a mutation in a C. elegans gene called daf-2resulted in a doubling of worm lifespan. These wormswere healthy, active, and fertile but lived much longerthan their wild-type controls. In 1997, daf-2 wascloned and shown to be the insulin receptor familymember IGF-1 (43). Mutations in other insulin path-way family members have since been shown to havesimilar effects on longevity. Inhibiting IGF pathwaysextends lifespan through changes in gene expression inmultiple other pathways, and these are reviewed indetail in reference 43. Similar lifespan extension wassoon shown for mutations in other genes within thesame signaling pathway. This pathway affects expres-sion of a DNA FOXO transcription factor, whichcontrols expression of activity in multiple metabolicpathways and ultimately results in lifespan extension.It was initially thought that a process that worked insmall simple organisms, such as C. elegans, wasunlikely to have a parallel in highly complex largerorganisms. However, manipulation of this pathwayhas now been successfully used to extend lifespan inmultiple organisms, including mice (44). Although itis clearly not possible experimentally to modulateactivity in this pathway in humans, there are cohorts ofcentenarians in which mutations that affect activity inIGF-1 signaling have been shown (45). It has alsobeen shown that this pathway plays a role in thelongevity response to calorie restriction.TOR Signaling The TOR pathway senses the avail-ability of food and nutrients and controls growth anddevelopment. TOR inhibition increases lifespan in allexperimental models from yeast to mammals (46). It is

the pathway that is implicated in calorie restriction,and when you restrict calories of a TOR-transgenicanimal, you do not get further extension of lifespan.When nutrients are plentiful, the TOR kinase stimu-lates protein translation for the purposes of growth andreproduction. When nutrients are scarce or when TORsignaling is experimentally inhibited, this pathwayswitches cells from active growth to somatic mainte-nance and extends lifespan (47,48). This pathwayinteracts with pathways that control mRNA transla-tion, autophagy, and mitochondrial metabolism. Theexact mechanism whereby aging is slowed is stillbeing elucidated. As noted in the Calorie Restrictionsection, experiments are under way in mice to try toextend lifespan by using rapamycin (41). Clearly,careful thought will need to be given to the use ofrapamycin for aging in humans in view of its manyadverse effects. However, the significant work on thispathway does hold the promise of finding ways toslow age-related pathologies and facilitate healthy,disease-free aging.Sirtuins The sirtuin pathways are believed to be thetarget of resveratrol, the much-touted “healthy ingre-dient” in red wine. Sirtuins are a broadly conservedfamily of enzymes found in all phyla of life from thesimplest to the most complex. These ancient proteinshave a common biochemistry that allows them tointeract with nicotinamide adenine dinucleotide anddeacetylate proteins. The first link with aging was thediscovery of silent information regulator 2 (SIR2) inyeast, which controlled healthy extension of lifespan(49,50). There are seven mammalian homologs:SIRT1 through 7 (51). These are present in severalcompartments of the cell, including the nucleus, mi-tochondria, and cytosol. Knockouts of at least onefamily member, SIRT6, causes premature aging (52).As this field continues to grow, new roles for theseubiquitous molecules are being added. They have beenshown to be important in adaptation to low-nutrientconditions, mitochondrial function, DNA repair, neu-ronal survival, and the maintenance of a youthfulpattern of gene survival. Which of these functions isresponsible for the increase in lifespan is not yet clear.There is, however, research implicating the sirtuins inmetabolic dysfunction such as diabetes, in cancersuppression, and in CVD, the world’s leading cause ofdeath (51). This is a rapidly expanding field with, asyet, no link to the kidney.Klotho Klotho was the Greek goddess who spun the

12 Nephrology Self-Assessment Program - Vol 10, No 1, January 2011

Page 25: 2011.01 - Geriatric Nephrology

thread of life, and the name was given to a gene thatcharacterized an accelerated aging phenotype. Therole of klotho in aging was an accidental findingpublished in 1997 (53). A group making a transgenicmouse had fortuitously inserted their transgene ran-domly into the promoter region of the klotho gene andproduced a prematurely aged mouse that lives only 5to 6% of normal captive mouse lifespan. Subsequentwork with an overexpressing model produced a mousethat lives 20 to 30% longer than wild-type littermates(54). Klotho is expressed as both a membrane proteinand a secreted protein, primarily in the distal tubularcells of the kidney. Its primary role seems to be as aco-factor or co-receptor regulating fibroblast growthfactor 23 signaling and activation of the ion channelTRPV5. It plays an important role in phosphorushomeostasis. Klotho promotes phosphate excretion,and reduced expression of klotho is associated withectopic calcification, increased concentrations of 1,25-dihydroxyvitamin D3, hyperphosphatemia, and therapy-resistant hyperparathyroidism (55,56). Although thesefunctions are important in the context of renal disease,the aging phenotype seems to be modulated throughthe IGF-1 signaling pathway. Secreted klotho inhibitsinsulin/IGF-1 signaling, and klotho-deficient miceare hypoglycemic and highly insulin sensitive. Theklotho-overexpressing mice are IGF-1 resistant. It isthis interaction with an evolutionary conserved mech-anism for regulating aging that seems to confer theaging phenotype. Reduced expression of klotho hasbeen observed in patients with CKD. Some single-nucleotide polymorphisms in human klotho are asso-ciated with altered lifespan and increased vasculardisease (57). Klotho is clearly an interesting proteinwith a role to play in the complications of CKD andmaybe in the altered lifespan associated with thisdisease.

Animal Models of Extended LifespanThe science of aging started with the manipula-

tion of genes in short-lived species such as yeast, C.elegans, and Drosophila. These allowed for rapidcharacterization of phenotype. In the 1980s, two long-lived dwarf mouse strains were discovered: Ames andSnell (58). Both of these were spontaneous mutations.These mice lack growth hormone, prolactin, and thy-roid-stimulating hormone as a result of disruption of acommon transcription factor necessary for all of thesepathways. These mice are not only smaller than their

littermates but also live 65% longer. These mice alsoshow a delay in aging-related diseases. There are nowseven genetic mouse models of delayed aging. Al-though there is no clear understanding of how thesemodels confer extended lifespan, research on thesemodels is extending our understanding of the agingprocess. Recent data suggest that reduced signaling ofthe IGF-1 pathway may play a role (59). Althoughthese models are clearly interesting from a scientificperspective, they do not offer realistic therapeuticavenues for managing aging and age-associated pa-thologies in humans.

The biology of human aging is an active andrapidly evolving field, and there are as many questionsas there are answers. The US Food Drug Administra-tion does not recognize aging as a disease, so until thisis changes, drugs will be approved to treat only age-associated diseases, not the aging process itself. Onceinterventions are developed and shown to be safe andeffective, we would hope that this would change.

References1. Anderson S, Halte JB, Hazzard WR, Himmelfarb J, Horne FM,

Kaysen GA, Kusek JW, Nayfield SG, Schmader K, Tian Y, Ash-worth JR, Clayton CP, Parker RP, Tarver ED, Woolard NF, High KP:Prediction, progression, and outcomes of chronic kidney disease inolder adults. J Am Soc Nephrol 20: 1199–1209, 2009

2. Centers for Disease Control: National Vital Statistics Reports. Availableat: http://www.cdc.gov/nchs/data/nvsr/nvsr54/nvsr54_14.pdf. AccessedNovember 26, 2010

3. Lindeman RD, Tobin J, Shock NW: Longitudinal studies on the rateof decline in renal function with age. J Am Geriatr Soc 33: 278–285,1985

4. Coresh J, Astor BC, Greene T, Eknoyan G, Levey AS: Prevalence ofchronic kidney disease and decreased kidney function in the adult USpopulation: Third National health and Nutrition Examination Survey.Am J Kidney Dis 41: 1–12, 2003

5. Glassock RJ, Winearls C: The global burden of kidney disease: Howvalid are the estimates? Nephron 110: c39–c47, 2008

6. Winearls CG, Glassock RJ: Dissecting and refining the staging ofchronic kidney disease. Kidney Int 75: 1009–1014, 2009

7. US Renal Data System: USRDS 2009 Annual Data Report, Bethesda,National Institutes of Health, National Institute of Diabetes andDigestive and Kidney Diseases, 2009

8. Dressler GR: The specification and maintenance of renal cell types byepigenetic factors. Organogenesis 5: 73–82, 2009

9. Wiggins JE, Patel SR, Shedden KA, Goyal M, Wharram BL, MartiniS, Kretzler M, Wiggins RC: NFkappaB promotes inflammation,coagulation, and fibrosis in the aging glomerulus. J Am Soc Nephrol21: 587–597, 2010

10. Bishop NA, Lu T, Yankner BA: Neural mechanisms of ageing andcognitive decline. Nature 464: 529–535, 2010

11. Karolina DS, Wintour EM, Bertram J, Jeyaseelan K: Riboregulatorsin kidney development and function. Biochimie 92: 217–225, 2010

12. Harvey SJ, Jarad G, Cunningham J, Goldberg S, Schermer B, HarfeBD, McManus MT, Benzing T, Miner JH: Podocyte-specific deletionof Dicer alters cytoskeletal dynamics and causes glomerular disease.J Am Soc Nephrol 19: 2069–2075, 2008

Nephrology Self-Assessment Program - Vol 10, No 1, January 2011 13

Page 26: 2011.01 - Geriatric Nephrology

13. Grillari J, Grillari-Voglauer R: Novel modulators of senescence,aging and longevity: Small non-coding RNAs enter the stage. ExpGerontol 45: 302–311, 2010

14. Geanacopoulos M: The determinants in lifespan in the nematodeCaenorhabditis elegans: A short primer. Sci Prog 87: 227–247, 2004

15. Edgar D, Trifunovic A: The mtDNA mutator mouse: Dissectingmitochondrial involvement in aging. Aging (Albany NY) 1: 1028–1032, 2009

16. Muller FL, Lustgarten MS, Jang Y, Richardson A, Van Remmen H:Trends in oxidative aging theories. Free Radic Biol Med 43: 477–503, 2007

17. Ferrucci L: The Baltimore Longitudinal Study of Aging (BLSA): A50-year-long journey and plans for the future. J Gerontol A Biol SciMed Sci 63: 1416–1419, 2008

18. Vlassara H, Uribarri J, Ferrucci L, Cai W, Torreggiani M, Post JB,Zheng F, Striker GE: Identifying advanced glycation end products asa major source of oxidants in aging: Implications for the managementand/or prevention of reduced renal function in elderly persons. SeminNephrol 29: 594–603, 2009

19. Makita Z, Yanagisawa K, Kuwajima S, Yoshioka N, Atsumi T,Hasunuma Y, Koike T: Advanced glycation endproducts and diabeticnephropathy. J Diabetes Complications 9: 265–268, 1995

20. Uribarri J, Peppa M, Cai W, Goldberg T, Lu M, He C, Vlassara H:Restriction of dietary glycotoxins reduces excessive advanced glyca-tion endproducts in renal failure patients. J Am Soc Nephrol 14:728–731, 2003

21. Navarro-Díaz M, Serra A, Lopez D, Granada M, Bayes B, Romero R:Obesity, inflammation, and kidney disease. Kidney Int Suppl (111):S15–S18, 2008

22. Vlassara H, Uribarri J, Ferrucci L, Cai W, Torreggiani M, Post JB,Zheng F, Striker GE: Identifying advanced glycation end products asa major source of oxidants in aging: Implications for the managementand/or prevention of reduced renal function in elderly persons. SeminNephrol 29: 594–603, 2009

23. Muntner P, Coresh J, Smith JC, Eckfeldt J, Klag MJ: Plasma lipidsand risk of developing renal dysfunction: The Atherosclerosis Risk inCommunities Study. Kidney Int 58: 293–301, 2000

24. Okamura DM, Pennathur S, Pasichnyk K, Lopez-Guisa JM, CollinsS, Febbraio M, Heinecke J, Eddy AA: CD36 regulates oxidativestress and inflammation in hypercholesterolemic CKD. J Am SocNephrol 20: 495–505, 2009

25. Wiggins JE, Goyal M, Wharram BL, Wiggins RC: Antioxidantceruloplasmin is expressed by glomerular parietal epithelial cells andsecreted into urine in association with glomerular aging and high-calorie diet. J Am Soc Nephrol 17: 1382–1387, 2006

26. Cuervo AM, Bergamini E, Brunk UT, Droge W, Ffrench M, TermanA: Autophagy and aging: The importance of maintaining “clean”cells. Autophagy 1: 131–140, 2005

27. Hartleben B, Godel M, Meyer-Schwesinger C, Liu S, Ulrich T,Kobler S, Wiech T, Grahammer F, Arnold SJ, Lindenmeyer MT,Cohen CD, Pavenstadt H, Kerjaschki D, Mizushima N, Shaw AS,Walz G, Huber TB: Autophagy influences glomerular disease sus-ceptibility and maintains podocyte homeostasis in aging mice. J ClinInvest 120: 1084–1096, 2010

28. Campisi J: Senescent cells, tumor suppression, and organismal aging:Good citizens, bad neighbors. Cell 120: 513–522, 2005

29. Westhoff JH, Schildhorn C, Jacobi C, Homme M, Hartner A, BraunH, Kryzer C, Wang C, von Zglinicki T, Kranzlin B, Gretz N, Melk A:Telomere shortening reduces regenerative capacity after acute kidneyinjury. J Am Soc Nephrol 21: 327–336, 2010

30. Feest TG, Round A, Hamad S: Incidence of severe acute renal failurein adults: Results of a community based study. BMJ 306: 481–483,1993

31. Wardle EN: Acute renal failure and multiorgan failure. Nephrol DialTransplant 9[Suppl 4]: 104–107, 1994

32. Schmitt R, Coca S, Kanbay M, Tinetti ME, Cantley LG, Parikh CR:

Recovery of kidney function after acute kidney injury in the elderly:A systematic review and meta-analysis. Am J Kidney Dis 52: 262–271, 2008

33. McCay CM, Crowell MF, Maynard LA: The effect of retardedgrowth upon the length of life span and upon the ultimate body size.Nutrition 5: 155, 1935

34. Yu BP, Masoro EJ, McMahan CA: Nutritional influences on aging ofFischer 344 rats: 1. Physical, metabolic and longevity characteristics.J Gerontol 40: 657–670, 1985

35. Maeda H, Gleiser CA, Masoro EJ, Murata I, McMahan CA, Yu BP:Nutritional influences on aging of Fischer 344 rats: II. Pathology. JGerontol 40: 671–688, 1985

36. Masoro EJ: Dietary restriction and aging. J Am Geriatr Soc 41:994–999, 1993

37. Colman RJ, Anderson RM, Johnson SC, Kastman EK, Kosmatka KJ,Beasley TM, Allison DB, Cruzen C, Simmons HA, Kemnitz JW,Weindruch R: Caloric restriction delays disease onset and mortalityin rhesus monkeys. Science 325: 201–204, 2009

38. Wiggins JE, Goyal M, Sanden SK, Wharram BL, Shedden KA,Misek DE, Kuick RD, Wiggins RC: Podocyte hypertrophy, “adapta-tion,” and “decompensation” associated with glomerular enlargementand glomerulosclerosis in the aging rat: Prevention by calorie restric-tion. J Am Soc Nephrol 16: 2953–2966, 2005

39. Hoeijmakers JH: DNA damage, aging, and cancer. N Engl J Med361: 1475–1485, 2009

40. Delahunt B, Stehbens WE, Gilbert-Barness E, Shozawa T, RugerBM: Progeria kidney has abnormal mesangial collagen distribution.Pediatr Nephrol 15: 279–285, 2000

41. Harrison DE, Strong R, Sharp ZD, Nelson JF, Astle CM, Flurkey K,Nadon NL, Wilkinson JE, Frenkel K, Carter CS, Pahor M, JavorsMA, Fernandez E, Miller RA: Rapamycin fed late in life extendslifespan in genetically heterogeneous mice. Nature 460: 392–395,2009

42. Kenyon C, Chang J, Gensch E, Rudner A, Tabtiang R: A C. elegansmutant that lives twice as long as wild type. Nature 366: 461–464,1993

43. Kimura KD, Tissenbaum HA, Liu Y, Ruvkun G: Daf-2, an insulinreceptor-like gene that regulates longevity and diapause in Caeno-rhabditis elegans. Science 277: 942–946, 1997

44. Yuan R, Tsaih SW, Petkova SB, Marin de Evsikova C, Xing S,Marion MA, Bogue MA, Mills KD, Peters LL, Bult CJ, Rosen CJ,Sundberg JP, Harrison DE, Churchill GA, Paigen B: Aging in inbredstrains of mice: Study design and interim report on median lifespansand circulating IGF1 levels. Aging Cell 8: 277–287, 2009

45. Pawlikowska L, Hu D, Huntsman S, Sung A, Chu C, Chen J, JoynerAH, Schork NJ, Hsueh WC, Reiner AP, Psaty BM, Atzmon G,Barzilai N, Cummings SR, Browner WS, Kwok PY, Ziv E: Associ-ation of common genetic variation in the insulin/IGF1 signalingpathway with human longevity. Aging Cell 8: 460–472, 2009

46. Kenyon CJ: The genetics of ageing. Nature 464: 504–512, 201047. Kapahi P, Zid B: TOR pathway: Linking nutrient sensing to life span.

Sci Aging Knowledge Environ 2004: PE34, 200448. Kapahi P, Chen D, Rogers AN, Katewa SD, Li PW, Thomas EL,

Kockel L: With TOR, Less Is More: A Key Role for the ConservedNutrient-Sensing TOR Pathway in Aging. Cell Metab 11: 453–465,2010

49. Sinclair DA, Guarente L: Extrachromosomal rDNA circles: A causeof aging in yeast. Cell 91: 1033–1042, 1997

50. Kaeberlein M, McVey M, Guarente L: The SIR2/3/4 complex andSIR2 alone promote longevity in Saccharomyces cerevisiae by twodifferent mechanisms. Genes Dev 13: 2570–2580, 1999

51. Haigis MC, Sinclair DA: Mammalian sirtuins: Biological insightsand disease relevance. Annu Rev Pathol 5: 253–295, 2010

52. Mostoslavsky R, Chua KF, Lombard DB, Pang WW, Fischer MR,Gellon L, Liu P, Mostoslavsky G, Franco S, Murphy MM, Mills KD,Patel P, Hsu JT, Hong AL, Ford E, Cheng HL, Kennedy C, Nunez

14 Nephrology Self-Assessment Program - Vol 10, No 1, January 2011

Page 27: 2011.01 - Geriatric Nephrology

N, Bronson R, Frendewey D, Auerbach W, Valenzuela D, Karow M,Hottiger MO, Hursting S, Barrett JC, Guarente L, Mulligan R,Demple B, Yancopoulos GD, Alt FW: Genomic instability andaging-like phenotype in the absence of mammalian SIRT6. Cell 124:315–329, 2006

53. Kuro-o M, Matsumura Y, Aizawa H, Kawaguchi H, Suga T, UtsugiT, Ohyama Y, Kurabayashi M, Kaname T, Kume E, Iwasaki H, IidaA, Shiraki-Iida T, Nishikawa S, Nagai R, Nabeshima YI: Mutation ofthe mouse klotho gene leads to a syndrome resembling ageing.Nature 390: 45–51, 1997

54. Kurosu H, Yamamoto M, Clark JD, Pastor JV, Nandi A, Gurnani P,McGuinness OP, Chikuda H, Yamaguchi M, Kawaguchi H, Shimo-mura I, Takayama Y, Herz J, Kahn CR, Rosenblatt KP, Kuro-o M:Suppression of aging in mice by the hormone klotho. Science 309:1829–1833, 2005

55. Wang Y, Sun Z: Current understanding of klotho. Ageing Res Rev 8:43–51, 2009

56. Kuro-o M: Klotho and aging. Biochim Biophys Acta 1790: 1049–1058, 2009

57. Arking DE, Krebsova A, Macek M Sr, Macek M Jr, Arking A, MianIS, Fried L, Hamosh A, Dey S, McIntosh I, Dietz HC: Association ofhuman aging with a functional variant of klotho. Proc Natl Acad SciU S A 99: 856–861, 2002

58. Liang H, Masoro EJ, Nelson JF, Strong R, McMahan CA, RichardsonA: Genetic mouse models of extended lifespan. Exp Gerontol 38:1353–1364, 2003

59. Bartke A, Brown-Borg H: Life extension in the dwarf mouse. CurrTop Dev Biol 63: 189–225, 2004

Age, Estimated GFR Formulas, and Assessmentof Risk for Adverse Cardiovascular and RenalOutcomes in Chronic Kidney DiseaseCross-Sectional Relationships among Age,Estimated GFR, and Proteinuria

Whether defined by estimated GFR (eGFR) aloneor by eGFR and level of urinary protein excretion, theprevalence of chronic kidney disease (CKD) increasesdramatically with age. In a recent analysis of a represen-tative sample of the US population, �45% of adults aged�70 years had an eGFR of �60 ml/min per 1.73 m2 oran albumin-to-creatinine ratio (ACR) of �30 mg/g (1).Although both abnormalities become increasingly com-mon with age, the prevalence of an eGFR of �60 ml/minper 1.73 m2 increases far more dramatically with agethan that of albuminuria. Furthermore, most of the age-associated increase in the prevalence of an eGFR of �60ml/min per 1.73 m2 is accounted for by an increase in theprevalence of moderate (eGFR 45 to 59 ml/min per 1.73m2) rather than severe (eGFR �30 ml/min per 1.73 m2)reductions in eGFR (2–6). Consequently, when olderand younger populations with CKD (defined as thepresence of either a low eGFR or albuminuria) arecompared, older patients who meet these criteria aremore likely to have an isolated low eGFR, whereasyounger patients are more likely to have isolated albu-minuria (5,8,7).

Estimates of the prevalence of CKD in olderadults are remarkably consistent across a wide rangeof populations, largely reflecting age-related changesin the distribution of eGFR on a population level(3,5,6). Wetzels et al. (9) provided detailed informa-tion on the distribution of eGFR by age group in apopulation-based cross-sectional study in the Nether-lands. Among Caucasian men without comorbidity,mean eGFR decreased with age, from 100 ml/min per1.73 m2 for those aged 18 to 24 to 62 ml/min per 1.73m2 for those aged �85. For Caucasian women, meaneGFR ranged from 91 to 59 ml/min per 1.73 m2 acrossthese same age groups. Values were slightly lower formen and women with comorbidity but not impres-sively so. For example, mean values of eGFR for menand women who were aged �85 and had self-reportedcomorbidity were 56 and 55 ml/min per 1.73 m2,respectively. Note that the mean value for eGFR atolder ages falls very close to the threshold level ofeGFR �60 ml/min per 1.73 m2 currently used todefine CKD.

Longitudinal Changes in Renal Function withAging

The higher prevalence of CKD at older agesobserved in cross-sectional studies likely suggests thatmost adults lose kidney function as they age. How-ever, because the prevalence of many of the comor-bidities associated with kidney disease, such as diabe-tes and hypertension, also increase with age, it is oftenhard to distinguish the effects of these comorbiditiesfrom those of increasing age. Limited data are avail-able to address the question of whether and how muchrenal function changes during healthy aging. In theBaltimore Longitudinal Study of Aging, 234 healthyadults were followed for �8 years with serial creati-nine clearance measurements (10). On average, theseindividuals lost 0.75 ml/min per y creatinine clearance.However, creatinine clearance slopes were normallydistributed around this mean value, with approxi-mately one third of individuals experiencing no de-cline with age. These limited data suggest that al-though on average there is an age-associated decline inrenal function even among healthy adults, this doesnot seem to be an inevitable consequence of aging.

It is often assumed that loss of eGFR with ageoccurs as the result of age-related pathologic changesto the kidney, including renal fibrosis and glomerulo-sclerosis. Although these pathologic changes are

Nephrology Self-Assessment Program - Vol 10, No 1, January 2011 15

Page 28: 2011.01 - Geriatric Nephrology

known to be strongly associated with aging, the extentto which they explain differences in level of renalfunction among older patients is not so clear. In arecent retrospective series of relatively healthy livingkidney donors, the presence of age-related structuralchanges in the kidney (termed nephrosclerosis bythese authors) on renal biopsy increased dramaticallywith age, ranging from 2.7% (95% confidence interval[CI] 1.1 to 6.7%) for patients aged 18 to 29 years to73% (95% CI 43 to 90%) for patients aged 70 to 77years (11). From the youngest to the oldest age group,GFR measured using iothalamate clearance rangedfrom 114 � 19 to 86 � 11 ml/min per 1.73 m2.However, after accounting for age, there was no rela-tionship between nephrosclerosis and measured GFR.Because most of the donors were healthy and did nothave significant loss of eGFR, the study was not ableto provide information on older and younger patientswith abnormal renal function. Nevertheless, the studyseems to suggest that an increase in the prevalence ofCKD with age cannot be fully explained by a parallelincrease in the prevalence of nephrosclerosis becausethese changes seem to be common even amonghealthy older patients with relatively preserved kidneyfunction.

The prevalence of CKD defined using cur-rent eGFR-based criteria increases dra-matically with age. However, the clinicalsignificance of moderate reductions ineGFR in the range of 45 to 59 ml/min per1.73 m2 is uncertain.

Validity of Methods for Estimating GFR in theElderly

The Cockroft-Gault equation was developed in acohort of 249 Caucasian men aged 18 to 92 to predict24-hour creatinine clearance (12). The four-variableModification of Diet in Renal Disease (MDRD) equa-tion is a simplified version of the six-variable MDRDStudy equation that was developed and validatedamong participants in the MDRD Study to predictiothalamate clearance (13). The MDRD study was arandomized, controlled clinical trial of both a dietaryand a BP intervention (14). Entry criteria were age 18to 70 years, a serum creatinine concentration of 1.2 to7.0 mg mg/dl in women and 1.4 to 7.0 mg/dl in men or

a creatinine clearance of �70 ml/min per 1.73 m2, anda mean arterial pressure of �125 mmHg. Relativelyfew studies have attempted to validate either of theseequations in a representative sample of older adults.Fehrman-Eckholm and Skeppholm recruited 52 healthyadults who were older than 70 to undergo iohexalclearance and compared this measure with estimates ofGFR and creatinine clearance obtained using the six-variable MDRD equation and the Cockroft-Gaultequation, respectively (15). The mean age of the co-hort was 82 and ranged from 71 to 110 years (andincluded the oldest woman alive in Sweden at thattime). Mean GFR was 67.7 � 10.8 ml/min based oniohexal clearance compared with 60.1 � 11.8 ml/minbased on the MDRD equation and 46.2 � 11.3 ml/minbased on the Cockroft-Gault equation. Thus, in thiscohort, there was fairly good agreement between meanvalues of the MDRD equation and that of a goldstandard measure of GFR. Consistent with this find-ing, several other studies have reported that theMDRD and Cockroft-Gault equations yield quite dis-crepant estimates of GFR in some populations of olderadults and that these estimates often differ from mea-sured 24-hour creatinine clearance (16,17). The MDRDequation in particular may not be reliable in patientswith an eGFR close to or within the normal range (18),which may carry particular relevance for older adultsgiven that, with increasing age, the mean level of renalfunction among community-dwelling older adultsmoves progressively closer to the upper limit of accu-racy of this equation (9). Recently, a new equationfrom the Chronic Kidney Disease Epidemiology(CKD-EPI) collaboration was developed using pooledindividual-level data from studies in which gold stan-dard measures of GFR were available (19). Like theabbreviated MDRD equation, the CKD-EPI equationincludes terms for age, race, gender, and serum creat-inine but is more complex. The mean age of partici-pants in studies used to develop this equation rangedfrom 24 to 59 years. Patients aged �70 composed only3% of the development and internal validation samplesand 7% of the external validation sample for thisstudy. Compared with the MDRD study equation, theCKD-EPI equation was more accurate for those withan eGFR of �60 ml/min per 1.73 m2 and had similaraccuracy for those with an eGFR of �60 ml/min per1.73 m2. The accuracy of this new equation amongpatient subgroups in an external validation data setwas recently reported (20). These results suggest that

16 Nephrology Self-Assessment Program - Vol 10, No 1, January 2011

Page 29: 2011.01 - Geriatric Nephrology

in most subgroups, including adults who are older than65, the CKD-EPI equation provides less biased esti-mates of true GFR than the MDRD equation at eGFRlevels of �60 ml/min per 1.73 m2. In adults aged �65,the CKD-EPI equation also provided less biased esti-mates at eGFR levels �60 ml/min per 1.73 m2. How-ever, this was not the case for adults who were olderthan 65 years, in whom bias was similarly low withboth equations. It should also be noted that of the 568members of the external validation data set older than65, only 92 had an eGFR of �60 ml/min per 1.73 m2.Consistent with the finding that the equation per-formed no better than the MDRD equation amongolder adults with an eGFR of �60 ml/min per 1.73 m2,the MDRD and CKD-EPI equations yielded similarprevalence estimates of an eGFR 30 to 59 ml/min per1.73 m2 among a representative sample of older adultsin the US population (19).

Cystatin C is an alternative marker to serumcreatinine for estimating renal function that is prob-ably less affected by changes in muscle mass (asoften occur with age). Compared with serum creat-inine and creatinine-based measures, this measuremay provide more accurate estimates of GFR withinthe “normal” range. Supporting this possibility, cys-tatin C seems to provide more accurate mortalityrisk assessment than serum creatinine at the lowerend of the range of values for both of these mea-sures (21). Cystatin C also has prognostic value inpatients with eGFR levels of �60 ml/min per 1.73m2. Older adults with a “normal” eGFR but elevatedlevels of cystatin C are at increased risk for mortal-ity and a variety of other adverse events comparedwith those with lower levels of cystatin C (22). Theprognostic value of cystatin C among patients witha low eGFR is less impressive. Among members ofthe MDRD study cohort, cystatin C, measured GFR,and creatinine-based measures of renal function hada similar relationship with mortality and kidneyfailure (defined as the need for dialysis or transplan-tation) (23). It is an open question as to whethercystatin C should be used instead of serum creati-nine to estimate renal function in older adults in theclinical setting. The usefulness of this measurewould largely depend on the added clinical value ofbeing able to estimate true GFR or to assess risk foradverse outcomes among older adults with rela-tively preserved levels of GFR.

Relationship between eGFR and Proteinuriaand Risk for Death in Older Adults

Although serum creatinine is an imperfect mea-sure of true GFR, particularly in the elderly, it never-theless has substantial prognostic significance in olderadults, as evidenced by its inclusion in several prog-nostic indices developed in older populations (24,25).Although the use of the MDRD equation allows serumcreatinine measurements to be standardized for age,race, and gender, it is important to recognize that theprognostic implications of a given level of eGFR varyby age. Thus, although a given level of eGFR canconvey valuable information on future risk for adverseoutcomes such as death, hospitalization, and progres-sion to ESRD, the relative and absolute risks for eachoutcome often vary substantially by age.Association of eGFR with Mortality The associ-ation of a low eGFR with an increased risk for deathis now well established. Although this association hasbeen described in a variety of cohorts, perhaps thelargest and most definitive study was conductedamong members of Kaiser Permanente Northern Cal-ifornia (26). This study demonstrated that among1,120,295 adults, risk for death, cardiovascular events,and hospitalization all increased at eGFR levels of�60 ml/min per 1.73 m2. Unlike many previous stud-ies, this study was large enough to provide detailedresults on risk for death, cardiovascular events, andhospitalization after separating patients with stage 3CKD into those with an eGFR of 45 to 59 and 30 to 44ml/min per 1.73 m2 while also adjusting for a widerange of other comorbidities. In so doing, this studyafforded the important insight that risk for each of theoutcomes examined increased exponentially as eGFRdeclined, with only minimal increase in risk at eGFRlevels of 45 to 59 ml/min per 1.73 m2 (hazard ratio 1.2;95% CI 1.1 to 1.2) and more substantial increases inrisk at eGFR levels �45 ml/min per 1.73 m2 (e.g.,hazard ratio 1.8; 95% CI 1.7 to 1.9). For this reason,some clinical practice guidelines (e.g., the UnitedKingdom NICE guidelines) now distinguish betweenpatients with stage 3A (eGFR 45 to 59 ml/min per 1.73m2) and stage 3B (eGFR 30 to 44 ml/min per 1.73 m2)CKD (27).

Although this study provided valuable risk infor-mation for a large, diverse population that includedolder adults, results were not stratified by age group.Among the relatively few studies that have comparedoutcomes associated with a low eGFR across age

Nephrology Self-Assessment Program - Vol 10, No 1, January 2011 17

Page 30: 2011.01 - Geriatric Nephrology

groups, most studies have indicated that the associa-tion of eGFR with mortality is attenuated in olderadults. Drey et al. (28) used a clinical laboratorydatabase in the United Kingdom to identify patientswith a persistently elevated serum creatinine level of�1.7 mg/dl for at least 6 months. During a mean of 5.5years of follow-up, there was an age-associated atten-uation in the standard mortality ratio for death associ-ated with abnormal kidney function. Compared withthe reference group with “normal” creatinine levels,mortality rates were increased by 36-fold in those aged16 to 49 years, 12-fold in those aged 50 to 64 years,and more than twofold in those older than 65 years.However, as the authors noted, the clinical signifi-cance of this observation is uncertain given that theabsolute risk for death was higher in older comparedwith younger patients with an elevated creatinine.

In a subsequent study, O’Hare et al. (6) exam-ined the impact of age on the association of eGFR withmortality in a national cohort of more than 2 millionveterans who underwent at least one serum creatininemeasurement within the VA Healthcare system be-tween October 1, 2001, and September 30, 2002.These authors examined both the relative and absoluterisk for death over 3 years among cohort members asa function of both age and eGFR after adjusting forother diagnosed comorbid conditions. The large sizeof this cohort allowed for fine stratification by bothage and eGFR. At all levels of eGFR, the absolute riskfor death was higher in older than in younger patients.However, as described previously by Drey et al. (28)for serum creatinine, the relative risk for death asso-ciated with a given level of eGFR was attenuated inolder patients. Although at lower levels of eGFR theabsolute risk for death was higher for older than foryounger patients, in patients with minimal reductionsin eGFR in the range of 50 to 59 ml/min per 1.73 m2,neither the absolute nor relative risk for death was anyhigher than for their peers with an eGFR in the“normal” range (eGFR �60 ml/min per 1.73 m2).Conversely, for patients who were younger than 65, aneGFR of 50 to 59 ml/min per 1.73 m2 was associatedwith a substantial increase in both relative and abso-lute mortality risk compared with their peers with aneGFR in the normal range. This study raises thequestion of whether very moderate reductions ineGFR—which in this cohort were present in more thanhalf of all patients who were aged �65 and had aneGFR of �60 ml/min per 1.73 m2—have clinical

significance in older adults. The same phenomenonwas also described by Raymond et al. (2) among106,366 adults (representing 49% of the population ofCoventry, England) who were followed for 3 years.These authors found that patients who were aged �75and had an eGFR of 45 to 59 ml/min per 1.73 m2 wereat no greater risk for death than their peers with higherlevels of eGFR. These results were not adjusted forcomorbidity but were present across strata defined bydiabetes, gender, and ethnicity. Of note, those with aneGFR of 45 to 59 ml/min per 1.73 m2 accounted forapproximately 40% of study participants who wereaged �75. A recent study among a cohort of patientswho were aged �75 years and enrolled in a clinicaltrial in the United Kingdom confirmed the uncertainclinical significance of such minimal reductions ineGFR in an older population but also provided theadditional insight that the threshold level of eGFRbelow which mortality risk increases may vary bygender (3). Women in this study with an eGFR of 45to 59 ml/min per 1.73 m2 were at no higher risk fordeath compared with their peers, whereas men with aneGFR in this range did have a higher risk for deathcompared with their peers with an eGFR of �60ml/min per 1.73 m2. Of note, women with an eGFR of45 to 59 ml/min per 1.73 m2 accounted for 26% ofcohort members and half of cohort members with aneGFR of �60 ml/min per 1.73 m2 (29).

The association of eGFR with mortality isattenuated with increasing age, which mayhave implications for the clinical signifi-cance of moderate reductions in eGFR.

Association of Proteinuria with Mortality It hasrecently become clear that level of proteinuria contrib-utes valuable additional prognostic information be-yond that provided by eGFR. It is increasingly recog-nized that level of urinary protein may be a valuablerisk stratification tool for patients with a low eGFR,including both those with and without diabetes (30–35). This finding may be of special relevance in theelderly, given the high prevalence and uncertain clin-ical significance of moderate reductions in eGFR atolder ages (5). The prognostic importance of albumin-uria has been recognized for many years, particularlyamong those with diabetes. More recently, a growing

18 Nephrology Self-Assessment Program - Vol 10, No 1, January 2011

Page 31: 2011.01 - Geriatric Nephrology

number of studies have examined the prognostic im-portance of proteinuria in the context of eGFR.Among 4098 participants in the Study of Cholesteroland Recurrent Events (CARE), a randomized trial ofpravastatin 40 mg/d versus placebo, Tonelli et al. (32)described an additive relationship between a loweGFR and dipstick proteinuria and both all-cause mor-tality and cardiovascular events. At all levels of renalfunction examined, the presence of proteinuria wasassociated with an increased risk for these events. Thistrial did not enroll patients who were older than 75,and the median age of participants ranged from a lowof 58 years among those without either proteinuria ora low eGFR to 65 years for those with a low eGFR.Among members of the Framingham cohort, Foster etal. (30) described an independent and additive associ-ation of an eGFR �60 ml/min per 1.73 m2 andmicroalbuminuria with mortality. The mean age ofparticipants in this study was 59 years, ranging from57 years for those without either a low eGFR ormicroalbuminuria to 70 years for those with both ofthese conditions. A study by Astor et al. (35) providedsimilar insights but stratified by level of eGFR.Among a representative sample of the US population,mortality increased at lower levels of eGFR and higherlevels of albuminuria, respectively. The weightedmean age of participants in that study was 44 years,ranging from 38.5 years for those with an eGFR of�60 ml/min per 1.73 m2 and no albuminuria to 70.4years for those with an eGFR �60 ml/min per 1.73 m2

and albuminuria. However, none of the aforemen-tioned studies presented age-stratified results. Hallanet al. (36) compared the association of albuminuriaand eGFR with mortality among older and youngerparticipants in a large Norwegian health survey. Inadults who were older and younger than 70 years,there was a strong additive association among albu-minuria, eGFR, and mortality. If anything, point esti-mates were greater for older participants, although CIsoverlapped. More recently, Hemmelgarn et al. (31)reported the association of level of eGFR and protein-uria with mortality, hospitalization for myocardial in-farction, and requirement for long-term dialysis ortransplantation or doubling of serum creatinine amongalmost 1 million patients identified in a Canadianlaboratory database. They found that at every level ofeGFR, the presence and severity of proteinuria byeither dipstick or ACR was associated with a highermortality risk regardless of eGFR. The mean age of

cohort patients ranged from 46.4 years for those withan eGFR of �60 ml/min per 1.73 m2 to 74.7 years forthose with an eGFR of 15 to 29 ml/min per 1.73 m2

and from 48.4 years for those with no proteinuria ondipstick to 55.4 years for those with heavy proteinuria.Age-stratified results were not presented, but the au-thors reported that results were similar for those whowere older and younger than 65.

The prognostic implications of proteinuria spe-cifically in older cohorts have provided conflictingresults. Roderick et al. (3) described inconsistent as-sociations between dipstick proteinuria and mortalityafter stratification by eGFR among adults aged �75years. Conway et al. (37) found no association be-tween dipstick proteinuria and mortality among anelderly referred population with advanced kidney dis-ease after adjustment for eGFR and other potentialconfounders. de Boer et al. (33) reported an indepen-dent and additive association between eGFR �60ml/min per 1.73 m2 (using cystatin C) and an ACR of�30 mg/g among 691 adults who had diabetes andwere aged �65 years enrolled in the CardiovascularHealth study. Rifkin et al. (34) described a similarassociation among members of the same cohort with-out diabetes. O’Hare et al. (5) measured the associa-tion of ACR with mortality in a national cohort of VApatients with diabetes after stratification by age (�65,65 to 74, and �75 years) and eGFR. Those authorsfound a robust association of level of ACR withmortality across a wide range of levels of eGFR inboth older and younger patients. As for the study byHemmelgarn et al., progressively higher levels ofalbuminuria were associated with a greater mortalityrisk. If anything, this association was more consis-tently present across all levels of eGFR in the oldestcompared with youngest age group. The mean age ofthis cohort was 66 years, and 26% of cohort memberswere aged �75 years. From the youngest to the oldestage group, the percentage of patients with an eGFR of�60 ml/min per 1.73 m2 ranged from 11 to 41%, thepercentage with microalbuminuria ranged from 19to 28%, and the percentage with macroalbuminuriaranged from 3.2 to 3.7%. In all age groups, mostpatients with an eGFR of �60 ml/min per 1.73 m2

had only a minimal reduction in eGFR (45 to 59ml/min per 1.73 m2). Collectively, these studiessuggest that the presence and level of urinary pro-tein may be particularly helpful for risk stratifica-tion among the large group of elderly patients with

Nephrology Self-Assessment Program - Vol 10, No 1, January 2011 19

Page 32: 2011.01 - Geriatric Nephrology

minimal (45 to 59 ml/min per 1.73 m2) reductions ineGFR.Combined Association of eGFR and Proteinuriawith Mortality Recently, the Chronic Kidney Dis-ease Prognosis Consortium published the results of apooled analysis of 21 general population studies de-scribing associations among eGFR, proteinuria, andall-cause and cardiovascular mortality in approxi-mately 1.2 million adults (38). The mean age ofparticipants in each cohort included in this analysisranged from 42 to 81 years. Using a reference groupwith an eGFR of 90 to 104 ml/min per 1.73 m2, theauthors noted an additive association of eGFR andproteinuria (measured by both dipstick and ACR) withall-cause and cardiovascular mortality. This relation-ship was present in both older and younger partici-pants (stratified at age 65), reaffirming the validity ofprevious observations that both measures contributeindependent prognostic information for patients of allages. Those authors also noted a nonlinear associationof eGFR with mortality. Mortality risk was lowest forpatients with an eGFR of 60 to 104 ml/min per 1.73m2, and a higher risk was noted both below (�60ml/min per 1.73 m2) and above (�105 ml/min per 1.73m2) this level. The authors observed that in analysesstratified by age, hazard ratios for a given level ofeGFR were attenuated among participants who wereolder compared with younger than 65 but that in bothage groups there was a similar nonlinear associationbetween eGFR and mortality with an increased risk atboth higher and lower levels of eGFR. This meta-analysis also presented data that may challenge thenotion that moderate reductions in eGFR (e.g., 45 to59 ml/min per 1.73 m2) may not be clinically signif-icant in older adults. The interaction between eGFRand age was NS in most studies, and the relative riskof some outcomes associated with moderate reduc-tions in eGFR did not differ greatly by age. In evalu-ating these results, it should be noted that age interac-tion tests for the association of eGFR with mortalityafter adjustment for proteinuria were statistically sig-nificant for five studies that collectively accounted foralmost 90% of all patients included in the meta-analysis, that there were likely large variations be-tween studies in the number of older participants andthe ages of these older participants, and that the studyused very broad age strata (e.g., older and youngerthan 65). To be able to describe the nonlinear associ-ation of eGFR with mortality, this study used a refer-

ence group with an eGFR of 90 to 104 ml/min per 1.73m2. However, it is important to keep in mind that onlya very small percentage of older adults will have aneGFR in this range, perhaps limiting the clinical utilityof any comparisons to this referent group (9).

Proteinuria is associated with an in-creased risk for mortality in older as inyounger adults, and this association is in-dependent of eGFR.

Relationship between eGFR and Proteinuriaand Risk for Progression of Kidney Disease inOlder Adults

Although renal-specific outcomes such as loss ofeGFR and progression to ESRD are of singular im-portance in treating patients with CKD, for many ofthese patients, these are not the most common out-comes. In a careful study of outcomes over 5 yearsamong members of Kaiser Permanente, Portland,Keith et al. (39) demonstrated that at all stages ofCKD, death is a more common outcome than ESRD.However, perhaps even more so than for mortality, therelationship between age and progression of kidneydisease is complex, particularly in relation to thecompeting risk for death.

Recently, the Chronic Kidney Disease PrognosisConsortium published the results of an analysis of therelationship between eGFR proteinuria and mortalityamong 105,872 participants (730,577 person-years) from14 studies with urine albumin-to-creatinine ratio (ACR)measurements and 1,128,310 participants (4,732,110person-years) from seven studies with urine proteindipstick measurements (38). Indeed, patients who areolder than 75 currently represent one of the fastestgrowing groups within the ESRD population. How-ever, several studies have now demonstrated thatamong patients with CKD, after accounting for levelof renal function, older age is associated with a lower,not higher, risk for progression to ESRD (4,37,40–42). Thus, although older adults are more likely tohave a lower level of eGFR and thus more likely toprogress to ESRD, when patients with similar levels ofeGFR are compared, older patients are generally lesslikely to progress to ESRD. However, it should benoted that the relationship between age and risk forESRD may vary somewhat as a function of eGFR and

20 Nephrology Self-Assessment Program - Vol 10, No 1, January 2011

Page 33: 2011.01 - Geriatric Nephrology

may be nonlinear among patients with higher levels ofeGFR. In a community screening cohort with a meanage of 41 years and a mean serum creatinine level of1 mg/dl, Hsu et al. (43) demonstrated that risk forprogression to ESRD was higher in middle-aged thanin younger adults. However, even in this cohort, ratesof progression among those older than 65 were lowerthan for both younger and middle-aged adults. Simi-larly, Ishani et al. (44) reported a higher risk for ESRDamong younger compared with older screenees in theMultiple Risk Factor Intervention Trial (MRFIT), witheach 10-year increase in age conferring a roughlytwofold increased risk for ESRD. However, membersof that cohort were relatively young and had relativelypreserved eGFR: Mean age was 46 years (range 35 to57 years) and mean eGFR was 79 ml/min per 1.73 m2.Thus, the increased risk for ESRD with increasing agein this cohort largely reflects the risk gradient betweenyounger and middle-aged adults. Findings from bothaforementioned studies are broadly consistent withage- and eGFR-stratified results among the nationalcohort of veterans described previously (4). Amongmembers of this cohort with higher levels of eGFR,there was a nonlinear association between age and riskfor ESRD, with the greatest risk seen among middle-aged patients.

Although easily measured, progression to ESRDis a somewhat complex outcome because it representsboth a treatment decision and a measure of diseaseprogression. It is thus possible that age differences inthe incidence of ESRD reflect age differences in thedecision to initiate long-term dialysis. However, it isalso likely that advanced kidney disease progressesmore slowly in older compared with younger patients.In a supplementary analysis among members of thelarge VA cohort described previously, older age wasin general associated with slower loss of eGFR, al-though this seemed to be true only at eGFR levels �45ml/min per 1.73 m2 (4). Conway et al. (37) describeda similar phenomenon among a referred populationwith advanced CKD. Conversely, results for patientswith less advanced kidney disease seem to indicatethat older age is associated with higher rates of eGFRloss (40). However, assessing change in eGFR on thebasis of clinical data is often challenging because thefrequency and spacing of serum creatinine measure-ments vary between patients and it may be difficult toaccount for differential rates of death among older andyounger patients. Furthermore, the accuracy of eGFR

as a measure of true GFR for older patients withadvanced kidney disease is uncertain, particularlygiven the possibility of concomitant loss of musclemass in these patients.

Competing Risks for Death and Progression toESRD

It is also important to recognize that the patternsof disease progression in patients with advanced kid-ney disease may differ from those in earlier diseasestages as a result of the large competing risk for death.This is particularly true for older patients. Althoughthe absolute risk for ESRD among patients with asimilar level of eGFR decreases with age, absolute riskfor death increases, with extremely high mortalityrates noted for elderly patients with advanced CKD(4,41). Consequently, for older patients, risk for deathtends to exceed that of ESRD at most levels of eGFR,whereas in younger patients, risk for ESRD mayexceed that of death at relatively higher levels ofeGFR (4). Understanding each patient’s relative riskfor each of these competing outcomes is often impor-tant when it comes to planning for future care needs.Furthermore, because death is generally much morecommon than ESRD and risk factors for ESRD anddeath may be different, these two outcomes maywarrant separate risk scores (42).

Older adults have a higher overall risk forprogressing to ESRD, reflecting a greaterprevalence of advanced CKD in the elderly.However, among patients with similar lev-els of eGFR, older adults are actually lesslikely to go on to be treated for ESRD.

Risk Factors for Progression to ESRD in theElderly

Few studies of risk factors for progression toESRD have specifically focused on older adults. Lim-ited data suggest that the importance of individual riskfactors may vary as a function of age. For example, acohort study among incident dialysis patients sug-gested that African American race was a stronger riskfactor for ESRD among middle-aged adults thanamong older age groups (45). The same may be truefor diabetes and hypertension. A study that examinedthe association of diabetes with future development ofESRD among men who were screened for MRFIT

Nephrology Self-Assessment Program - Vol 10, No 1, January 2011 21

Page 34: 2011.01 - Geriatric Nephrology

found that this association was strongest among thoseaged 35 to 39 years and grew progressively weaker inolder age groups (46). The association of hypertensionwith ESRD in the same study was noted to be weakestamong older men (47). In a referred population withkidney disease that included 71.7% of patients aged�65 years, in addition to younger age, risk factors forrenal replacement therapy included a lower eGFR,greater eGFR slope, proteinuria, and anemia (37).Population-based studies suggested that as for mortal-ity, both eGFR and proteinuria contribute importantinformation on ESRD risk among older as in youngerpatients (31,48). However, detailed population-basedresults of the combined effect of eGFR and level ofurinary protein on risk for progression to ESRD forolder adults are not available; data also are not avail-able on the effect of age on progression of kidneydisease according to underlying cause.

References1. Coresh J, Selvin E, Stevens LA, Manzi J, Kusek JW, Eggers P, Van

Lente F, Levey AS: Prevalence of chronic kidney disease in theUnited States. JAMA 298: 2038–2047, 2007

2. Raymond NT, Zehnder D, Smith SC, Stinson JA, Lehnert H, HigginsRM: Elevated relative mortality risk with mild-to-moderate chronickidney disease decreases with age. Nephrol Dial Transplant 22:3214–3220, 2007

3. Roderick PJ, Atkins RJ, Smeeth L, Mylne A, Nitsch DD, HubbardRB, Bulpitt CJ, Fletcher AE: CKD and mortality risk in older people:A community-based population study in the United Kingdom. Am JKidney Dis 53: 950–960, 2009

4. O’Hare AM, Choi AI, Bertenthal D, Bacchetti P, Garg AX, KaufmanJS, Walter LC, Mehta KM, Steinman MA, Allon M, McClellan WM,Landefeld CS: Age affects outcomes in chronic kidney disease. J AmSoc Nephrol 18: 2758–2765, 2007

5. O’Hare AM, Hailpern SM, Pavkov ME, Rios-Burrows N, Gupta I,Maynard C, Todd-Stenberg J, Rodriguez RA, Hemmelgarn BR,Saran R, Williams DE: Prognostic implications of the urinary albu-min to creatinine ratio in veterans of different ages with diabetes[erratum appears in Arch Intern Med 170: 1639, 2010]. Arch InternMed 170: 930–936, 2010

6. O’Hare AM, Bertenthal D, Covinsky KE, Landefeld CS, Sen S,Mehta K, Steinman MA, Borzecki A, Walter LC: Mortality riskstratification in chronic kidney disease: One size for all ages? J AmSoc Nephrol 17: 846–853, 2006

7. McCullough PA, Li S, Jurkovitz CT, Stevens LA, Wang C, CollinsAJ, Chen SC, Norris KC, McFarlane SI, Johnson B, Shlipak MG,Obialo CI, Brown WW, Vassalotti JA, Whaley-Connell AT, KidneyEarly Evaluation Program Investigators: CKD and cardiovasculardisease in screened high-risk volunteer and general populations: TheKidney Early Evaluation Program (KEEP) and National Health andNutrition Examination Survey (NHANES) 1999–2004. Am J KidneyDis 51[Suppl 2]: S38–S45, 2008

8. O’Hare AM, Kaufman JS, Covinsky KE, Landefeld CS, McFarlandLV, Larson EB: Current guidelines for using angiotensin-convertingenzyme inhibitors and angiotensin II-receptor antagonists in chronickidney disease: Is the evidence base relevant to older adults? AnnIntern Med 150: 717–724, 2009

9. Wetzels JF, Kiemeney LA, Swinkels DW, Willems HL, den Heijer

M: Age- and gender-specific reference values of estimated GFR inCaucasians: The Nijmegen Biomedical Study. Kidney Int 72: 632–637, 2007

10. Lindeman RD, Tobin JD, Shock NW: Association between bloodpressure and the rate of decline in renal function with age. Kidney Int26: 861–868, 1984

11. Rule AD, Amer H, Cornell LD, Taler SJ, Cosio FG, Kremers WK,Textor SC, Stegall MD: The association between age and nephro-sclerosis on renal biopsy among healthy adults. Ann Intern Med 152:561–567, 2010

12. Cockcroft DW, Gault MH: Prediction of creatinine clearance fromserum creatinine. Nephron 16: 31–41, 1976

13. Levey AS, Bosch JP, Lewis JB, Greene T, Rogers N, Roth D: A moreaccurate method to estimate glomerular filtration rate from serumcreatinine: A new prediction equation. Modification of Diet in RenalDisease Study Group. Ann Intern Med 130: 461–470, 1999

14. Klahr S: The modification of diet in renal disease study. N EnglJ Med 320: 864–866, 1989

15. Fehrman-Ekholm I, Skeppholm L: Renal function in the elderly (�70years old) measured by means of iohexol clearance, serum creatinine,serum urea and estimated clearance. Scand J Urol Nephrol 38: 73–77,2004

16. Carnevale V, Pastore L, Camaioni M, Mellozzi M, Sabatini M,Arietti E, Fusilli S, Scillitani A, Pontecorvi M: Estimate of renalfunction in oldest old inpatients by MDRD study equation, MayoClinic equation and creatinine clearance. J Nephrol 23: 306–313,2010

17. Heras M, Guerrero MT, Fernandez-Reyes MJ, Sanchez R, Munoz A,Cruz Macías M, Molina A, Rodríguez A, Prado F, Alvarez-Ude F:Estimation of glomerular filtration rate in persons aged 69 years orolder: Agreement between distinct calculation methods [in Spanish].Rev Esp Geriatr Gerontol 45: 86–88, 2010

18. Rule AD, Larson TS, Bergstralh EJ, Slezak JM, Jacobsen SJ, CosioFG: Using serum creatinine to estimate glomerular filtration rate:Accuracy in good health and in chronic kidney disease. Ann InternMed 141: 929–937, 2004

19. Levey AS, Stevens LA, Schmid CH, Zhang YL, Castro AF 3rd,Feldman HI, Kusek JW, Eggers P, Van Lente F, Greene T, Coresh J,CKD-EPI (Chronic Kidney Disease Epidemiology Collaboration): Anew equation to estimate glomerular filtration rate. Ann Intern Med150: 604–612, 2009

20. Stevens LA, Schmid CH, Greene T, Zhang YL, Beck GJ, Froissart M,Hamm LL, Lewis JB, Mauer M, Navis GJ, Steffes MW, Eggers PW,Coresh J, Levey AS: Comparative performance of the CKD Epide-miology Collaboration (CKD-EPI) and the Modification of Diet inRenal Disease (MDRD) study equations for estimating GFR levelsabove 60 ml/min/1.73 m2. Am J Kidney Dis 56: 486–495, 2010

21. Shlipak MG, Sarnak MJ, Katz R, Fried LF, Seliger SL, Newman AB,Siscovick DS, Stehman-Breen C: Cystatin C and the risk of death andcardiovascular events among elderly persons. N Engl J Med 352:2049–2060, 2005

22. Shlipak MG, Katz R, Sarnak MJ, Fried LF, Newman AB, Stehman-Breen C, Seliger SL, Kestenbaum B, Psaty B, Tracy RP, SiscovickDS: Cystatin C and prognosis for cardiovascular and kidney out-comes in elderly persons without chronic kidney disease. Ann InternMed 145: 237–246, 2006

23. Menon V, Shlipak MG, Wang X, Coresh J, Greene T, Stevens L,Kusek JW, Beck GJ, Collins AJ, Levey AS, Sarnak MJ: Cystatin Cas a risk factor for outcomes in chronic kidney disease. Ann InternMed 147: 19–27, 2007

24. Fried LP, Kronmal RA, Newman AB, Bild DE, Mittelmark MB,Polak JF, Robbins JA, Gardin JM: Risk factors for 5-year mortalityin older adults: The Cardiovascular Health Study. JAMA 279: 585–592, 1998

25. Walter LC, Brand RJ, Counsell SR, Palmer RM, Landefeld CS,Fortinsky RH, Covinsky KE: Development and validation of a

22 Nephrology Self-Assessment Program - Vol 10, No 1, January 2011

Page 35: 2011.01 - Geriatric Nephrology

prognostic index for 1-year mortality in older adults after hospital-ization. JAMA 285: 2987–2994, 2001

26. Go AS, Chertow GM, Fan D, McCulloch CE, Hsu CY: Chronickidney disease and the risks of death, cardiovascular events, andhospitalization. N Engl J Med 351: 1296–1305, 2004

27. Crowe E, Halpin D, Stevens P: Early identification and managementof chronic kidney disease: Summary of NICE guidance. BMJ 337:a1530, 2008

28. Drey N, Roderick P, Mullee M, Rogerson M: A population-basedstudy of the incidence and outcomes of diagnosed chronic kidneydisease. Am J Kidney Dis 42: 677–684, 2003

29. O’Hare AM: The management of older adults with a low eGFR:Moving toward an individualized approach. Am J Kidney Dis 53:925–927, 2009

30. Foster MC, Hwang SJ, Larson MG, Parikh NI, Meigs JB, Vasan RS,Wang TJ, Levy D, Fox CS: Cross-classification of microalbuminuriaand reduced glomerular filtration rate: Associations between cardio-vascular disease risk factors and clinical outcomes. Arch Intern Med167: 1386–1392, 2007

31. Hemmelgarn BR, Manns BJ, Lloyd A, James MT, Klarenbach S,Quinn RR, Wiebe N, Tonelli M, Alberta Kidney Disease Network:Relation between kidney function, proteinuria, and adverse out-comes. JAMA 303: 423–429, 2010

32. Tonelli M, Jose P, Curhan G, Sacks F, Braunwald E, Pfeffer M:Proteinuria, impaired kidney function, and adverse outcomes inpeople with coronary disease: Analysis of a previously conductedrandomised trial. BMJ 332: 1426, 2006

33. de Boer IH, Katz R, Cao JJ, Fried LF, Kestenbaum B, Mukamal K,Rifkin DE, Sarnak MJ, Shlipak MG, Siscovick DS: Cystatin C,albuminuria, and mortality among older adults with diabetes. Diabe-tes Care 32: 1833–1838, 2009

34. Rifkin DE, Katz R, Chonchol M, Fried LF, Cao J, de Boer IH,Siscovick DS, Shlipak MG, Sarnak MJ: Albuminuria, impairedkidney function and cardiovascular outcomes or mortality in theelderly. Nephrol Dial Transplant 25: 1560–1567, 2010

35. Astor BC, Hallan SI, Miller ER 3rd, Yeung E, Coresh J: Glomerularfiltration rate, albuminuria, and risk of cardiovascular and all-causemortality in the US population. Am J Epidemiol 167: 1226–1234,2008

36. Hallan S, Astor B, Romundstad S, Aasarod K, Kvenild K, Coresh J:Association of kidney function and albuminuria with cardiovascularmortality in older vs younger individuals: The HUNT II Study. ArchIntern Med 167: 2490–2496, 2007

37. Conway B, Webster A, Ramsay G, Morgan N, Neary J, Whitworth C,Harty J: Predicting mortality and uptake of renal replacement therapyin patients with stage 4 chronic kidney disease. Nephrol Dial Trans-plant 24: 1930–1937, 2009

38. Chronic Kidney Disease Prognosis Consortium, Matsushita K, vander Velde M, Astor BC, Woodward M, Levey AS, de Jong PE,Coresh J, Gansevoort R: Association of estimated glomerular filtra-tion rate and albuminuria with all-cause and cardiovascular mortalityin general population cohorts: A collaborative meta-analysis. Lancet375: 2073–2081, 2010

39. Keith DS, Nichols GA, Gullion CM, Brown JB, Smith DH: Longi-tudinal follow-up and outcomes among a population with chronickidney disease in a large managed care organization. Arch Intern Med164: 659–663, 2004

40. Eriksen BO, Ingebretsen OC: The progression of chronic kidneydisease: A 10-year population-based study of the effects of genderand age. Kidney Int 69: 375–382, 2006

41. Evans M, Fryzek JP, Elinder CG, Cohen SS, McLaughlin JK, NyrenO, Fored CM: The natural history of chronic renal failure: Resultsfrom an unselected, population-based, inception cohort in Sweden.Am J Kidney Dis 46: 863–870, 2005

42. Johnson ES, Thorp ML, Yang X, Charansonney OL, Smith DH:

Predicting renal replacement therapy and mortality in CKD. Am JKidney Dis 50: 559–565, 2007

43. Hsu CY, Iribarren C, McCulloch CE, Darbinian J, Go AS: Riskfactors for end-stage renal disease: 25-year follow-up. Arch InternMed 169: 342–350, 2009

44. Ishani A, Grandits GA, Grimm RH, Svendsen KH, Collins AJ,Prineas RJ, Neaton JD: Association of single measurements ofdipstick proteinuria, estimated glomerular filtration rate, and hemat-ocrit with 25-year incidence of end-stage renal disease in the multiplerisk factor intervention trial. J Am Soc Nephrol 17: 1444–1452, 2006

45. Lopes AA: Relationships of race and ethnicity to progression ofkidney dysfunction and clinical outcomes in patients with chronickidney failure. Adv Ren Replace Ther 11: 14–23, 2004

46. Brancati FL, Whelton PK, Randall BL, Neaton JD, Stamler J, KlagMJ: Risk of end-stage renal disease in diabetes mellitus: A prospec-tive cohort study of men screened for MRFIT. Multiple Risk FactorIntervention Trial. JAMA 278: 2069–2074, 1997

47. Klag MJ, Whelton PK, Randall BL, Neaton JD, Brancati FL, FordCE, Shulman NB, Stamler J: Blood pressure and end-stage renaldisease in men. N Engl J Med 334: 13–18, 1996

48. Hallan SI, Ritz E, Lydersen S, Romundstad S, Kvenild K, Orth SR:Combining GFR and albuminuria to classify CKD improves predic-tion of ESRD. J Am Soc Nephrol 20: 1069–1077, 2009

Geriatric HypertensionThe recent NephSAP (1) on hypertension pro-

vided an up-to-date review of hypertension for thenephrologist; in this section, previous content on hy-pertension involving the geriatric population is up-dated and expanded. A majority of Americans who areolder than 60 years of age have high BP, and theprevalence is increasing (2)—a “hypertension epi-demic” (3), mainly of the isolated systolic hyperten-sion type. Hypertension is defined by a systolic BP of�140 mmHg or a diastolic BP of �90 mmHg inde-pendent of age, yet its incidence rises with aging toaffect more than two thirds of elderly individuals(Figures 1 and 2) (4,5). Hypertension in the aged is amajor risk factor for all major aspects of cardiovascu-lar disease, including coronary ischemic events (6),heart failure (7), peripheral vascular disease (8), andstroke (9). As emphasized by the National High BloodPressure Education Working Group (10), elevation ofsystemic pressure in the elderly parallels the risk forcardiovascular morbidity and mortality. Advanced ageis one of the primary risk factors for atheroscleroticcardiovascular disease, and it has recently been pro-posed that early vascular aging (the cumulative influ-ence of cardiovascular risk factors on the arterialvasculature) should be a central concept in understand-ing individual who are at increased risk (11) (Figure 3).Nonetheless, a diagnosis of hypertension seems tocarry less importance in the elderly: A recent Instituteof Medicine report indicated that physicians are lessaggressive about treating older patients with hyperten-

Nephrology Self-Assessment Program - Vol 10, No 1, January 2011 23

Page 36: 2011.01 - Geriatric Nephrology

sion despite the fact that they are more likely to havethe condition and benefit from therapy (12). However,efficacy of antihypertensive treatment seems to dimin-ish with age (13) such that not more than 25% ofelderly patients with hypertension have controlled BPlevels (3).

MechanismsDespite the high incidence of hypertension in

aging humans, a rise in BP is not intrinsic to aging inmany populations or in other species, and the precisepathophysiology causing hypertension in the elderlycompared with younger individuals is not fully deter-mined. Mechanisms that contribute to vessel senes-cence continue to be investigated. Although measure-ment of age is simple, it may be considered a surrogatefor the more complex phenomena involving geneticsand environment. For example, the effects of genesinvolved in the complex trait BP may be modulated byage. Recent data from the Hypertensive Genetics Ep-idemiology Network (HyperGEN) study suggestedthat genetic effects on BP vary by age (14), using avariance component statistical method that incorpo-rates age variation. Another recent report (15) sug-gested that age modified the effect of glycemic statuson aortic compliance. In individuals from the Multi-

Ethnic Study of Atherosclerosis (MESA), general lin-ear models were used to determine whether age itselfmodified the effect of glucose status (normal or im-paired fasting or diabetes) on aortic distensibility. Inthose aged �65 years, the fasting glucose category nolonger was predictive of aortic distensibility. Abnor-malities in the regulatory functions of the vascularendothelium, including overproduction of vasocon-strictor prostaglandins, are associated with advancingage (16). Aging is associated with progressive declineof nitric oxide activity in vessel walls (17). Impairedprostacyclin-mediated vasodilatation was recently re-ported in older patients after local infusion of prosta-cyclin into the brachial artery (18).

It is not commonly appreciated that the normal

Figure 1. Prevalence of high BP in adults aged �20 yearsby age and gender in the United States (NHANES 2005 to2006). Hypertension is defined as systolic BP �140 mmHgor diastolic BP �90 mmHg, taking antihypertensive medi-cation, or being told twice by a physician or other profes-sional that one has hypertension. Reprinted with permissionfrom reference 4 (Lloyd-Jones D, Adams R, Caranethon M,DeSimone G, Ferguson TB, Flegal K, Ho EM, Howard V,Kissela B, Kittner S, Lackland D, Lisabeth L, MarelliA, McDermott M, Meigs J, Mozaffarian D, Nichol G,O’Donnell C, Roger V, Thom WT, Wassertheil-Smoller S,Wong N, Wylie-Rosett J, Hong Y: Heart disease and strokestatistics: 2009 Update. Circulation 119: e21–e181, 2009).

100

110

120

130

140

150

5 15 25 35 45 55 65 75 85

Age (y)

Nichols et al,17 1985 (aortic, regression line)

National Heart Foundation of Australia,12 1989

Burt et al,14 1995—white males (US NHANES)

Franklin et al,16 1997—Framingham Heart Study

Uiterwaal et al,15 1997

Staessen et al,13 1990

Jiang,18 2005 (China)

Pres

sure

(m

m H

g)

30

40

50

60

70

80

5 15 25 35 45 55 65 75 85

Age (y)

Burt et al,14 1995 (US NHANES)

Franklin et al,16 1997—Framingham Heart Study

Lakatta,19 2003

Jiang,18 2005 (China)

Pres

sure

(m

m H

g)

Figure 2. Changes in systolic BP (top) and pulse pressure(bottom) with age in studies of over 1 million individualscollectively. See reference 5 for individual reports. Origi-nally published in Current Hypertension Reports. Adaptedby O’Rourke and Seward in reference 5. Figure above hasbeen adapted from reference 5 with kind permission fromSpringer Science�Business Media: O’Rourke MF, Isolatedsystolic hypertension, pulse pressure, and arterial stiffnessas risk factors for cardiovascular disease. Curr HypertensRep 1: 204–211, 1999.

24 Nephrology Self-Assessment Program - Vol 10, No 1, January 2011

Page 37: 2011.01 - Geriatric Nephrology

aorta serves as a low compliance reservoir for storingcardiac stroke volume and evening out circulatoryflow to target organs. Aging even in the absence ofhypertension displaces the curve that relates volume topressure in large arteries, indicating a decrease inelastic arterial compliance. Arterial stiffness increaseswith advancing age, even in relatively healthy individ-uals (19). When calculated indirectly for the aorta,aortic, systolic, and pulse pressures all rise progres-sively and linearly with age (20). The associated risein systolic and pulse pressure with aging largely ac-counts for the fact that two thirds of the elderlypopulation has hypertension, mainly in the form ofisolated systolic hypertension (21).

Arterial stiffness is an important predictor ofcardiovascular events beyond traditional risk factors(22,23). Older people with elevated systolic and dia-stolic pressure levels have increased cardiovascularrisks (24). However, increased systolic and pulse pres-sures are stronger risk factors for cardiovascular mor-bidity and mortality in elderly individuals than isincreased diastolic pressure (25), and in those withisolated systolic hypertension, an increased pulse pres-sure is the best risk marker. In a recent report, adramatic decrease in aortic arch distensibility wasshown before the fifth decade of life in individualswho were of varying ages (26) and were free of overtcardiovascular disease, using magnetic resonance im-aging, ultrasound, and tonometry. The results suggest

that large artery stiffening may be an early manifesta-tion of vascular aging in humans.

Carotid-femoral pulse wave velocity (cfPWV),an established measure of intrinsic aortic wall stiff-ness, was recently reviewed as a predictor of cardio-vascular events (27). The association of cfPWV withboth aging and BP was confirmed. Other powerful riskfactors for atherosclerosis, such as gender, smoking, andlipids, however, were not associated with cfPWV. Inde-pendent factors that result in early vascular aging, includ-ing those related to endothelial dysfunction, have beensought to determine how aortic stiffness occurs indepen-dent of the atherosclerotic process, in the form of “tissue”or ”circulating” biomarkers. Tissue biomarkers are mea-surable parameters, such as arterial stiffness, central BP,carotid intimal-medial thickness, and endothelial dys-function, and are associated with target organ damage inthe form of left ventricular hypertrophy, microalbumin-uria, reduced GFR, and cerebral white matter lesions(11). Circulating biomarkers, such as high-sensitivityC-reactive protein and homocysteine, remain popular butseem to have negligible value.

Arterial stiffness increases with advancingage and is an independent predictor ofcardiovascular events. Cf-PWV is a mea-sure of intrinsic aortic wall stiffness. Cen-tral pressure assessment may improveidentification and treatment of patientswith increased cardiovascular risk.

In a recent systematic review of cross-sectionalpublished literature using multivariable regressionmodels, Cecilja et al. (27) found that age was inde-pendently associated with cfPWV changes in �90%of studies. Elevated pulse pressure is a surrogatemeasure for increased proximal aortic stiffness. Struc-tural changes in the anatomy and mechanical proper-ties of the large arteries have traditionally been used toproduce directly and indirectly the common pattern ofisolated systolic hypertension and have been increas-ingly analyzed through wave reflection analysis (28).The major sites of origin of the reflected wave are thearterioles. The incident and reflected waves combine toproduce the pressure measured at any point in the arterialcircuit. Aortic stiffness indirectly increases the pressurewave generated by ventricular ejection and also prema-

100%abnormal

100%normal

0

arterial stiffness

40th yearCV events

80th year

MBP

glycemia

lipids

Risk score

Time at measurement

Figure 3. Damaging effects of cardiovascular risk factors onthe arterial wall with aging. Arterial stiffness increases withaging, although traditional risk factors such as hypertension,glycemia, and lipids may fluctuate and their combined riskscore not increase. MBP, mean BP; CV, cardiovascular.Reprinted with permission from reference 11 (Nilsson PM,Boutouyrie P, Laurent S: Vascular aging: A tale of EVA andADAM in cardiovascular risk assessment and prevention.Hypertension 54: 3–10, 2009).

Nephrology Self-Assessment Program - Vol 10, No 1, January 2011 25

Page 38: 2011.01 - Geriatric Nephrology

turely returns the wave reflection to the heart fromperipheral sites. With increasing age, if the reflectionwave advances from diastole into systole, it will in-crease afterload. Clinically, the reflected wave willincrease systolic but not diastolic pressure, therebyincreasing the pulse pressure. In a recent report, aorticpressure waveforms were recorded and analyzed sep-arately for 3682 healthy individuals who were olderand younger than 60 years to evaluate the relativeimportance of premature return of wave reflection(indirect effect of vessel stiffness) and largeness of theincident wave (direct effect) (29). Whereas the re-flected wave was important across the lifespan, theincident wave contributed to systolic and pulse pres-sure elevations only beyond age 60 years. The hypoth-esis that systolic pressure elevations with aging aredue to changes in wave reflection was also analyzed ina recent meta-analysis (28) of 64 studies that reportedthe timing of wave reflection. In individuals of allages, reflection times were within systole, not diastole,with only a small tendency for younger individuals tohave later reflection. Finally, the link between loss ofaortic compliance and systolic hypertension was con-firmed in an animal model of aortic banding, whichresulted in a pressure waveform suggestive of aging(30) but no change in reflection timing.

Basic hemodynamic mechanisms of blood flowinvolving central arteries are of increasing relevance tothe basic pathophysiology of cardiovascular disease(31). Pulse pressure, as calculated by pulse waveanalysis, is predictive of cardiovascular outcomes. Ithas been proposed that central pressure assessmentmay improve the identification and treatment of pa-tients with increased cardiovascular risk (32). A morecentral pressure reading, instead of an estimate frombrachial pressure readings, could better represent thehemodynamic load imposed on the left ventricle andimportant coronary and brain circulation because of itsproximity (33). Some studies were able to combinebrachial cuff values and the radial pulse contour togenerate noninvasively a central aortic pressure wave-form (34,35). In a recent cross-analysis of the StrongHeart Study (36), 2585 Native Americans underwentcentral BP monitoring with a validated device fornoninvasive measurements based on radial artery pres-sure waveforms and left ventricular assessment. Cen-tral systolic and pulse pressures correlated better withleft ventricular mass. Recent studies have suggestedthat central arterial pressure may rise disproportionately

with aging (29) and be an even better predictor ofcardiovascular outcomes. Pini et al. (37) evaluated acohort of 398 unselected elderly individuals with carotidultrasonography and applanation tonometry to assesscarotid artery central pressures over 8 years. In a multi-variate analysis, higher carotid systolic pressure andpulse pressure but neither brachial systolic nor pulsepressure independently predicted cardiovascular events.

Central Nervous System ComplicationsThe risks for central nervous system complica-

tions of hypertension such as stroke and dementiaincrease as a function of BP levels, and risk reductionis key to preventing these complications (38). The riskfor stroke increases exponentially with age, and theelderly experience worse outcomes and higher risk fordementia after strokes. Diuretics and angiotensin-con-verting enzyme inhibitors (ACEIs) were recom-mended in the Joint National Committee on Preven-tion, Detection, Evaluation, and Treatment of HighBlood Pressure (JNC7) to prevent recurrent stroke inthe elderly patient with hypertension (39). Stroke-related issues in the patient with hypertension wereextensively addressed in a previous issue of NephSAP(1), including which agents to use, optimal BP targets,worsening of stroke damage with excessive BP low-ering, and stroke protection. As recently addressed bySpence (40), elderly patients with hypertension arelikely to have hypertrophy in their cerebral arteries,resulting in impaired autoregulation, better toleranceof higher BP, and intolerance to falls in BP. In thepresence of ischemia, cerebral blood flow autoregula-

Figure 4. Loss of cerebral blood flow regulation duringacute ischemic stroke. The physiologic regulation of cere-bral blood flow over a wide range of perfusion pressures isshifted to the right in chronic hypertension. In conditions ofacute ischemia, cerebral blood flow no longer autoregulatesand drops with hypotension. Note that with the rightwardshift in hypertension, cerebral blood flow will autoregulateat higher BP. Reprinted with permission from reference 40(Spence JD: Treating hypertension in acute ischemic stroke:Editorial commentary. Hypertension 54: 702–703, 2009).

26 Nephrology Self-Assessment Program - Vol 10, No 1, January 2011

Page 39: 2011.01 - Geriatric Nephrology

tion is further altered (Figure 4), with higher pressuresaggravating cerebral edema. Reduction in perfusionpressure in ischemic cerebral regions risks neurologicdeterioration. Systolic pressure is the most importantrisk predictor for stroke. In a recent prospective ob-servational study involving 1092 patients with isch-emic stroke, initial systolic BP �187 mmHg wasassociated with worse prognosis (41). The influence ofsystolic pressure changes in the first poststroke hoursdepended on patient age: Excessive reductions in sys-tolic pressure determined a worse prognosis in theelderly. In patients with systolic pressure reductions of�27 mmHg within the first 8 hours, the likelihood ofa poor subsequent outcome was multiplied by six inpatients who were aged 70 to 76, by 10 in patients whowere aged 76 to 80, and by 15 in patients who wereolder than 80 years, irrespective of treatment.

Hypertension and advancing age are the stron-gest risk factors for dementia, for both the vascularand the Alzheimer types. In patients with cerebrovas-cular disease, the risk for dementia is a function of BPlevels. However, the nature of the relation betweenhypertension and dementia in the elderly is complex.Longstanding studies have indicated that systolic BP isa risk factor for dementia (42). Raised BP in midlifeseems to be associated with dementia, particularly inthose whose hypertension is untreated (43). In un-treated adults who were aged �65 years, high systolicBP was recently associated with cerebrovascular dam-age at autopsy (44). Hypertension doubled the pro-gression to dementia in patients who had an averageage of 83 years and cognitive impairment and werefollowed for 5 years in the recent Canadian Study ofHealth and Aging (45). Another study examined long-term trajectories in BP in a cohort of 1879 Japanese-American men who were followed for 32 years in theHonolulu Heart Program/Honolulu-Asian Aging Study(46). In an analysis based on survivors to ages 77 to 98years, men who were not being treated for hyperten-sion and went on to develop dementia had a slightage-adjusted increase in midlife systolic BP (0.26mmHg/y) compared with survivors without dementiaand a greater age-adjusted decrease in systolic pres-sure (�10 mmHg in 58%) during a 6-year periodbefore the dementia diagnosis. Dementia was previ-ously found to be associated with low resting andorthostatic pressures. In a recent review of studiespublished during a 13-year period, Shah et al. (47)examined the relationship between use of antihyper-

tensive mediations and the incidence/progression ofAlzheimer dementia, vascular dementia, and unspeci-fied dementia, focusing on randomized, controlledtrials of patients who were older than 45 years. In 12original studies included in the review, only ACEIsand diuretics reduced significantly the risk for demen-tia and its progression in the majority of studies. In theHypertension in the Very Elderly Trial (HYVET)substudy, dementia decreased 14% with BP loweringbut not significantly (48). The association of ACEIsand cognition may depend on whether the drug crossesthe blood-brain barrier: Exposure to non–centrallyactive ACEIs was associated with a greater risk forincident dementia compared with other classes ofdrugs, whereas ACEIs that do cross the barrier (cap-topril, fosinopril, lisinopril, perindopril, ramipril, andtrandolapril) were associated with a 65% reduction incognitive decline per year in a recent study (49).

TreatmentThe efficacy, use, and adverse effects of antihy-

pertensive drug therapy in older adults have beenrecently reviewed (50). Antihypertensive therapyshould be considered in all patients, regardless of age,although caution is necessary in frail elderly patients(51) and higher BP goals may be appropriate in pa-tients with early dementia. The benefits of treatmentare more likely to outweigh the risks for adverseeffects the younger the patient and the milder thehypertension. All antihypertensive drugs may predis-pose the elderly patient to symptomatic orthostatic andpostprandial hypotension (52). Older patients withhypertension managed appropriately will gain particu-lar benefit in cardiovascular outcomes such as coronaryevents, congestive heart failure, and stroke comparedwith younger patients (53). The JNC7 recommended thatthe general BP goal in older patients be �140/�90, and�130/�80 for those with diabetes or chronic renalinsufficiency, and some subsequent position papershave extended the lower goal to patients with coronaryartery disease and heart failure (54). Several majorclinical trials of older patients have shown markedreductions in cardiovascular events with the use ofantihypertensive medications (Table 1) (55–57). Arecent Cochrane Database analysis of 15 randomized,controlled trials of at least 1 year’s duration of patientswho were at least 60 years of age and had moderate tosevere hypertension concluded that treating healthyolder patients with hypertension reduced all-cause

Nephrology Self-Assessment Program - Vol 10, No 1, January 2011 27

Page 40: 2011.01 - Geriatric Nephrology

mortality (relative risk 0.90) and cardiovascular mor-bidity and mortality (relative risk 0.72) (58). Three ofthe trials that were restricted to patients with isolatedsystolic hypertension showed similar benefits. Out-come trials of isolated systolic hypertension in theelderly were the subject of a previous meta-analysis(59). In eight trials, active treatment reduced totalmortality by 13%, cardiovascular mortality by 18%,coronary events by 23%, stroke by 30%, and allcardiovascular complications by 26%.

Nonetheless, old age remains a barrier to suc-cessful treatment for hypertension. Despite this un-equivocal prevention of cardiovascular disease achiev-able by reduction in BP, roughly half of patients havefailed to reach the goal pressures in studies such asSystolic Hypertension in the Elderly Program (SHEP)and SYSTEur (60). In an analysis of National Healthand Nutrition Examination Survey (NHANES) datacomparing African American and Caucasian adultswith hypertension over time, the increased likelihoodof hypertension that was pharmacologically treated butundercontrolled in African American patients ex-tended to those who were older than 60 years (61).

Clinical observations also indicate that the inten-sity and efficacy of treatments decrease in the elderly.A study from Sweden reported that practitioners ac-cepted higher BP than recommended in clinical guide-lines for the elderly (62), with only 20% of treatedpatients reaching goal pressures. Such “clinical iner-tia” (63) reflecting reluctance to implement therapieswhen combined with poor adherence in the elderlycontribute to the lowest rates of achievement of BPgoals in the elderly. A study by Bailey et al. (13)explored why efficacy of treatments decreases withage using survival analysis techniques applied to hy-pertension treatment intensity from a Mayo Clinicdatabase. Across age ranges, increased treatment in-tensity was associated with control of hypertension

(Figure 5). However, a decrease in control rates wasobserved at any given intensity with older age, andolder patients experienced a lower intensity of treat-ment, which the investigators attributed to both phy-sician choice and patient acceptance.

Hypertension in the Very Elderly TrialConcerns about patient safety may also affect

treatment intensity in the elderly. One pilot study for atrial of lowering BP in the elderly reported an in-creased mortality that offset the benefit of a reductionin stroke risk (64). A post hoc analysis of 1670individuals who were aged �80 years from severalantihypertensive trials noted reductions in cardiovas-cular morbidity but a possible small increase in all-cause mortality (65). The benefits and concerns about

Table 1. Effects of antihypertensive treatment on cardiovascular outcomes in the elderly in four major clinicalstudies

Parameter HYVET SHEP STOP Syst-Eur

Mean treatment BP reduction, SBP/DBP, mmHg �29/�13 �27/�9 �29/�17 �23/�7Stroke, % reduction �30 �32 �47 �42Coronary disease, % reduction �23a �27 �13 �30Heart failure, % reduction �64 �55 �51 �29

HYVET, Hypertension in Very Elderly Trial (66); SHEP, Systolic Hypertension in the Elderly Program (55); STOP, Swedish Trial in Old Patients (56); Syst-Eur, EuropeanSystolic Hypertension in the Elderly (57); DBP, diastolic BP; SBP, systolic BP.

aPercentage of cardiovascular mortality reduction.

0

20

40

60

80

100

0 0.5 1.0 1.5 2.0 2.5 3.0Total intensity

(number of maximum doses)

In c

ontro

l, (%

)

15-39 40-49 50-59 60-69 70-79 80 +

Figure 5. Treatment efficacy and intensity according to age.Shown is the predicted cumulative probability of BP controlwith varying treatment intensity, for various age groups,using Kaplan-Meier methods. The model suggests that evenif elderly patients are treated aggressively, control rates willbe lower than for younger patients. Reprinted with permis-sion from reference 13 (Bailey KR, Grossardt BR, GravesJW: Novel use of Kaplan-Meier methods to explain age andgender differences in hypertension control rates. Hyperten-sion 51: 841–847, 2008).

28 Nephrology Self-Assessment Program - Vol 10, No 1, January 2011

Page 41: 2011.01 - Geriatric Nephrology

aggressively treating those who are older than 80 yearswere evaluated in the recent HYVET (66), the firstrandomized, controlled trial of elderly patients withhypertension. After a run-in period of 2 months, 3868patients who were from 13 countries and had a meanage of 83.6 years (range 80 to 105 years) and sustainedsystolic BP of �160 mmHg were randomly assignedto either the diuretic indapamide (sustained release 1.5mg) or placebo. The ACEI perindopril (2 or 4 mg) orplacebo was added as needed to achieve the target BPof 150/80 mmHg. Exclusion criteria were acceleratedhypertension, secondary hypertension, a serum creat-inine level of �1.7 mg/dl, a recent hemorrhagicstroke, heart failure, dementia, and need for nursingcare. All antihypertensive medications were stoppedbefore run-in. Secondary end points included deathfrom cardiovascular event, stroke, or any cause. Thetarget BP was reached after a median duration offollow-up of 1.8 years in only 48% of the activetreatment group (versus 20% with placebo); three quar-ters required combination therapy. The benefits of treat-ment became apparent within the first year (Figure 6),and the trial was terminated at the second interimanalysis. Active treatment was associated with a 30%reduction in the rate of fatal or nonfatal stroke (P �0.06), 39% reduction in the rate of death from stroke(P � 0.05), 64% reduction in heart failure (P �0.0001), and unexpected reductions cardiovascular(23%; P � 0.06) and all-cause (21%; P � 0.02)mortality. The overall death rate in the trial was only53.1 per 1000 patient-years. Furthermore, the numberof serious adverse events was higher in the placebogroup (P � 0.001). Changes from baseline in serumpotassium or creatinine did not differ.

In the HYVET, the benefits of treatmentbecame apparent within the first year andat conclusion were associated with reduc-tions in stroke, heart failure, and mortality.

One important caveat is that HYVET-eligible patientswere generally healthier than the general very oldpopulation. The prevalence of baseline cardiovasculardisease, for example, was only 12%. Increased riskmay be anticipated should lower BP targets be set.Conversely, reductions in cardiovascular events in

elderly patients with a higher prevalence of cardiovas-cular disease may be even greater. A recent analysisthat applied the HYVET study to the Swiss health caresystem showed economic benefit for those who wereaged �80 years (67).

Lifestyle ModificationBecause antihypertensive medications reduce but

do not eliminate risk of hypertension in the generalpopulation and substantial evidence supports a rela-tionship between BP and nutrition, medical adviceabout lifestyle modification, such as sodium restrictionand weight loss, is a common part of the antihyper-tensive regimen. In general, lifestyle interventions arestrongly encouraged for hypertension. A study of olderpatients indicated that patient belief that medicationsare not the only way to treat hypertension was asso-ciated with better BP control rates (68). The CanadianSociety for Hypertension’s evidence-based recom-mendations in 2005 included six lifestyle modifica-tions: Aerobic exercise, healthy body weight, alcohollimitation, DASH diet, sodium restriction, and stressmanagement where appropriate. Reduced salt tasteperception may lead to increased salt intake in theelderly (69). Whether lifestyle modifications apply toelderly hypertension is unclear. Few studies have ex-amined the role of formal lifestyle modification inhypertension management in the elderly. The role ofnonpharmacologic treatment in elderly patients withhypertension was conclusively evaluated in the Trialof Nonpharmacologic Interventions in the Elderly(TONE), the largest multicenter clinical trial of therole of lifestyle modifications in the elderly withhypertension, which randomly assigned 585 over-weight and 390 normal-weight elderly individuals whohad mild hypertension (�145/�85) and were taking asingle antihypertensive medication (70). The over-weight participants were randomly assigned to sodiumrestriction (1.8 g/24 h), weight reduction (goal 10 lb),both, or usual care. Normal-weight participants wererandomly assigned to sodium reduction or usual care.The primary end points were the development ofhypertension (190/110 mmHg at a single visit or meanpressure of 150/90 mmHg) after withdrawal of medi-cation. Nutritionists and exercise counselors moni-tored and motivated participants during a 4-monthperiod. During the study, the sodium restriction goalwas met by 38 to 40% of individuals. Weight lossaveraged 8 to 10 lb. At termination of the study, 30%

Nephrology Self-Assessment Program - Vol 10, No 1, January 2011 29

Page 42: 2011.01 - Geriatric Nephrology

No.

of E

vent

s pe

r 100

Pat

ient

s

8

7

6

4

3

1

5

2

00 1 2 3 4

Placebogroup

Active-treatment

group

Placebogroup

Active-treatment

group

Placebogroup

Active-treatment

group

Placebogroup

Active-treatment

group

Follow-up (yr)

C Death from Cardiovascular Causes

A Fatal or Nonfatal Stroke

P=0.06

No.

of E

vent

s pe

r 100

Pat

ient

s

8

6

10

12

4

2

00 1 2 3 4

Follow-up (yr)

P=0.06

No. at RiskPlacebo groupActive-treatment group

19121933

14841557

807873

374417

194229

19121933

14921565

814877

379420

202231

19121933

14921565

814877

379420

202231

19121933

14921565

814877

379420

202231

No. at RiskPlacebo groupActive-treatment group

No.

of E

vent

s pe

r 100

Pat

ient

s

30

10

20

00 1 2 3 4

Follow-up (yr)

D Death from Stroke

B Death from Any Cause

P=0.02

No.

of E

vent

s pe

r 100

Pat

ient

s

5

4

3

1

2

00 1 2 3 4

Follow-up (yr)

P=0.05

No. at RiskPlacebo groupActive-treatment group

No. at RiskPlacebo groupActive-treatment group

Placebogroup

Active-treatment

group

E Heart Failure

No.

of E

vent

s pe

r 100

Pat

ient

s 6

7

4

3

1

5

2

00 1 2 3 4

Follow-up (yr)

P<0.001

19121933

14801559

794872

367416

188228

No. at RiskPlacebo groupActive-treatment group

Figure 6. Results of active hypertension treatment in patients aged �80 years, from HYVET. The benefits of treatment beganto be evident within the first year of treatment. Reprinted with permission from reference 66 (Beckett NS, Peters R, FletcherAE: Treatment of hypertension in patients 80 yr of age or older. N Engl J Med 358: 1887–1898, 2008). Copyright © 2008Massachusetts Medical Society.

30 Nephrology Self-Assessment Program - Vol 10, No 1, January 2011

Page 43: 2011.01 - Geriatric Nephrology

were off antihypertensive medications, including 44%of those in the actively treated groups. Both interven-tions lowered the use of antihypertensive medications(sodium restriction group 31%; weight loss group36%; combined 55%). Despite these findings, olderpatients are less likely than other groups to receiveadvice from physicians about lifestyle modificationthat can lower BP levels (71). One recent study ana-lyzed data collected from the Centers for DiseaseControl and Prevention in a national survey of 38,457adults with hypertension. Those who were older than60 years were less likely to receive advice regardingeating habits or alcohol intake. A survey of US phy-sicians reported that lifestyle recommendations wereeven less likely to be given to the very old (72).

Renin-Angiotensin-Aldosterone BlockadeA meta-analysis of 31 hypertension trials involv-

ing 190,606 patients who had a mean age of 65 yearsreported that all classes of drugs were equally success-ful and that risk reduction was proportional to thereduction in systolic BP (73). Although the differencesbetween drug regimens on outcomes may be smallerthan the differences between BP levels (74), initialantihypertensive drug selection remains an importantdecision for the physician and the patient beingtreated. Despite preference given to thiazide diuretics,which were shown to reduce mortality in previousoutcome trials (75) as first-line therapy primarily onthe basis of JNC7 recommendations, debate continuesregarding the role of alternative classes in the elderly.Medication choices frequently depend on the extent ofother comorbid conditions. For example, JNC7 recom-mended various agents as accepted therapy for heartfailure (diuretics, � blockers, ACEIs, and angiotensinreceptor blockers [ARBs]), myocardial infarction (�blockers and ACEIs), and diabetes with chronic kid-ney disease (ACEI/ARBs) when hypertension requirestherapy (39).

Renin-angiotensin system (RAS)-blocking drugssuch as ACEIs and ARBs have been underused amongthe elderly, out of concern for potential complicationsof therapy and with awareness of limited life expect-ancy (76,77). Whether RAS blockade for hypertensionmight be more effective, the same, or less effective inthe elderly compared with younger patients remainsundetermined. The RAS may become less active withaging, with plasma renin and aldosterone concentra-tions generally reduced, although stimulation by up-

right posture, salt restriction, or volume depletion frombaseline is similar to younger individuals (78). Elderlyindividuals are largely considered to have low-renin,salt-sensitive hypertension. However, RAS blockadecould target two pathophysiologic perturbations in theelderly patient with hypertension, increased peripheralresistance, and decreased arterial compliance, as wellas improve endothelial dysfunction by inhibiting cy-clooxygenase-related vasoconstrictive forces (79). An-tihypertensive and Lipid-Lowering treatment to pre-vent Heart Attack Trial (ALLHAT) (80) includedpatients who were older than 55 years in a complexfour-drug protocol that yielded some comparisons ofthe ACEI lisinopril with the thiazide chlorthalidone.The two agents had similar efficacy for reducing theprimary endpoint of all-cause, fatal, and nonfatal cor-onary heart disease events and multiple secondaryoutcomes. However, benefit of the diuretic in prevent-ing some cardiovascular outcomes led to the studyconclusion that thiazide diuretics are superior toACEIs. That conclusion has been tempered by the factthat in the ACEI arm, BPs were higher by an averageof 4 to 5 mmHg. Subsequently, the Second AustralianNational Blood Pressure Study (ANBP2) compared anACEI with diuretic agents in 6083 patients who hadhypertension and were aged 65 to 84 years during amedian of 4.1 years in a prospective, open-label trialwith blinded assessment study design. BP reductionwas similar in both treatment groups. In the ACEIgroup, fewer cardiovascular events or deaths werenoted (81), and no increase in risks despite effectiveBP reduction were noted with the ACEI. Other studieshave indicated similar hypotensive efficacy of ARBsin elderly patients as well as evaluating their role inelderly patients with hypertension. Losartan was com-pared with atenolol in patients who had isolated sys-tolic hypertension and whose mean age was 70 yearsin the Losartan Intervention for Endpoint Reduction(LIFE) study (82). BP reduction of 28/9 mmHg wasachieved in both arms. The main outcome measure, acomposite of cardiovascular death, stroke, and myo-cardial infarction, was reduced by 25% with losartan.Patients who received the ARB had reductions incardiovascular mortality, fatal and nonfatal stroke,total mortality, and new-onset diabetes, whereas theincidence of myocardial infarction did not differ. Lo-sartan was also better tolerated. Valsartan was com-pared with the calcium channel blocker amlodipine in15,245 patients who were aged �50 years and had

Nephrology Self-Assessment Program - Vol 10, No 1, January 2011 31

Page 44: 2011.01 - Geriatric Nephrology

hypertension and risk for cardiac events in the ran-domized, double-blind Valsartan AntihypertensiveLong-term Use Evaluation (VALUE) trial (83). De-spite slightly lower BP in the amlodipine group (2.2 �0.2 versus 1.6 � 0.1 mmHg), the primary compositeend points of cardiac mortality and morbidity or totalmortality did not differ between the treatment groups.Prespecified safety analysis revealed a significant 23%reduction in new-onset diabetes (P � 0.0001) with theAT1 receptor antagonist compared with the calciumchannel blocker. Several studies of elderly patientshave reported excellent tolerance of AT1 receptorantagonists, including adverse event rates lower thangroups receiving ACEIs (77). Tolerance and lack ofsignificant drug interactions may be of particular valuein elderly patients who have hypertension and multiplecomorbidities and are taking several medications.

� Blockers are no longer supported as mono-therapy in elderly patients with hypertension unlessadministered for ischemic heart disease, past myocar-dial infarction, or heart failure with an impaired ejec-tion fraction (25). Data suggest marked underuse (84)even when there is no contraindication in the elderlypopulation. Most elderly patients will not have mildhypertension and will therefore require combinationtherapy.

Adverse Effects and AdherenceIn assessing the effect of age on control of

hypertension, adverse effects of medications and ad-herence to the treatment regimen are highly relevant(85). All antihypertensive drugs can predispose elderlypatients to symptomatic orthostatic hypotension andpostprandial hypotension, syncope, and falling epi-sodes. Poor medication adherence, cited by the Na-tional Council on Patient Information and Educationas “America’s other drug problem” (86), is a greaterrisk in the elderly: Adherence to antihypertensivemedications is commonly compromised because ofcost and complicated prescription regimens; whilecognitive impairment, depression, concerns aboutsafety and adverse effects of medications, poor com-munication with care providers, lack of symptomsattributable to elevated BP, fear of excessive loweringof pressure, and other attitudes and beliefs of theelderly individual (87) further reduce motivation. Arecent study showed that increased ambulatory carecopayments led to more hospitalizations, an effect thatwas magnified in the elderly population with hyper-

tension (88). The effect of adverse drug reactions onachieving treatment targets was examined in a recentanalysis of implementation of the HYVET recommen-dations in the very elderly (89). One hundred patientswho were aged �100 years, most with resistant hy-pertension, in a hypertension clinic were treated ac-cording to HYVET-based guidelines. Although morethan one third achieved BP control, 40% had docu-mented adverse drug reactions that limited furtherintervention (i.e., treatment was reduced, stopped, orleft unchanged because of adverse effects). However,clinically meaningful reductions in BP were achievedin a randomized, controlled trial of 200 community-based elderly patients who were taking medicationsfor hypertension and hypercholesterolemia using stan-dardized medication education, regular follow-up bypharmacists, and medications dispensed in a time-specific pack (90). Medication adherence was sus-tained through the entire 6 months of the interventionin 95% who were randomized to pharmacy care versus69% in the usual care group.

References1. Townsend RR, Textor SC: Nephrology Self-Assessment Program.

Hypertension 9: 55–105, 20102. Aslam F, Haque A, Agostini JV, Wang Y, Foody JM: Hypertension

prevalence and prescribing trends in older US adults: 1999–2004.J Clin Hypertens 12: 75–81, 2010

3. Stamler J: Setting the TONE for ending the hypertension epidemic.JAMA 279: 878–879, 1998

4. Lloyd-Jones D, Adams R, Caranethon M, DeSimone G, FergusonTB, Flegal K, Ho EM, Howard V, Kissela B, Kittner S, Lackland D,Lisabeth L, Marelli A, McDermott M, Meigs J, Mozaffarian D,Nichol G, O’Donnell C, Roger V, Thom WT, Wassertheil-Smoller S,Wong N, Wylie-Rosett J, Hong Y: Heart disease and stroke statistics:2009 update. Circulation 119: e21–e181, 2009

5. O’Rourke MF, Seward JB: Central arterial pressure and arterialpressure pulse: New views entering the second century after Korot-kov. Mayo Clin Proc 81: 1057–1068, 2006

6. Franklin SS, Larson MG, Khan SA: Does the relation of bloodpressure to coronary heart disease risk change with aging? TheFramingham Heart Study. Circulation 103: 1245–1249, 2001

7. Burney BO, Bakris GL: Hypertension and its management in theelderly. Semin Nephrol 29: 604–609, 2009

8. Aronow WS, Ahmed MI, Ekundayo OJ: A propensity-matched studyof the association of peripheral arterial disease with cardiovascularoutcomes in community-dwelling older adults. Am J Cardiol 103:13–135, 2009

9. Aronow WS, Frishman WH: Treatment of hypertension and preven-tion of ischemic stroke. Curr Cardiol Rep 6: 124–129, 2004

10. National High Blood Pressure Education Program Working Group:National High Blood Pressure Education Program working groupreport on hypertension in the elderly. Hypertension 23: 275–285,1994

11. Nilsson PM, Boutouyrie P, Laurent S: Vascular aging: A tale of EVAand ADAM in cardiovascular risk assessment and prevention. Hy-pertension 54: 3–10, 2009

12. Institute of Medicine: A Population-Based Policy and Systems

32 Nephrology Self-Assessment Program - Vol 10, No 1, January 2011

Page 45: 2011.01 - Geriatric Nephrology

Change Approach to Prevent and Control Hypertension, Washing-ton, DC, National Academies Press, 2010

13. Bailey KR, Grossardt BR, Graves JW: Novel use of Kaplan-Meiermethods to explain age and gender differences in hypertensioncontrol rates. Hypertension 51: 841–847, 2008

14. Shi G, Gu CC, Kraja AT, Arnett DK, Myers RH, Pankow JS, HuntSC, Rao DC: Genetic effect on blood pressure is modulated by age:The Hypertension Genetic Epidemiology Network Study. Hyperten-sion 53: 35–41, 2009

15. Stacey RB, Bertoni AG, Eng J, Bluemjke DA, Hundley WG, Her-rington D: Modification of the effect of glycemic status on aorticdistensibility by age in the Multi-Ethnic Study of Atherosclerosis.Hypertension 55: 26–32, 2010

16. Barac A, Panza JA: Mechanisms of decreased vascular function withaging. Hypertension 53: 900–902, 2009

17. Brandes RP, Fleming I, Busse R: Endothelial aging. Cardiovasc Res66: 286–294, 2005

18. Nicholson WT, Vaa B, Hesse C, Eisenach JH, Joyner MJ: Aging isassociated with reduced prostacyclin-mediated dilation in the humanforearm. Hypertension 53: 973–978, 2009

19. Mitchell GF, Parise H, Benjamin EJ: Changes in arterial stiffness andwave reflection with advancing age in healthy men and women: TheFramingham Heart Study. Hypertension 43: 1239–1245, 2004

20. O’Rourke MF: Isolated systolic hypertension, pulse pressure, andarterial stiffness as risk factors for cardiovascular disease. CurrHypertens Rep 1: 204–211, 1999

21. Payne RA, Wilkinson IB, Webb DJ: Arterial stiffness and hyperten-sion: Emerging concepts. Hypertension 55: 9–14, 2009

22. Ferrara LA, Guida L, Innelli P, Limauro S, Celentano A: Clinic andambulatory pulse pressure segregate a cluster of cardiovascular riskfactors. J Hum Hypertens 16: 719–723, 2002

23. Gasowski J, Fagard RH, Staessen JA, Grodzicki T, Pocock S,Boutitie F, Gueyffier F, Boissel JP: Pulsatile blood pressure compo-nent as predictor of mortality in hypertension: A meta-analysis ofclinical trial control groups. J Hypertens 20: 145–151, 2002

24. Domanski M, Mitchell G, Pfeffer M, Neaton JD, Norman J, SvendsenK, Cohen J, Stamler J, MRFIT Research Group: Pulse pressure andcardiovascular disease-related mortality. Follow-up study of the mul-tiple risk factor intervention trial (MRFIT). JAMA 287: 2677–2683,2002

25. Aronow WS: Hypertension in the elderly. Clin Geriatr Med 25:579–590, 2009

26. Reidheuil A, Yu WC, Wu CO, Mousseaux E, deCesare A, Yan R,Kachenoura N, Bluemke D, Lima JA: Reduced ascending aorticstrain and distensibility: Earliest manifestation of vascular aging inhumans. Hypertension 55: 319–326, 2010

27. Cecelja M, Chowienczyk P: Dissociation of aortic pulse wave veloc-ity with risk factors for cardiovascular disease other than hyperten-sion: A systematic review. Hypertension 54: 1328–1336, 2009

28. Baksi AJ, Treibel TA, Davies JE, Hadjiloizou N, Foale RA, ParkerKH, Francis DP, Mayet J, Hughes AD: A meta-analysis of themechanism of blood pressure change with aging. J Am Coll Cardiol54: 2087–2092, 2009

29. Namasivayam M, Mcdonnell BJ, McEniery CM, O’Rourke MF,Anglo-Cardiff Collaborative Trial Study Investigators: Does wavereflection dominate age-related change in aortic blood pressure acrossthe human life span? Hypertension 53: 979–985, 2009

30. Iannou CV, Stergiopulos N, Katsamouris AN: Hemodynamics in-duced after acute reduction of proximal thoracic aorta compliance.Eur J Vasc Endovasc Surg 26: 195–204, 2003

31. Cameron JD: Editorial commentary: Wave intensity analysis andcentral blood pressure. Hypertension 54: 958–959, 2009

32. McEniery CM, Yasmin A, McDonnell B, Munnery M, Wallace SM,Rowe CV, Cockcroft JR, Wilkinson IB, Anglo-Cardiff CollaborativeTrial Investigators: Central pressure: Variability and impact of car-

diovascular risk factors—The Anglo-Cardiff Collaborative Trial II.Hypertension 51:1476–1482, 2008

33. Schillaci G, Grassi G: Central blood pressure: Getting to the heart ofthe matter. J Hypertens 28: 237–239, 2010

34. O’Rourke MF, Hashimoto J: Mechanical factors in arterial ageing.J Am Coll Cardiol 50: 1–13, 2007

35. Roman MJ, Devereux RB, Kizer JR, Lee ET, Galloway JM, Ali T,Umans JG, Howard BV: Central pressure more strongly relates tovascular disease and outcome than does brachial pressure: The StrongHeart Study. Hypertension 50: 197–203, 2007

36. Roman MJ, Okin PM, Kizer JR, Lee ET, Howard BV, Devereux RB:Relations of central and brachial blood pressure to left ventricularhypertrophy and geometry: The Strong Heart Study. J Hypertens 28:384–388, 2010

37. Pini R, Cavallini MC, Palmiere V, Marchionni N, Bari MD, De-vereux RB, Masotti G, Roman MJ: Central but not brachial pressurepredicts cardiovascular events in an unselected geriatric population.J Am Coll Cardiol 51: 2432–2439, 2008

38. Pedelty L, Gorelick PB: Management of hypertension and cerebro-vascular disease in the elderly. Am J Med 121: S23–S31, 2008

39. Chobanian AV, Bakris GL, Black HR, Cushman WC, Green LA,Izzo JL Jr, Jones DW, Materson BJ, Oparil S, Wright JT Jr, RocellaEJ, Joint National Committee on Prevention, Detection, Evaluation,and Treatment of High Blood Pressure: National Heart, Lung, andBlood Institute; National High Blood Pressure Education ProgramCoordinating Committee: Seventh Report of the Joint National Com-mittee on Prevention, Evaluation, and Treatment of High BloodPressure. Hypertension 42: 1206–1252, 2003

40. Spence JD: Treating hypertension in acute ischemic stroke: Editorialcommentary. Hypertension 54: 702–703, 2009

41. Leira R, Millan M, Dfez-Tejedor Exuperio, Blanco M, Serena J,Fuente B, Rodriguez-Yanez M, Castellanos M, Lago A, Davalos A,Castillo J, TICA Study, Stroke Project, Cerebrovascular DiseasesGroup of the Spanish Neurological Society: Age determines theeffects of blood pressure lowering during the acute phase of ischemicstroke. Hypertension 54: 769–774, 2009

42. Qiu C, Winblad B, Fratiglioni L: The age-dependent relation of bloodpressure to cognitive function and dementia. Lancet Neurol 4: 487–499, 2005

43. Launer LJ, Masaki K, Petrovitch H, Foley D, Havlik RJ: Theassociation between midlife blood pressure levels and late-life cog-nitive function. The Honolulu-Asia Aging Study. JAMA 274: 1846–1851, 1995

44. Wang LY, Larson EB, Sonnen JA, Shofer JB, McCormick W, BowenJD, Montine TJ, Ge L: Blood pressure and brain injury in olderadults: Findings from a community-based autopsy study. J AmGeriatr Soc 57: 1975–1981, 2009

45. Oveishgaran S, Hachinski V: Hypertension, executive dysfunction,and progression to dementia: The Canadian Study of Health andAging. Arch Neurol 67: 187–192, 2010

46. Stewart R, Xue Q-L, Masaki K, Petrovitch H, Ross GW, White LR,Launer LJ: Change in blood pressure and incident dementia: A32-year prospective study. Hypertension 54: 233–240, 2009

47. Shah K, Qureshi SU, Johnson M, Parikh N, Schultz PE, Kunik ME:Does use of antihypertensive drugs affect the incidence or progres-sion of dementia? A systemic review. Am J Geriatr Pharmacother 7:250–261, 2009

48. Peters R, Beckett N, Forette F, Tuomilehto J, Clarke R, Ritchie C,Waldman A, Walton I, Poulter R, Ma S, Comsa M, Burch L, FletcherA, Bulpitt C, HYVET Investigators: Incident dementia and bloodpressure lowering in the Hypertension in the Very Elderly TrialCognitive Function Assessment (HYVET-COG): A double-blind,placebo controlled trial. Lancet Neurol 7: 683–689, 2008

49. Sink KM, Leng X, Williamson J, Kritchevsky SB, Yaffe K, Kuller L,Yasar S, Atkinson H, Robbins M, Psaty B, Goff DC Jr: Angiotensin-

Nephrology Self-Assessment Program - Vol 10, No 1, January 2011 33

Page 46: 2011.01 - Geriatric Nephrology

converting enzyme inhibitors and cognitive decline in older adultswith hypertension. Arch Intern Med 169: 1195–1202, 2009

50. Aronow WS: Treating hypertension in older adults. Drug Saf 32:111–118, 2009

51. Khan NA, Hemmelgarn B, Herman RJ, Bell CM, Mahon JL, LeiterLA, Rabkin SW, Hill MD: The 2009 Canadian Hypertension Edu-cation Program recommendations for the management of hyperten-sion: Part 2—Therapy. Can J Cardiol 25: 287–298, 2009

52. Aronow WS: Treatment of hypertension in the elderly. Geriatrics 63:21–25, 2008

53. Prospective Studies Collaboration: Age-specific relevance of usualblood pressure to vascular mortality: A meta-analysis of individualdata for one million adults in 61 prospective studies. Lancet 360:1903–1913, 2002

54. Gradman AH: Role of angiotensin II type 1 receptor antagonists inthe treatment of hypertension in patients aged ��65 years. DrugsAging 26: 751–767, 2009

55. Perry HM Jr, Davis BR, Price TR: Effect of treating isolated hyper-tension on the risk of developing various types and subtypes ofstroke: The Systolic Hypertension in the Elderly Program (SHEP).JAMA 284: 465–471, 2000

56. Dahlof B, Lindholm LH, Hansson L, Schersten B, Ekbom T, WesterPO: Morbidity and mortality in the Swedish Trial in Old Patients withHypertension (STOP-Hypertension). Lancet 338: 1281–1285, 1991

57. Staessen JA, Fagard R, Thijs L, Celis H, Arabidze GG, BirkenhagerWH, Bulpitt CK, de Leeuw PW, Dollery CT, Fletcher AE, Forette F,Leonetti G, Nachev C, O’Brien ET, Rosenfeld J, Rodicio JL, Tuom-ilehto J, Zanchetti A: Randomized double-blind comparison of pla-cebo and active treatment for older patients with isolated systolichypertension. The Systolic Hypertension in Europe (Syst-Eur) TrialInvestigators. Lancet 350: 757–764, 1997

58. Musini VM, Tejani AM, Bassett K, Wright JM: Pharmacotherapyfor hypertension in the elderly. Cochrane Database Syst Rev 7:CD000028, 2009

59. Staessen JA, Wang JG, Thijs L: Cardiovascular protection and bloodpressure reduction: A meta-analysis. Lancet 358: 1305–1315, 2001

60. Krokoff LR: Older patients need better guidelines for optimal treat-ment of high blood pressure. Hypertension 51: 817, 2008

61. Hertz RP, Unger AN, Cornell JA, Saunders E: Racial disparities inhypertension prevalence, awareness, and management. Arch InternMed 165: 2098–2104, 2005

62. Midlov P, Ekesbo R, Johansson L, Gerward S, Persson K, NerbrandC, Hedblad B: Barriers to adherence to hypertension guidelinesamong GPs in southern Sweden: A survey. Scand J Primary HealthCare 26: 154–159, 2008

63. Phillips LS, Branch WT, Cook CB, Doyle JP, El-Kebbi IM, GallinaDL: Clinical inertia. Ann Intern Med 160: 825–834, 2000

64. Williams H: The year in hypertension. J Am Coll Cardiol 55: 65–73,2009

65. Gueyffier F, Bulpitt C, Boissel JP: Antihypertensive drugs in very oldpeople: A subgroup meta-analysis of randomized controlled trials.Lancet 353: 793–796, 1999

66. Beckett NS, Peters R, Fletcher AE: Treatment of hypertension inpatients 80 years of age or older. N Engl J Med 358: 1887–1898, 2008

67. Szucs TD, Waeber B, Tormonaga Y: Cost-effectiveness of antihy-pertensive treatment in patients 80 years of age or older in Switzer-land: An analysis of the HYVET study from a Swiss perspective. JHum Hypertens 24: 117–123, 2010

68. Okonofua EC, Cutler NE, Lackland DT, Egan BM: Ethnic differ-ences in older Americans: Awareness, knowledge, and beliefs abouthypertension. Am J Hypertens 18: 972–979, 2005

69. Stanton JA, Lowenthal DT: The evidence for lifestyle modification inlowering blood pressure in the elderly. Am J Geriatr Cardiol 9:27–33, 2000

70. Whelton PK, Appel LJ, Espeland MA, Applegate WB, Ettinger WH,Kostis JB, Kumanyika S, Lacy CR, Johnson KC, Folmar S, Cutler

JA, TONE Collaborative Research Group: Sodium reduction andweight loss in the treatment of hypertension in older persons: Arandomized controlled trial of nonpharmacologic interventions in theelderly (TONE) JAMA 279: 839–846, 1998

71. Viera AJ, Kshirsagbar AV, Hinderliter AL: Lifestyle modificationadvice for lowering or controlling high blood pressure: Who’s gettingit? J Clin Hypertens 9: 850–858, 2007

72. Hajjar I, Miller K, Hirth V: Age-related bias in the management ofhypertension: A national survey of physicians’ opinions on hyper-tension in elderly adults. J Gerontol A Biol Sci Med Sci 57: M487–M491, 2002

73. Turnbull F, Neal B, Ninomiya T, Algert C, Arima H, Barzi F, BulpittC, Chalmers J, Fagard R, Gleason A, Heritier S, Li N, Perkovic V,Woodward M, MacMahon S: Effects of different regimens to lowerblood pressure on major cardiovascular events in older and youngeradults: Meta-analysis of randomized trials. BMJ 336: 1121–1123,2008

74. Rashidi A, Wright JT Jr: Drug treatment of hypertension in olderhypertensives. Clin Geriatr Med 25: 235–244, 2009

75. Staeesen JA, Gasowski J, Wang JG, Thijs L, Den Hond E, BoisselJ-P, Coope J, Ekborn T, Gueyffier F, Liu L, Kerlikowske K, PopockS, Fagard RH: Risks of untreated and treated isolated hypertension inthe elderly: Meta-analysis of outcome trials. Lancet 355: 865–872,2000

76. Pappoe LS, Winkelmayer WC: ACE inhibitor and angiotensin II type1 receptor antagonist therapies in elderly patients with diabetesmellitus: Are they underutilized? Drugs Aging 27: 87–94, 2010

77. Gradman AH: Role of angiotensin II type 1 receptor antagonists inthe treatment of hypertension in patients aged ��65 years. DrugsAging 26: 751–767, 2009

78. Thomas GN, Chan P, Tomlinson B: The role of angiotensin II type 1receptor antagonists in elderly patients with hypertension. DrugsAging 23: 131–155, 2006

79. Mukai Y, Shimokawa H, Higashi M: Inhibition of renin-angiotensinsystem ameliorates endothelial dysfunction associated with aging inrats. Aterioscler Thromb Vasc Biol 22: 1445–1450, 2002

80. The ALLHAT Officers and Coordinators for the ALLHAT Collab-orative Research Group: Major outcomes in high-risk hypertensivepatients randomized to angiotensin-converting enzyme inhibitor orcalcium channel blocker vs diuretic: The Antihypertensive and Lipid-Lowering Treatment to Prevent Heart Attack Trial (ALLHAT).JAMA 288: 2981–2997, 2002

81. Wing LM, Reid CM, Ryan Beilin LJ, Brown MA, Jennings GL,Johnston CI, McNeil JJ, Macdonald GJ, Marley JE, Morgan TO,West MJ: A comparison of outcomes with angiotensin-converting-enzyme inhibitors and diuretics for hypertension in the elderly.N Engl J Med 348: 583–592, 2003

82. Kjeldsen SE, Dahlof B, Devereux RB, Julius S, Aurup P, Edelman J,Beevers G, de Faire U, Fyhrquist F, Ibsen H, Kristianson K, Leder-balle-Pedersen O, Lindholm LH, Nieminen MS, Omvik P, Oparil S,Snapinn S, Wedel H, LIFE (Losartan Intervention for EndpointReduction) Study Group: Effects of losartan on cardiovascular mor-bidity and mortality in patients with isolated systolic hypertensionand left ventricular hypertrophy: A Losartan Intervention for End-point Reduction (LIFE) substudy. JAMA 288: 1491–1498, 2002

83. Julius S, Kjeldsen SE, Weber M: Outcomes in hypertensive patients at highcardiovascular risk treated with regimens based on valsartan or amlodipine:The VALUE randomized trial. Lancet 363: 2022–2031, 2004

84. Mendelson G, Aronow WS: Underutilization of beta-blockers inolder patients with prior myocardial infarction or coronary arterydisease in an academic, hospital-based geriatrics practice. J AmGeriatr Soc 45: 83–86, 1997

85. Krakoff LR: Older patients need better guidelines for optimal treat-ment of high blood pressure. Hypertension 51: 817, 2008

86. National Council on Patient Information and Education: The Other

34 Nephrology Self-Assessment Program - Vol 10, No 1, January 2011

Page 47: 2011.01 - Geriatric Nephrology

Drug Problem: Statistics on Medicine Use and Compliance, Be-thesda, National Council on Patient Information and Education, 1997

87. Simpson RJ Jr: Challenges for improving medication adherence.JAMA 296: 2614–2616, 2006

88. Trivedi AN, Moloo H, Mor V: Increased ambulatory care copay-ments and hospitalizations among the elderly. N Engl J Med 362:320–328, 2010

89. Butt TF, Branch RL, Beesley L, Martin U: Managing hypertension inthe very elderly: Effect of adverse drug reactions (ADRs) on achiev-ing targets. J Hum Hypertens 24: 514–518, 2010

90. Lee JK, Grace KA, Taylor AJ: Effect of a pharmacy care program onmedication adherence and persistence, blood pressure, and low-density lipoprotein cholesterol: A randomized controlled trial. JAMA296: 2563–2571, 2006

Diabetic Kidney Disease in the Elderly

EpidemiologySeveral studies that examined the prevalence

of chronic kidney disease (CKD) and ESRD strati-fied by age similarly concluded that both are nowdiseases of the elderly (1–3). Particularly in thosewho are older than 60 years, the most commoncause of CKD and ESRD in the United States isdiabetic kidney disease (4). One third of new ESRDcases in patients who are older than 75 years are dueto diabetic nephropathy. The number of people whohave diabetes and are older than 65 years in theUnited States, having doubled between 1900 and2000, is projected to double again by 2030 (5)(Figure 7), a growing epidemic that has been linkedto obesity, tobacco use, urbanization, physical inac-tivity, poor nutrition, improved survival of patientswith diabetes, and aging (6). According to preva-lence estimates from the Third National Health andNutrition Examination Survey (NHANES III), ap-proximately one third of older individuals withdiabetes have microalbuminuria, the earliest clinical

stage of diabetic kidney disease, although a nonspe-cific finding. A similar proportion has kidney im-pairment, with a risk twice that of the generalelderly population. Among 2570 older patients withdiabetes in the US Veterans’ Integrated ServiceNetwork, nearly half (48%) had CKD, most of thetime mild to moderate in severity (7). A cross-sectional report from Finland of 187 patients withdiabetes and 1073 individuals aged 64 to 100 yearsreported that 21.4% of patients with diabetes and12.7% of participants without diabetes had cystatinC levels above the age-adjusted limits (8). Diabetesis a more powerful determining factor for kidneyimpairment in the very old than is hypertension.

A recent study reported increased preva-lence of CKD in elderly participants withdiabetes in data sets from KEEP, NHANES,and Medicare.

According to the Centers for Disease Control andPrevention, at least 8% of the population in the UnitedStates has diabetes (9). The prevalence rises to 23% inthose who are older than 60 years (10). Between 1994and 2004, the prevalence of diabetes in individualswho were older than 65 in the United States increasedby 62% (11). A recent report documented that theprevalence of both diabetes and prediabetes havereached new levels (12), with total “crude prevalence”(diagnosed and undiagnosed cases) reported in as 30%for those who were older than 60 years. Another reportexamined trends in the rates of occurrences of diabetes

16

14

12

10

8

6

4

2

0

Perc

ent

0-19

0-1925

-2930

-3435

-3940

-4445

-4950

-5455

-5960

-6465

-7070

-7475

-79 80+

Age group (years)

FemalesMales

200180160140

1008060

200

Est

imat

ed n

umbe

rs o

f peo

ple

with

dia

bete

s (m

illio

ns)

120

40

20-44 45-64 65+Age group (years)

20002030

Figure 7. Global diabetes prevalence by age and gender in 2000 (left) and estimated number of adults with diabetes byage group worldwide in 2000 and 2030 (right). Copyright 2004 American Diabetes Association. Adapted from DiabetesCare, Vol. 27, 2004; 1047–1053 (reference 5). Reprinted with permission from The American Diabetes Association.

Nephrology Self-Assessment Program - Vol 10, No 1, January 2011 35

Page 48: 2011.01 - Geriatric Nephrology

and its complications in those who were older than 65years in the United States and had diabetes diagnosedin 1994, 1999, and 2003 (11). Control groups con-sisted of a similar sample size in each year but notdiagnosed with diabetes. The frequency of diagnosisof renal disease was substantially higher in the diabe-tes group, and within 1 year of diagnosis of diabetes,it rose progressively, doubling by 2003, although thetrend upward did not differ from the control groupswithout diabetes. A recent study (13) sought to deter-mine the prevalence of CKD and comorbid illness inelderly individuals determined from laboratory tests inthe Kidney Early Evaluation Program (KEEP; a freecommunity-based health screening program that tar-gets adults at high risk for kidney disease on the basisof personal or family history) and the NHANES (thecross-sectional probability samples of the civilian non-institutionalized US population), as well as the prev-alence of diagnosed CKD determined from billingcodes of a random 5% sample of the US Medicarepopulation. In all three data sets, the prevalence ofCKD was higher for participants with diabetes (KEEP48 versus 40%, NHANES 58 versus 41%, Medicare14 versus 4%). As expected, diabetes as a comorbidcondition was higher in those with than without CKD(KEEP 45 versus 37%, NHANES 21 versus 12%,Medicare 46 versus 20%).

A recent analysis from the Center for Diseasecontrol and Prevention determined that the decline indiabetes-related ESRD incidence included all agegroups, including those who were older than 75 years(Figure 8) (14). Using US Renal Data System data,Burrows et al. analyzed incident patients who hadESRD and had diabetes listed as their primary diag-nosis between 1990 and 2006. Incidence was calcu-lated using the estimated US population with diabetesfrom the National Health Survey, followed by ageadjustment. Whereas the number of those with diabe-tes-related ESRD treatment almost tripled between1990 and 2006, the age-adjusted diabetes-relatedESRD incidence decreased from 1996 to 2006, by3.9% per year. Among individuals aged 65 to 74 years,rates decreased by 3.4% (beginning in 1998), and forthose aged �75 years, by 2.1% (beginning in 1999).(Previously reported data had shown declining inci-dence only for those who were younger than 65 years.)The age-adjusted diabetes-related ESRD incidenceamong African American patients with diabetes was1.5 to 2.0 times greater than for Caucasian patients.

The authors cited widespread use of renin-angiotensin-aldosterone system blockers, estimated to be 70 to75% of those who were older than 65 years (3), for thedecreasing trends.

PathophysiologyPathology of the aging kidney is described else-

where in this issue of NephSAP. The kidney biopsy ofa healthy elderly individual may include pathologicfindings that have been considered a nonspecific partof the “normal” aging process (15). Common findingsin kidney biopsies of the elderly, variable in severity,include advanced vascular changes, fibrosis related tocollagen accumulation, thickened glomerular base-ment membrane, and global sclerosis (16). The kidneyduring aging sustains vascular changes associated withatherosclerosis, hyperplasia, and hypertension. Theglomerulus undergoes enlargement with mesangial ex-pansion, increases in mesangial and endothelial cellnumbers, and relative podocyte depletion. The contri-bution made by the aging process itself to age-associ-ated CKD carries the potential that it could be “accel-erated” by superimposed conditions such as diabetes(17), particularly considering the overlap in the pathol-ogy of diabetes and aging in the kidney. Conversely,diabetes accelerates aging. Thus, clinical signs of ag-ing pointed out in a recent report included increasedprevalence of cataracts; accelerated atherosclerosis;

Figure 8. Age-specific incidence of diabetes-related ESRDin the US population with diabetes, from 1990 to 2006. Œ,�45 years; f, 45 to 64 years; F, 65 to 74 yr; line, �75years. Copyright 2010 American Diabetes Association.From Diabetes Care, Vol. 33, 2010; 73–77 (reference 14).Reproduced by permission of The American Diabetes As-sociation.

36 Nephrology Self-Assessment Program - Vol 10, No 1, January 2011

Page 49: 2011.01 - Geriatric Nephrology

higher risk for myocardial infarction, stroke, and pe-ripheral vascular disease; increased cognitive decline;and increased functional decline (18). In older patientswith diabetes, histologic changes of diabetic nephrop-athy are compounded by advanced vascular changes ofaging. Smith reported in 1951 (19) that intercapillaryglomerulosclerosis (itself first described by Kimmel-stiel and Wilson in 1936) increases in frequency withage and could be seen in elderly individuals withoutdiabetes. How much do the renal phenotype of agingand diabetes have in common? Aging-related morpho-logic renal changes are similar to those detected inCKD as a result of diabetes (20). Cortical globalglomerulosclerosis, tubular atrophy, and interstitialfibrosis correlate with interlobular arterial sclerosis,which is accelerated in diabetes (20,21). Pathologi-cally, the aging kidney may be associated with mes-angial matrix expansion and basement membranethickening, nonspecific findings that are also recog-nized as key features of diabetic glomerulopathy. Al-though recent research has emphasized the role ofage-related changes as they coexist with other under-lying renal conditions, such as diabetic kidney disease,a report of 100 renal biopsies in patients who wereaged �80 years from multiple referral centers fornephrotic syndrome, nephritis, acute and subacute re-nal failure, chronic renal failure, and asymptomaticrenal abnormalities found diabetic nephropathy as ex-pected to be a rare finding. Findings associated withthe aging kidney itself were omitted in the report (22).

As an age-associated disease, diabetes has beentheorized to accelerate cell and organ senescence inhumans; in the kidney, these changes would hinder thealready limited ability of aged kidney tissue to repairitself. Verzola et al. (23) proposed that diabetic ne-phropathy is associated with an acceleration of asenescent phenotype in kidney cells, using assays forsenescence markers in kidney biopsies of patients withtype 2 diabetic nephropathy. Kidneys with diabeticnephropathy displayed an increased expression of se-nescence markers p16INK4A (a major cell-cycle regu-latory protein associated with replicative senescence)and SA-B-Gal (a marker of cellular senescence) com-pared with age-matched controls (Figure 9). Bothmarkers were markedly upregulated in tubular cellsand less so in glomerular podocytes, p16INK4A asso-ciated with glomerular ischemia and tubular atrophy,and SA-B-Gal associated with interstitial fibrosis. Asimilar senescence pattern was observed when tubular

cell cultures were incubated under high glucose con-ditions. The authors proposed that hyperglycemia maytrigger the loss of repair capabilities, promote the earlyoccurrence of senescence, and contribute to nephronloss in diabetic kidney disease.

It has been proposed that diabetes mayhasten cell and organ senescence. In re-cent studies, kidney biopsies with diabeticnephropathy displayed increased expres-sion of senescence markers, and geneticoverlap of loci in mice with age-associatedalbuminuria and in patients with diabetesand nephropathy was found.

Can the diabetic environment have effects sim-ilar to aging itself? A recent study by Tsaih et al.(24) suggested a common pathway for diabetes- andage-related renal disease by finding genetic overlapof loci associated with albuminuria in aging miceand with proteinuria in patients with diabetes. Ahaplotype-association mapping approach was usedto identify quantitative trait locus linkage for age-related proteinuria, a known feature of aging inmice, in an inbred strain. Haplotype blocks from themouse genome were tested for association with thehigh albumin-to-creatinine ratio phenotype. Onesignificant and eight suggestive loci were found.These loci were then compared with genome-wideassociation scans for diabetic nephropathy from apreviously reported genome-wide association study(25). Two of the nine mouse loci for age-associatedalbuminuria were significantly associated with dia-betic nephropathy. The findings were critically an-alyzed in an accompanying commentary (26) withregard to limitations related to genotyping data frominbred strains, modeling of the albumin-to-creati-nine ratio phenotype, and the potential for differentgenetic determinants in mice and humans.

Role of Advanced Glycation End ProductsThe changes of aging may also reflect accu-

mulation of advanced glycation end products(AGEs) in kidney tissues. Glycation is a slow,nonenzymatic reaction between free amino groupsin proteins and reducing sugars such as glucose thatleads to the formation of heterogeneous end prod-ucts. It is better known that AGEs accumulate at an

Nephrology Self-Assessment Program - Vol 10, No 1, January 2011 37

Page 50: 2011.01 - Geriatric Nephrology

accelerated rate during the course of diabetes (27)and have been implicated in diabetic complications.Increasing evidence suggests that AGEs accumulateduring normal aging and contribute to the agingphenotype (28,29). The expression for receptor forAGE (RAGE), an important transducer of AGEeffects, is increased in both aging and diabetes. Arecent study reported on glycation end products andtheir circulating receptors and the level of kidneyfunction in older community-dwelling women fromthe Women’s Health and Ageing Study 1 in Balti-more, MD (30). Approximately half had decreasedGFR at baseline. Of the remainder, 13.9% devel-oped decreased GFR 1 year later. In multivariatelogistic regression models, the serum AGE CMLand circulating RAGE levels were associated withimpaired GFR or prediction of impaired GFR. Gly-cation has therefore has been implicated in chrono-logical aging in tissues such as vascular tissues,skin, bone, cartilage, brain, and kidney. The signif-icance of AGE accumulation in aging is supported

by the effects of both an AGE-inhibitory drug andrestriction of dietary AGE sources to protect againstCKD in aging animals (31–33). Nonenzymatic gly-cation of collagen IV contributes to the thickeningof basement membranes and accumulation of extra-cellular matrix, hallmarks of diabetic kidney disease(34,35). AGEs also contribute to inflammation inaging. Aging humans are known to have increasedlevels of oxidative stress and inflammation. It wasrecently proposed by Vlassara et al. (36) that thedecline in kidney function with aging may be linkedto oxidative stress and inflammation. In aging mice,greater oxidant intake is associated with increasedage-related CKD (37). AGEs promote oxidation andinflammation through cell surface RAGE receptors.Patients with diabetes also have high serum AGElevels and increased oxidative stress and inflamma-tion in the diabetic kidney. AGEs are believed to beimportant contributors to inflammation in aging(31). The relation in elderly humans with diabetickidney disease remains to be determined. It was

Figure 9. Accelerated senescence phenotype in renal biopsies from patients with type 2 diabetes with nephropathy. (Top)p16INK4A expression. Left, nuclear expression in podocytes; right, staining in podocytes and mesangial-endothelial cells.(Bottom) SA-B-Gal staining. Left, tubular, right, tubular. Adapted with permission from reference 23 (Verzola D, GandolfoMT, Gaetani G, Ferraris A, Mangerini R, Ferrario F, Villaggio B, Gianiorio F, Tosetti F, Weiss U, Traverso P, Mji M,Deferrari G, Garibotto G: Accelerated senescence in the kidneys of patients with type 2 diabetic nephropathy. Am J PhysiolRenal Physiol 295: F1563–F1573, 2008).

38 Nephrology Self-Assessment Program - Vol 10, No 1, January 2011

Page 51: 2011.01 - Geriatric Nephrology

recently proposed that age-associated glomerulopa-thy may be understood as a senescence processaffecting kidney cells such as the podocyte andaccelerated by superimposed conditions such asdiabetes, such that age-related changes are aggra-vated by diabetes. In rats, the development of glo-merulosclerosis occurs in relation to aging itself. Arecent biopsy study of 1203 adult living kidneytransplant donors found that age-related renal fibro-sis may be collagen accumulation in the glomerularperitubular capillaries and tubulointerstitium (38).Donor nephrosclerosis, defined as a pattern of glo-merulosclerosis, tubular atrophy, interstitial fibro-sis, and arteriolosclerosis, occurred with older ageand was not attributable to CKD risk factors orkidney function (39).

Kidney FunctionModerate reductions in kidney function asso-

ciated with chronological aging have been demon-strated in both cross-sectional and longitudinal sur-veys such as NHANES and also in a series ofhealthy potential kidney donors (40). The decline isassociated with a proportionate fall in renal bloodflow, redistribution of flow from renal cortex tomedulla, and a slight increase in filtration fraction,which minimizes the GFR loss. Glomerular hyper-filtration is a well-characterized maladaptive featureof type 1 diabetes that is regarded as a potential riskfactor for nephropathy complications. Age-unad-justed definitions of hyperfiltration range from 125to 140 ml/min per 1.73 m2. In a survey of 662patients with type 2 diabetes, GFR was measured by99-tech-DTPA, and hyperfiltration was determinedusing an age-unadjusted threshold of �130 ml/minper 1.73 m2 and incorporating a decline of 1 ml/min/per 1.73 m2/y after age 40 years (41) (Figure 10). Theprevalence of hyperfiltration was 7.4% with age-unadjusted and 16.6% with age-adjusted definitions.In those who were older than 65 years, adjusting forage increased the prevalence of hyperfiltration from0.3 to 9.0%. The pathogenic significance in thoseaffected by hyperfiltration remains to be deter-mined.

TreatmentTo what extent should issues of CKD treat-

ment and outcomes be addressed differently in thegeriatric population with diabetes? Elderly patients

with diabetes and CKD have different needs ema-nating from their greater frailty, higher comorbidityindex, and shorter life expectancy and may warranta lower renoprotection treatment intensity than ayounger population (42). Published clinical guide-lines on the treatment of diabetic CKD fail todistinguish between age groups; European and USclinical guidelines for type 2 diabetes in the elderlydo not address the CKD population. As noted in areview by Abatteruso et al. (43), the European Diabetesand Ageing Guidelines (EDAC), the American Diabetesand Ageing Guidelines (ADAG), the Kidney DiseaseOutcomes Quality Initiative (KDOQI) ClinicalPractice Guidelines and Clinical Practice Recom-mendations for Diabetes and Chronic Kidney Dis-ease, and the Quality Indicators for the Care ofVulnerable Elders 3 (ACOVE-3) do not adequatelyaddress the subject of diabetic CKD in the elderly.Glycemic Control A publication on the treatmentof elderly patients with diabetes complicated byCKD (42) reviewed guidelines and data limitationson the topic. Regarding metabolic control, theKDOQI 2005 guidelines on diabetes and CKD em-phasized the benefit of strict metabolic control earlyin progression (i.e., prevention of microalbumin-uria), with evidence weak in later stages (44). Al-though improved glycemic control, a cornerstone ofdiabetes management in type 2 diabetes, may sig-

Figure 10. Age-related decline in kidney function in pa-tients with type 2 diabetes. Reprinted from reference 41with kind permission from Springer Science�Business Me-dia: Premaratne E, MacIsaac RJ, Tsalamandris C, Panagio-topoulos S, Smith T, Jerums G: Renal hyperfiltration in type2 diabetes: Effect of age-related decline in GFR. Diabeto-logia 48: 2486–2493, 2005, Figure 3.

Nephrology Self-Assessment Program - Vol 10, No 1, January 2011 39

Page 52: 2011.01 - Geriatric Nephrology

nificantly reduce the risk for microvascular compli-cations, the benefit on cardiovascular outcomes re-mains uncertain. No studies have prospectivelyevaluated the impact of intense control in patientswho are older than 65 year. The United KingdomProspective Diabetes Study (UKPDS) is usuallycited as the major large randomized, controlledstudy of patients with type 2 diabetes (age 53.4 �8.0 years) to confirm the benefit of glycemic controlin reducing microvascular complications (45). Gly-cemic control may take as long as 8 years to have apositively impact on microvascular complications(42).

The risks of tight glycemic control haveemerged in several recent studies of the generaldiabetes population. Three large trials—Action toControl Cardiovascular Risk in Diabetes (AC-CORD; mean age 62 years) (46), Action in Diabetesand Vascular Disease: Preterax and DiamicronModified Release Controlled Evaluation (AD-VANCE; mean age 66 years) (47), and VeteransAffairs Diabetes Trial (VADT; mean age 60 years)(48)— collectively evaluated nearly 23,000 individ-uals with type 2 diabetes. Renal benefit variedamong the studies, whereas cardiovascular effectslacked benefit in all three trials. The renal benefit(ADVANCE) involved a 21% reduction in the in-cidence of kidney disease (albuminuria) in the in-tensive treatment group. However, hypoglycemiaaffected a significant proportion of patients in theintensive treatment groups of all three studies. TheACCORD study compared a strategy of intensivecontrol (hemoglobin A1c target �6.0%) or standardcontrol (hemoglobin target 7.0 to 7.9%). The target-ing of normal glycemic levels for 3.5 years led toincreased mortality and did not significantly reducemajor cardiovascular events or renal outcomes. Inthe ADVANCE study, severe hypoglycemia oc-curred occasionally but more commonly in the in-tensive control group. Intensification of therapy inolder patients should be approached cautiously.Guidelines regarding older patients with diabetesrecommend individualized care on the basis of co-morbid conditions, projected life expectancy, healthcare goals, and treatment preferences (49). Thepresence of advanced CKD will further complicatetreatment and require prudence because of increas-ing comorbidities, limited life expectancy, and ef-fects of kidney impairment on drug metabolism.

Oral hypoglycemic agents to be avoided includechlorpropamide and glyburide (severe hypoglyce-mia) and metformin (fatal lactic acidosis) (50).

In a post hoc analysis of RENAAL results,the effectiveness of ARB therapy extendedto the oldest tertile (>65 years), in whomdoubling of serum creatinine was reducedby 38% and ESRD by 50% with losartan,with no increase in important adverse ef-fects.

Renin-Angiotensin-Aldosterone System Block-ade A recent review firmly established that currentguidelines for the use of angiotensin-converting en-zyme inhibitor (ACEI)/angiotensin receptor blocker(ARB) antagonists in CKD are based on limited rele-vance to the elderly patient with CKD (51). Frequentlycited research studies on the prevention/treatment ofdiabetic complications such as nephropathy of CKDhave also generally excluded the elderly. However, apost hoc analysis using primary data from the Reduc-tion of Endpoints in NIDDM with the Angiotensin IIAntagonist Losartan (RENAAL) study (52,53) evaluatedthe efficacy and safety of the ARB (50 to 100 mg/d) inelderly patients with diabetes by stratifying the fullstudy cohort of 1513 patients into age groups at timeof enrollment: �57 (33.4% of enrollees; maximumage 74 years), �57 to 65 (38.8%), and �65 years(27.8%). The oldest tertile was composed predomi-nantly of men (64.8%) and individuals of Caucasianrace (57.5%). The authors tested for effect modifica-tion by age of the impact of losartan on the incidenceof the predefined end points doubling of serum creat-inine, ESRD, or death; the incidence of adverse eventswas also analyzed. In the entire cohort, losartan sig-nificantly lowered the risk for the primary end point by16%, doubling of creatinine by 25%, and progressionto ESRD by 28%. Remarkably, the effectiveness ofARB therapy on the primary composite outcome or itsindividual components in the trial did not differ byage. Furthermore, in the oldest tertile, the rate ofdoubling of baseline serum creatinine was reduced by38% with losartan and the event rate of ESRD by 50%.Age did not increase the risk for important adverseeffects from losartan, including a rise in serum creat-inine or hyperkalemia. Although not specifically con-

40 Nephrology Self-Assessment Program - Vol 10, No 1, January 2011

Page 53: 2011.01 - Geriatric Nephrology

ducted in an elderly population, the study provides thebest clinical trial evidence available in support of ARBblockade of the renin-angiotensin system in olderpatients. Little is known about the time horizon fortreatment responses in this population with reducedlife expectancy. Older age was one of the strongpredictors for cardiovascular events in RENAAL (53).

Recent studies have indicated that elderly pa-tients were not sufficiently being prescribed ACEI orARBs (54,55). Winkelmayer et al. reviewed Medicaredata for 2002 on patients residing in Pennsylvaniawho had diabetes (54). Of 30,750 patients identified,21,053 had hypertension and 1243 were identified ashaving proteinuria or proteinuria and kidney disease.Most patients were 75 to 84 years old. Of the patientswith hypertension only, 50.5% were on an ACEI orARB; of the patients with proteinuria, 40%; and forthose with both hypertension and proteinuria, 54.7%.In each diagnostic category, roughly 25% fewer wereon ACEI/ARB therapy than in a separate report re-garding a younger cohort. The authors speculated thatsafety concerns about hyperkalemia and reduction inkidney function either acutely or during progression toESRD and lack of efficacy data in the elderly underliethe prescription pattern. However, among patientswith diabetic CKD in the Medicare Beneficiary Sur-vey, more than two thirds reported using ACEIs/ARBs, well above the patients without diabetes report-ing (3). A recent study of a large community-basedcohort in Canada also provided data on ACEI/ARBuse (56). The study evaluated the impact of estimatedGFR reporting with nephrology visits and health careresource use. After the implementation of estimatedGFR reporting, the rate of first outpatient nephrologyvisits for CKD increased by 68.4% (56), and referralrates were even higher for elderly patients with diabe-tes. Of note, reporting of estimated GFR was notassociated with an increase in ACEI/ARB use, perhapsbecause the majority (77.5%) of the elderly withdiabetes were already under treatment. Preventing car-diovascular morbidity and mortality may be a moreimportant factor than delaying progression to ESRD inuse of ACEIs/ARBs in the elderly. Guidelines suggestthat when reducing cardiovascular risk is the priority,ACEIs should be considered first-line therapy andARBs the first alternative (43). However, CKD in theelderly population with diabetes may frequently lackproteinuria and therefore be less responsive to renin-

angiotensin system blockade. In NHANES III, onethird of those who had type 2 diabetes, were aged 60to 70 years, and had normal urinary albumin excretionnonetheless had a GFR of �30 ml/min, and almosthalf were between 30 and 60 ml/min (57).

For these and other treatment options of diabeticCKD not covered here (including renal replacementtherapy), it is reasonable to assume that treatmentapproaches considered for younger patients may notapply to the elderly. These include drugs alreadyapproved for hypertension (aldosterone blockers, renininhibitors) or for other indications (vitamin D ana-logues, thiazolidinediones, statins). The risk/benefitratios of therapies for diabetic CKD as applied to theelderly population remain largely undetermined. Foremerging therapies, inclusion of elderly patients inregulatory trials while taking into account the GFRdecline as a result of aging, the prevalence of nondi-abetic kidney disease, safety, and other factors may beadvisable.

References1. Hallan SI, Coresh J, Astor BC: International comparison of the

relationship of chronic kidney disease prevalence and ESRD risk.J Am Soc Nephrol 16: 2275–2284, 2006

2. Coresh J, Astor BC, Gren T, Eknoyan G, Levey AS: Prevalence ofchronic kidney disease and decreased kidney function in the adult USpopulation: Third National Health and Nutrition Examination Survey.Am J Kidney Dis 41: 1–12, 2003

3. US Renal Data System: 2009 Annual Data Report: Atlas of ChronicKidney Disease and End-Stage Renal Disease in the United States,Bethesda, National Institutes of Health, National Institute of Diabetesand Digestive and Kidney Diseases

4. Rosner M, Abdel-Rahman E, Williams ME: Geriatric nephrology: Anew ASN priority. Clin J Am Soc Nephrol 5: 936–942, 2010

5. Wild S, Roglic G, Gren A, Sicree R, King H: Global prevalence ofdiabetes: Estimates for the year 2000 and projections for 2030.Diabetes Care 27: 1047–1053, 2004

6. Zimmet P, Alberti KG, Shaw J: Global and societal implications ofthe diabetes epidemic. Nature 414: 782–787, 2001

7. Patel UD, Young EW, Ojo AO, Hayward RA: CKD progression andmortality among older patients with diabetes. Am J Kidney Dis 46:406–414, 2005

8. Wasen E, Isoaho R, Mattila K, Vahlberg T, Kivela S-L, Irjala K:Renal impairment associated with diabetes in the elderly. DiabetesCare 27: 2648–2653, 2004

9. Centers for Disease Control and Prevention: Diabetes data & trends.Available at: www.cdc.gov/diabetes/statistics/esrd. Accessed June25, 2010

10. Williams ME: Diabetic kidney disease in the older adult: an update.Ageing Health 5: 625–633, 2009

11. Sloan FA, Bethel A, Ruiz D Jr, Shea AH, Feinglos MN: The growingburden of diabetes mellitus in the U.S. elderly population. ArchIntern Med 168: 192–199, 2008

12. Covie CC, Rust KF, Ford ES, Eberhardt MD, Byrd-Holt DD, Li C,Williams DE, Gregg EW, Bainbridge KE, Saydah SH, Geiss LS: Fullaccounting of diabetes and pre-diabetes in the U.S. population in1988–1994 and 2005–2006. Diabetes Care 32: 287–294, 2009

Nephrology Self-Assessment Program - Vol 10, No 1, January 2011 41

Page 54: 2011.01 - Geriatric Nephrology

13. Stevens LA, Li S, Wang C, Huang C, Becker BN, Bomback AS,Brown WW, Burows NR, Jurkoviz CT, McFarlane SI, Norris KC,Shlipak M, Whaley-Connell AT, Chen S-C, Bakris GL, McCulloughPA: Prevalence of CKD and comorbid illness in elderly patients inthe United States: Results from the Kidney Early Evaluation Program(KEEP). Am J Kidney Dis 55[Suppl 2]: S23–S33, 2010

14. Burrows NR, Li Y, Geiss LS: Incidence of treatment for end-stagerenal disease among individuals with diabetes in the U.S. continues todecline. Diabetes Care 33: 73–77, 2010

15. Jefferson JA, Alpers CE: Should renal biopsies be performed in thevery elderly? Nat Rev Nephrol 5: 561–562, 2009

16. Martin JE, Sheaff MT: Renal ageing. J Pathol 21: 198–205, 200717. Wiggins RC: The spectrum of podocytopathies: A unifying view of

glomerular diseases. Kidney Int 71: 1205–1214, 200718. Morley JE: Diabetes and ageing: Epidemiologic overview. Clin

Geriatr Med 24: 395–405, 200819. Smith HW: The Kidney: Structure and Function in Health and

Disease, New York, Oxford University Press USA, 1951, p 64720. Rodriguez-Puyol D: The ageing kidney. Kidney Int 54: 2247–2265,

199821. Zhou XJ, Rakheja D, Yu X, Saxena R, Vaziri ND, Silva FG: The

ageing kidney. Kidney Int 74: 710–720, 200822. Nair R, Bell JM, Walker PD: Renal biopsy in patients 80 years and

older. Am J Kidney Dis 44: 618–626, 200423. Verzola D, Gandolfo MT, Gaetani G, Ferraris A, Mangerini R,

Ferrario F, Villaggio B, Gianiorio F, Tosetti F, Weiss U, Traverso P,Mji M, Deferrari G, Garibotto G: Accelerated senescence in thekidneys of patients with type 2 diabetic nephropathy. Am J PhysiolRenal Physiol 295: F1563–F1573, 2008

24. Tsaih S-W, Pezzolesi MG, Yuan R, Warram JH, Krolewski AS,Korstanje R: Genetic analysis of albuminuria in ageing mice andconcordance with loci for human diabetic nephropathy found in agenome-wide association scan. Kidney Int 77: 201–210, 2010

25. Pezzolesi ZM, Poznik GD, Mychaleckyj JC: Genome-wide associa-tion scan for diabetic nephropathy susceptibility genes in type 1diabetes. Diabetes 58: 1403–1410, 2009

26. Sterken R, Kiryluk K: the genetics of albuminuria: From haplotypeassociation mapping in mice to genetic causation in humans. KidneyInt 77: 173–175, 2010

27. Kasper M, Funk RH: Age-related changes in cells and tissues due toadvanced glycation end products (AGEs). Arch Gerontol Geriatr 32:233–243, 2001

28. Semba RD, Nicklett EJ, Ferrucci L: Does accumulation of advancedglycation end products contribute to the ageing phenotype? J Ger-ontol A Biol Sci Med Sci 65: 963–975, 2010

29. Abdelhafiz AH, Brown SH, Bello A, El Nahas M: Chronic kidneydisease in older people: Physiology, pathology or both? Nephron ClinPract 116: 19–24, 2010

30. Semba RD, Ferrucci L, Fink JC, Sun K, Beck J, Dalal M, GuralnikJM, Fried LP: Advanced glycation end products and their circulatingreceptors and level of kidney function in older community-dwellingwomen. Am J Kidney Dis 53: 51–58, 2009

31. Mei C, Zheng F: Chronic inflammation potentiates kidney ageing.Semin Nephrol 29: 555–568, 2009

32. Cai W, He JC, Zhu L, Chen X, Zheng F, Striker GE: Oral glycotoxinsdetermine the effects of calorie restriction on oxidant stress, age-related diseases, and lifespan. Am J Pathol 173: 327–336, 2008

33. Li YM, Steffes M, Donnelly T, Liu C, Fuh H, Basgen J: Preventionof cardiovascular and renal pathology of ageing by the advancedglycation inhibitor aminoguanidine. Proc Natl Acad Sci U S A 93:3902–3907, 1996

34. Mott JD, Khalifah RG, Nagase H, Shield CF, Hudson JK, HudsonBG: Nonenzymatic glycation of type IV collagen and matrix metal-loproteinase susceptibility. Kidney Int 52: 1302–1312, 2007

35. Brownlee M, Cerami A, Vlassara H: Advanced glycosylation end

products in tissue and the biochemical basis of diabetic complica-tions. N Engl J Med 348: 1315–1322, 1988

36. Vlassara H, Uribarri J, Ferrucci L, Cai W, Torreggiani M, Post JB,Zheng F, Striker GE: Identifying advanced glycation end products asa major source of oxidants in ageing: Implications for the manage-ment and/or prevention of reduced renal function in elderly persons.Semin Nephrol 29: 594–603, 2009

37. Vlassara H, Torreggiani M, Post JB, Zheng F, Uribarri J, Striker GE:Role of oxidants/inflammation in declining renal function in chronickidney disease and normal ageing. Kidney Int 76[Suppl 114]: S3–S11, 2009

38. Anderson S, Halte JB, Hazzard WR, Himmelfarb J, Horne FM,Kaysen GA, Kusek JW, Nayfield SG, Schmader K, Tian Y, Ash-worth JR, Clayton CP, Parker RP, Tarver ED, Woolard NF, High KP:Prediction, progression, and outcomes of chronic kidney disease inolder adults. J Am Soc Nephrol 20: 1199–1209, 2009

39. Rule AD, Amer H, Cornell LD, Taler SJ, Cosio FG, Kremers WK,Textor SC, Stegall MD: The association between age and nephro-sclerosis on renal biopsy among healthy adults. Ann Inern Med 152:561–567, 2010

40. Ibrahim HN, Foley R, Tan L, Rogers T, Bailey RF, Guo H, Gross CR,Matas AJ: Long-term consequences of kidney donation. N EnglJ Med 360: 459–469, 2009

41. Premaratne E, MacIsaac RJ, Tsalamandris C, Panagiotopoulos S,Smith T, Jerums G: Renal hyperfiltration in type 2 diabetes: Effect ofage-related decline in glomerular filtration rate. Diabetologia 48:2486–2493, 2005

42. Brown AF, Mangione CM, Saliba D, Sarkisian CA: Guidelines forimproving the care of the older person with diabetes mellitus. J AmGeriatr Soc 51: S265–S280, 2003

43. Abaterusso C, Lupo A, Ortalda V, DeBiase V, Pani A, Muggeo M,Gambaro G: Treating elderly people with diabetes and stages 3–4chronic kidney disease. Clin J Am Soc Nephrol 3: 1185–1194, 2008

44. KDOQI clinical practice guidelines and clinical practice recommen-dations for diabetes and chronic kidney disease. Am J Kidney Dis49[Suppl 2]: S12–S154 2007

45. Tkac I: Effect of intensive glycemic control on cardiovascular out-comes and all-cause mortality in type 2 diabetes: Overview andmetaanalysis of five trials. Diabetes Res Clin Pract 86[Suppl 1]:S57–S62, 2009

46. The Action to Control Cardiovascular Risk in Diabetes Study Group:Effects of intensive glucose lowering in type 2 diabetes. N EnglJ Med 358: 2545–2559, 2008

47. The ADVANCE Collaborative Group: Intensive blood glucose andvascular outcomes in patients with type 2 diabetes. N Engl J Med358: 2560–2572, 2008

48. Duckworth W, Abraira C, Moritz T, Reda D, Emanuele N, ReavenPD, Zieve FJ, Marks J, Davis SN, Hayward R, Warren SR, GoldmanS, McCarren M, Vitek ME, Henderson WG, Huang GD, VADTInvestigators: Glucose control and vascular complications in veteranswith type 2 diabetes. N Engl J Med 360: 129–139, 2009

49. Abrahamson M: A 74-year-old woman with diabetes: Clinical cross-roads. JAMA 297: 196–204, 2007

50. Abbatecola AM, Paolisso G, Corsonello A, Bustacchini S, LattanzioF: Antidiabetic oral treatment in older people: Does frailty matter?Drugs Aging 26[Suppl 1]: 53–62, 2009

51. O’Hare AM, Kaufman JS, Covinsky KE, Landefeld CS, McFarlandLV, Larson EB: Current guidelines for using angiotensin-convertingenzyme inhibitors and angiotensin II-receptor antagonists in chronickidney disease: Is the evidence base relevant to older adults? AnnIntern Med 150: 717–724, 2009

52. Winkelmayer WC, Zhang Z, Shahinfar S, Cooper ME, Avorn J,Brenner BM: Efficacy and safety of angiotensin II receptor blockadein elderly patients with diabetes. Diabetes Care 29: 2210–2217, 2006

53. Brenner BM, Cooper ME, deZeeuw D, Keane WF, Mitch WE,Parving HH, Remuzzi G, Snapinn SM, Zhang Z, Shahinfar S: Effects

42 Nephrology Self-Assessment Program - Vol 10, No 1, January 2011

Page 55: 2011.01 - Geriatric Nephrology

of losartan on renal and cardiovascular outcomes in patients with type2 diabetes and nephropathy. N Engl J Med 345: 861–878, 2001

54. Winkelmayer WC, Fischer MA, Schneeweiss S, Wang PS, Levin R,Avorn J: Underuse of ACE inhibitors and angiotensin II receptorblockers in elderly patients with diabetes. Am J Kidney Dis 46:1080–1087, 2005

55. Rosen AB, Karter AJ, Liu JY, Selby JV, Schneider EC: Use ofangiotensin-converting enzyme inhibitors and angiotensin receptorblockers in high-risk clinical and ethnic groups with diabetes. J GenIntern Med 19: 669–675, 2004

56. Hemmelgarn BR, Zhang J, Manns B: Nephrology visits and healthcare resource use before and after reporting estimated glomerularfiltration rate. JAMA 303: 1151–1158, 2010

57. Kramer HJ, Nguyen QD, Curhan G, Hsu CY: Renal insufficiency inthe absence of albuminuria and retinopathy among adults with type 2diabetes mellitus. JAMA 289: 3273–3277, 2003

Glomerular Diseases

Epidemiology of Glomerular Disease in theElderly

Glomerular disorders, both primary and second-ary to an underlying systemic disease, are commoncauses of kidney disease in the elderly (1–4). Thepatterns of glomerular pathology differ in the elderlycompared with younger individuals. In large epidemi-ologic studies, the prevalence of secondary forms ofglomerular disease increase whereas primary formsdecrease with age (5–7). Table 2 summarizes the mostcommon lesion encountered in elderly compared withindividuals of less advanced age.

Among elderly individuals who present with thenephrotic syndrome (NS), the most common lesionsare membranous nephropathy (MN), FSGS, minimalchange disease (MCD), and amyloidosis (AM) (1–7).Diabetic nephropathy (DN) as a cause of kidney dis-ease is usually underrepresented in epidemiologicstudies that are based on renal biopsy pathology be-cause of the infrequency with which renal biopsy isperformed in patients with overt diabetes, microvas-cular disease (retinopathy), and NS (ascertainmentbias). In a recent survey of 235 renal biopsies per-formed in patients who were aged 80 to 99 years,

Moutzouris et al. (8) made some interesting observa-tions. Among those who presented with NS, 22% hadMN, 18% had AM, 16% had MCD, and 6% had IgAnephropathy (IgAN). FSGS and other lesion were lesscommon. Patients who presented with acute kidneyinjury or rapidly progressive renal failure were foundto have “pauci-immune” crescentic glomerulonephritis(PICGN) in 33%, IgAN in 7%, postinfectious glomer-ulonephritis (GN) in 3%, anti–glomerular basementmembrane (anti-GBM) nephritis in 3%, AM in 2%,and FSGS in 2%. Other, primarily tubulointerstitialdiseases or vascular disease accounted for the remain-der. Among those with a presentation with chronic(slowly) progressive kidney disease, FSGS and ne-phrosclerosis accounted for 44% and PICGN for 10%of cases. Interestingly, patients who presented with acombination of NS and acute kidney injury predomi-nantly had MCD or PICGN (35% of cases total),whereas other lesions were seen less commonly Themost frequent glomerular lesion encountered in thisseries was PICGN (18%), and MN was less commonthan observed in a younger group. Not surprising,lupus nephritis (LN) is infrequently encountered in theelderly, accounting for �1% in the series of Mout-zouris et al. Thus, the pattern of renal pathologydepended on the mode of presentation in the veryelderly cohort, but PICGN, MN, MCD, and AM werecommonly encountered. Renal biopsy in the elderlycan contribute importantly to therapeutic decisionmaking, particularly in NS and in rapidly progressiveGN and is reasonably safe in experienced hands(9,10).

DN, secondary to type 2 diabetes, is a verycommon cause of glomerular disease in the elderly butwill not be dealt with in this section. The reader isreferred to a detailed analysis of recent finding pub-lished in a previous issue of NephSAP (11).

MCD in the ElderlyThe diagnosis of MCD by renal biopsy can be

difficult in the elderly because of concomitantchanges of senile nephrosclerosis (global glomeru-losclerosis, arteriolonephrosclerosis, tubular atro-phy, and interstitial fibrosis). Electron microscopicexamination may be crucial to demonstrate the typ-ical diffuse and generalized foot process effacementso typical (but not pathognomic) of MCD. However,remember that similar foot process effacement isseen in primary FSGS.

Table 2. Common glomerular diseases in the elderly

Crescentic glomerulonephritis (pauci-immune necrotizingand crescentic glomerulonephritis, usually ANCApositive)

Membranous nephropathy (idiopathic and secondary todrugs and neoplasia)

Monoclonal Ig deposition disease (primary �AL and AHamyloidosis and non-amyloid Ig deposition diseases)

Minimal change disease/FSGSPostinfectious glomerulonephritis (diabetic nephropathy)

Nephrology Self-Assessment Program - Vol 10, No 1, January 2011 43

Page 56: 2011.01 - Geriatric Nephrology

As pointed out already, MCD is a relativelycommon lesion encountered in elderly with a presen-tation of NS. Unlike children, the elderly with primaryMCD are very prone to developing acute kidney injury(AKI) concomitantly with the onset of NS. As many as40% of elderly patients with MCD and NS will de-velop AKI (12). Typically, such patients have severeproteinuria (�10 g/d) and profound hypoalbumine-mia. Occasionally, a similar syndrome of NS�AKIcan develop secondarily to the use of nonsteroidalanti-inflammatory agents. The course of AKI super-imposed on MCD may be progressive and irreversible,but the majority of cases will remit with appropriatetreatment.

The treatment of MCD in the elderly is the sameas for younger adults, although the time from initiationof treatment to first response may be prolonged (�6months) (12,13) and treatment periods much longerthan in children are required to achieve completeremissions. Among adults, the efficacy of steroid treat-ment of MCD with NS is not much different betweenyounger adults and the elderly (13). Oral prednisone,usually given in doses of 2 mg/kg every other day asa single dose between 8 and 9 am, is often usedinitially, but no randomized, controlled trials of thisapproach have ever been conducted in the elderly.After approximately 2 to 3 months, the dosage isgradually reduced to approximately 1 mg/kg everyother day or less if adverse effects limit dosage.Remissions, when they occur, are usually complete(�300 mg/d protein excretion). Partial remission orlack of response often indicates an underlying FSGSlesion, missed in the initial renal biopsy. Althoughthe data are limited, one can expect remission inapproximately 50% of patients by 2 to 4 months oftreatment and in approximately 75% of patients by6 months of treatment. Relapses of NS do occur, butthey are less frequent in the elderly (13). Steroid-related adverse effects are common with prolongedtherapy. Elderly patients with concomitant type 2diabetes, severe osteoporosis, immunodeficiency, orprevious gastrointestinal bleeding may be poor can-didates for steroid therapy because a narrow thera-peutic index.

Such patients may be candidates for alternativeregimens, although the efficacy and safety of theseapproaches have not been well tested in the elderly(14,15). Oral calcineurin inhibitors (CNIs; cyclospor-ine or tacrolimus) combined with low-dosage alter-

nate-day steroid may be preferred, although theywould be contraindicated in patients with AKI�NS.Remissions can be produced by CNIs in a high per-centage of patients (�70%), but relapses are verycommon when the agents are stopped. Prolonged treat-ment with CNIs may be associated with nephrotoxic-ity, particularly when trough blood levels are main-tained too high. The usual starting dosages forcyclosporine are 3.5 mg/kg per d (given in two divideddoses) and 0.05 mg/kg per d for tacrolimus. Bloodlevels should be monitored for toxicity. Cyclophosph-amide is sometimes recommended for multiple relaps-ing disease; however, this agent is not well tolerated inelderly patients and can be associated with activationof latent herpes viral infections (herpes zoster) or otheropportunistic infections. Azathioprine is ineffective,and only limited data on mycophenolate mofetil areavailable in the elderly. Rituximab may be effectivefor induction of remissions in younger patients withMCD, but no data are available on its efficacy orsafety in the elderly (16).

MN in the ElderlyThe lesion of MN is quite commonly encoun-

tered in the elderly, particularly in association with theNS, but less commonly in the very elderly (�80 years)(5–8,17). One of the main issues when a lesion of MNis found in an elderly patient is whether it is a second-ary or a primary (idiopathic) lesion. The most commonsecondary form of MN in the elderly is that associatedwith underlying malignancy (18). Most of the cancersassociated with MN are of stomach, colon, lung, orbreast origin, but other malignancies may also beinvolved, including hematopoietic neoplasia such aslymphocytic leukemia or multiple myeloma. Lefau-cher et al. (18) studied 240 patients with MN (24associated with a malignancy). Except for a smokinghistory, the clinical presentation was not different forthose with or without neoplasia. The standardizedincidence ratio of cancer was much higher in patientswith MN than the general population (9.8 for men and12.3 for women), especially with advancing age. Thenumber of infiltrating inflammatory cells in glomeruliwas a useful pathologic feature distinguishing cancer-from non–cancer associated MN. In addition, thepattern of IgG subclass deposition may be helpful insuggesting a possible secondary form of MN. In theidiopathic lesion, IgG4 predominates, whereas IgG1and IgG3 are primarily found in malignancy associ-

44 Nephrology Self-Assessment Program - Vol 10, No 1, January 2011

Page 57: 2011.01 - Geriatric Nephrology

ated MN (19,20). Other features suggesting a second-ary MN lesion (possibly from cancer) are isolatedmesangial electron-dense deposits in addition to thetypical subepithelial electron-dense deposits and neg-ative tests for antibodies to the phospholipase 2 recep-tor, which are characteristic of the idiopathic form ofMN (IMN) (19). The diagnosis of cancer can besignificantly delayed after the diagnosis of MN. In theexperience of Bjorneklett et al. (21), the median timefrom diagnosis of MN to the diagnosis of cancer was60 months. Roughly one third of patients have adiagnosis of cancer made before or at the same time asthe diagnosis of MN. Thus, it is always appropriate to“screen” for underlying malignancy when MN is di-agnosed in the elderly patient. Such screening usuallyconsists of a chest x-ray (or a high-resolution com-puted tomography scan in a smoker), stool for occultblood, hemogram for hemoglobin and differential leu-kocyte count, prostate-specific antigen (in men), andmammography (in women). A colonoscopy or flexiblesigmoidoscopy is also recommended if this has notbeen done as a part of routine surveillance in theprevious year.

No randomized, controlled trials of treatment ofMN have been carried out exclusively in the elderly,so its treatment in the elderly has been based on trialsin somewhat younger patients (average age 50 to 55years) (17). However, the use of alkylating agents(cyclophosphamide or chlorambucil) can be associatedwith activation of latent herpes virus (herpes zoster).Because of poor bone marrow tolerance, the dosage ofthese agents may have to be reduced to avoid seriousleucopenia Alternatives to alkylating agents includeCNI or possibly rituximab, mycophenolate mofetil, orsynthetic ACTH, but there is little experience with andno randomized, controlled trials of use of the latteragents in the elderly population with IMN (17,22,23).Steroid monotherapy and azathioprine are equally in-effective and should not be used for treatment of IMN.The overall efficacy of treatment may be somewhatdiminished in the elderly, although this is far fromcertain. Spontaneous remission occurs in approxi-mately 30% of patients with IMN, so a period ofconservative therapy (�6 months) with observation isusually indicated, unless the NS is severe or disablingor if renal function is steadily deteriorating (17).Slowly diminishing proteinuria is often a sign of animpending remission. Inhibitors of the renin-angioten-sin system may show an antiproteinuric effect but

usually only when the urine protein excretion is �10g/d (24).

Crescentic GN and Small Vessel Vasculitis inthe Elderly

Among elderly patients who present with a pro-gressive loss of kidney function and urinary findingssuggestive of GN (dysmorphic hematuria and protein-uria), a lesion of crescentic GN, with or withoutfeatures of systemic vasculitis involving small to me-dium-sized vessel, is quite common (4–7,25,26).

Crescentic GN is a common cause of a“nephritic” urine sediment and progres-sive renal dysfunction in the elderly pa-tient. It should always be considered in thedifferential diagnosis of this clinical pre-sentation.

The “subset” of crescentic GN most frequentlyobserved in the elderly is a “pauci-immune” necrotiz-ing and crescentic GN associated with circulatinganti-neutrophil cytoplasmic autoantibodies (ANCA-associated crescentic GN) (4,25,26). Some of thesepatients will have a systemic form of vasculitis, eitherWegener’s granulomatosis or microscopic polyangi-itis, whereas others will have “renal-limited” disease.Much less common, elderly patients may have anti-GBM antibody–induced disease or even a lesion sec-ondary to systemic lupus erythematosus (SLE; see theLN in the Elderly section), a drug hypersensitivity(e.g., allopurinol, rifamipicin, hydralazine), a malig-nancy, or an infection. Pulmonary alveolar hemor-rhage (Goodpasture syndrome) is also seen in associ-ation with both ANCA-associated GN and anti-GBMdisease. However, in the elderly, ANCA-associateddisease is the most frequent disease found with pul-monary hemorrhage. Anti-GBM and ANCA-associ-ated disease can occur simultaneously or sequentiallyin the elderly as well as in younger individuals (27,28).

The treatment of crescentic GN and vasculitishas undergone dramatic changes in the past decade(29). Cyclophosphamide (oral or intravenous) plushigh-dose glucocorticoids are now the treatment ofchoice for most patients (29). The addition of intensiveplasma exchange for those with severe acute renalfailure has been shown to be effective, at least in theshort term. Whether intensive plasma exchange is still

Nephrology Self-Assessment Program - Vol 10, No 1, January 2011 45

Page 58: 2011.01 - Geriatric Nephrology

life saving over the long term is still unknown oruncertain (30,31). The issue in treating the elderlypatient with crescentic GN or systemic vasculitis is notefficacy but safety. Treatment regimens are associatedwith a high degree of immunosuppression and theemergence of complicating infections (especially op-portunistic ones) is not uncommon, often leading tofatal outcome, particularly during the first 6 to 12months of treatment with combined cyclophospha-mide and glucocorticoids (32–34). Using a “combinedburden of events” (CBOE) scoring system, Little et al.(32) found that both advanced age and initial low GFRwere independent predictors of high CBOE score andsubsequent mortality. Elderly patients should be re-garded as “high risk” for complications, and when theCBOE score is �7, consideration of withdrawal oftherapy should be entertained. There is no evidence foran age-associated elevated risk for relapse, so mainte-nance regimens are the same for younger and olderpatients with ANCA� vasculitis. Newer treatmentregimens using rituximab show great promise and mayeventually replace the standard cyclophosphamide-glucocorticoid treatment protocol, but few data areavailable to answer questions about the overall risk/benefit ratio of these regimens in the elderly withANCA� vasculitis (31). Relapsing disease seems to beespecially responsive to rituximab (31A). Anti-GBMantibody nephritis (alone or in combination withANCA) should be treated aggressively and promptlywith combined immunosuppression and intensiveplasma exchange, especially when life-threateningpulmonary hemorrhage is present (28). A delay oftreatment until the serum creatinine exceeds 7 to 8mg/dl is associated with poor results (35).

IgAN in the ElderlyIgAN is relatively uncommon in the elderly, but

this may be more apparent than real and might berelated to a lower incidence of renal biopsy perfor-mance in older patients with hematuria and slight tomodest proteinuria. In the elegant epidemiologic studyof Briganti et al. (36) from Victoria, Australia (aregion with a very high renal biopsy rate), the age-associated incidence of biopsy-proven IgA N wasapproximately 80 per million population per year inmen and approximately 25 per million population peryear in women. These gender-associated differences inoccurrence of IgAN remain unexplained. The courseof IgAN in the elderly is similar to younger patients,

after correcting for the effect of initial GFR and themagnitude of time-averaged proteinuria, but somestudies have suggested an independent adverse effectof older age on long-term outcome. The literature doesnot permit any conclusions regarding the efficacy orsafety of treatment of IgAN in the elderly because toofew patients older than 65 years were randomly as-signed in the very limited number of controlled trialsreported so far. In general, a conservative approach iswarranted using renin-angiotensin blockade if protein-uria is persistently �1.0 g/d (37). Adjunctive use ofimmunosuppression should probably be limited tothose who have clear evidence of progressive andpotentially reversible disease and are at low risk forcomplications. Patients with superimposed crescenticGN should be treated the same as for ANCA-associ-ated disease (see the Crescentic GN and Small VesselVasculitis in the Elderly section). The entity known as“IgA-dominant post-staphylococcal glomerulonephri-tis” is discussed in the Postinfectious GN in the El-derly section.

Membranoproliferative GN in the ElderlyThe morphologic pattern of membranoprolifera-

tive GN (MPGN) is uncommon in the elderly andwhen present usually is indicative of an underlyingdisease, such as a malignancy, a monoclonal gam-mopathy, a chronic viral infection (most often hepati-tis C), an autoimmune disease, or a disorder of com-plement regulation (e.g., deficiency in complementfactor H) (38). All elderly patients with a pathologicdiagnosis of MPGN need to be fully evaluated for oneor more of these disorders, especially with carefulimmunopatholgy studies using reagents to detectmonoclonal Ig deposits and electron microscopy.When associated with extensive crescentic disease,MPGN can be a devastating disease and produceESRD in a matter of months. Type II MPGN (alsoknown as dense deposit disease) is rare in the elderly(38); most cases in the older age groups fall into thecategory of type I MPGN, associated with subendo-thelial electron-dense deposits. When immunofluores-cence microscopy reveals C3 deposits only, oneshould consider a disorder of complement dysregula-tion, such as a complement factor H deficiency (38).

The prognosis of MPGN in the elderly may beworse than in younger patients, especially when su-perimposed crescentic disease is present. Not surpris-ing, the treatment of MPGN in the elderly is not well

46 Nephrology Self-Assessment Program - Vol 10, No 1, January 2011

Page 59: 2011.01 - Geriatric Nephrology

understood. If a complement dysregulation disorder isdefined, then infusion of fresh frozen plasma may beuseful (38). Treatment of an underlying malignancy ormonoclonal gammopathy can result in improvement,at least temporarily (39). The benefit-to-risk relation-ship of the of IFN and ribaviran combination regimensfor hepatitis C–related disease is not well known inelderly patients. When severe mixed (IgG/IgM) cryo-globulinemia is present, cryopheresis or plasma ex-change combined with immunosuppression can be lifesaving (40). Rituximab treatment may also be benefi-cial in severe symptomatic cryoglobulinemia, but therisks of this treatment are not well known in theelderly (41).

Postinfectious GN in the ElderlyAlthough acute postinfectious GN is most often

encountered in young children, it is a growing cause ofkidney disease in older individuals (42). In the elderly,postinfectious GN is most often observed in patientswith diabetes and those with impaired immune re-sponses. Unlike children, older patients commonlyhave staphylococcal (methicillin-sensitive and -resis-tant strains) rather than group A, �-hemolytic strepto-coccal infections as the cause. Skin abscesses, surgicalwound, and deep visceral infection predominate ratherthan pharyngeal infections. Acute renal failure, fluidoverload, and signs of congestive heart failure arequite common. Lowered serum complement values arefrequently seen. A proliferative form of GN is seen bylight microscopy on renal biopsy. Not infrequent,immunopathology reveals an IgA-dominant pattern ofIg deposition (43,44). Electron microscopy oftenshows electron-dense subepithelial deposits (humps)and subendothelial electron-dense deposits. The out-come, in general, is poor, particularly in the presenceof diabetes (44). Most patients will not recover com-pletely, and 30 to 50% will develop ESRD. Thetreatment is largely preventive. Early recognition andprompt antimicrobial treatment of the offending infec-tion may prevent or modify the course of disease. Latetreatment may have little effect on outcome, but thisis difficult to prove because of the lack of random-ized, controlled studies. Rarely, a crescentic GNmay ensue with a rapidly progressive course toESRD. The treatment of this uncommon complica-tion in the elderly is uncertain, but high-dosagesteroids and immunosuppression have been usedwith variable results. The complication rate of this

therapeutic approach is likely to be quite high in theelderly.

GN as a result of common viral infections, suchas hepatitis C and hepatitis B virus, may also affect theelderly. Typically, chronic hepatitis B infection isassociated with MN, normocomplementemia, and NS,whereas chronic hepatitis C infection is associatedwith MPGN hypocomplementemia (C4 � C3)andmixed IgG/IgM cryoglobulinemia with the NS. Hep-atitis B infection may respond to lamivudine, andhepatitis C infection may respond to �-IFN and riba-virin therapy, but these agents may produce seriousadverse effects in the elderly and should e used withcaution. Glomerular disease secondary to HIV infec-tion is largely confined to younger patients and is notdiscussed here.

LN in the ElderlySLE is an uncommon disease in the elderly, with

a reported prevalence of �20% in those who wereolder than 65 years in most reported series (45–47).Renal involvement is also less common in older com-pared with younger patients with SLE, and the severityof the renal disease is less in older patients. Womenpredominate as in younger patients with SLE, but thefemale-to-male ratio may be lower than in a youngerpopulation. Caucasian patients predominate in mostseries that have described SLE in the elderly (46,47).Serositis, lung involvement, and Sjogren-like syn-drome are more common, whereas cytopenias, skinrash, and photosensitivity are less common in theelderly with SLE. Overt renal disease at the onset ofSLE in the elderly is very uncommon. The serologicfeatures of SLE in the elderly are similar to those inyounger patients, although there may be less hypo-complementemia and more positive rheumatoid fac-tor and anti-Ro and anti-La autoantibodies in theelderly (47).

The spectrum of renal pathologic manifestationsof LN is similar to that found in younger patients, butthere may be an increased prevalence of MN (class VLN) (45–47). Amyloidosis is a very rare finding in theelderly with SLE.

The treatment of LN in the elderly can be chal-lenging because of the enhanced risk for complicationsfrom combined immunosuppression and the uncertain-ties engendered by the lack of randomized clinicaltrials of treatment focusing on this older age group.Treatment is largely based on the findings in renal

Nephrology Self-Assessment Program - Vol 10, No 1, January 2011 47

Page 60: 2011.01 - Geriatric Nephrology

biopsy and the severity of the clinical disease (48).Avoidance of cyclophosphamide-based regimens andfavoring mycophenolate-base regimens might be rea-sonable, but this is not proved (48). High dosages ofsteroids for prolonged periods may be very hazardousin the elderly age group, with a predilection for osteo-porosis, diabetes, and opportunistic infections. Overallsurvival is somewhat lower in the elderly with LN, butthis is mostly due to the consequences of aging ratherthan to the severity of the SLE on renal involvement.Overall life expectancy of women with LN now ap-proaches that of the general population (49).

Monoclonal Ig Deposition Diseases in theElderly

The monoclonal Ig deposition diseases (MIDD)are an important cause of renal disease in the elderly(50–53). Two broad forms can be distinguished: ALor AH amyloidosis and non-amyloid MIDD. The dis-tinction is based on the ability of some monoclonal Igsor fragments thereof (monoclonal light chains orheavy chains) to form �-pleated sheets that organizeinto fibrils staining with Congo Red or thioflavinedyes and exhibit birefringence in polarized light (amy-loid). It is also important to recognize that not alldeposits of amyloid are of Ig origin (e.g., hereditaryfibrinogen, lipoprotein or transthyretin amyloid, orsecondary serum amyloid A [SAA]) amyloidosis, seenwith chronic inflammatory diseases). Furthermore, notall fibrillar deposits of Igs are amyloid in nature (seethe Other Glomerular Diseases in the Elderly section).Collectively, the MIDD are seen much more fre-quently in the elderly than in younger individuals.Amyloidosis Systemic amyloidosis is a cause of NSin approximately 10% of elderly patients (52). It maypresent as an isolated renal disease (and thus seem tobe an “idiopathic” NS) or more commonly with someextrarenal manifestation (e.g., postural hypotension,carpal tunnel syndrome, easy bruising, organomegaly,macroglossia, diastolic cardiac dysfunction, diarrhea).The NS may be severe, often �20 g/day proteinexcretion. Systemic amyloidosis is most commonlydue to deposition of monoclonal � light chains (pri-mary AL Amyloid), but heavy-chain amyloid can alsooccur (AH amyloidosis). Approximately 10 to 15% ofpatients with multiple myeloma develop systemicamyloidosis, so most patients with systemic amyloid-osis do not have overt features of multiple myeloma(osteolytic bone lesions, anemia, M-protein spikes on

plasma protein electrophoresis). The diagnosis is usu-ally made upon detection of typical Congo Red–positive amyloid fibrils in biopsy material (kidney,abdominal fat pad). It should be emphasized that sometypical AL amyloid deposits do not stain for mono-clonal light chains, and AH amyloid deposits will notcontain any monoclonal light chains. Furthermore,hereditary amyloidosis will not contain light or heavyIg chains but will stain for the abnormal protein (e.g.,fibrinogen). Secondary amyloidosis will stain for theSAA protein only. These distinctions are importantbecause the treatment of AL/AH amyloidosis, hered-itary amyloidosis, and secondary (SAA) amyloidosisare so different. Serum free light chain (FLC) assaysare very useful in this regard because serum FLCs arevery commonly elevated and the �/� ratio is disturbedin primary AL systemic amyloidosis and AL amyloid-osis caused by multiple myeloma, whereas they arenormal in hereditary and secondary (SAA) amyloid-osis (54). Urinary excretion of monoclonal light chainsmay be increased even if no monoclonal paraprotein isdetected in the serum. Light-chain cast nephropathy(“myeloma kidney”) may coexist with MIDD whenmonoclonal light-chain production is marked (partic-ularly of the � variety) (53). Acute renal failure mayensue in this setting (53).

It should also be stressed that the disorder knownas “monoclonal immunoglobulinemia of uncertain sig-nificance” (MGUS) is a relatively common finding inthe elderly (55). This disorder, often discovered inci-dentally after serum protein electrophoresis of Ig im-munofixation studies of serum, is a precursor of overtmultiple myeloma (55). It may coexist with otherdiseases, including hereditary amyloidosis, and thuscreate confusion in diagnosis, particularly when thelevel of MGUS is low (�200 mg/dl) (56,57).

The prognosis of primary systemic (AL or AH)amyloidosis is very poor in the elderly, but it isimproving as a result of advances in treatment(52,58,59). Patients with NS often develop ESRD in afew years after discovery. Combined cardiac and renalinvolvement and elevated B-natriuretic peptide levelsin the serum is particularly ominous (52,58,59). Ag-gressive therapy with high-dosage melphalan plusdexamethasone or bortezomide and lenolinamide (acongener of thalidomide) have improved outcome inprimary systemic (AL/AH) amyloidosis (58,60). Au-tologous bone marrow or peripheral stem cell trans-plantation may also achieve improvement in outcome,

48 Nephrology Self-Assessment Program - Vol 10, No 1, January 2011

Page 61: 2011.01 - Geriatric Nephrology

but these procedures are poorly tolerated in the elderly(60,61), especially when there is cardiac involvement.The role of autologous stem cell transplantation is lessclear because of the effectiveness of modern chemo-therapy (61). Combined liver and kidney transplanta-tion may be curative in hereditary (fibrinogen) amy-loidosis with renal involvement (62).Non-Amyloid MIDD Disorders associated with thedeposition of monoclonal Igs or fragments thereof butwithout amyloid fibril formation are also common inthe elderly (50,51,53). These disorders consist of light-chain deposition disease (LCDD), heavy-chain depo-sition disease (HCDD), light/heavy-chain depositiondisease (L/HCDD), monoclonal (type I) cryoglobu-linemia, crystal cryoglobulinemia, monoclonal IgGdeposition disease (MIgGDD), and fibrillary/immuno-tactoid GN (FGN/ITGN) (50,51,53,63–70). The mostcommon of these MIDD in the elderly is LCDD. Thetypical presentation of LCDD is proteinuria, oftennephrotic range, and progressive renal impairment.Increased urinary monoclonal light-chain excretion(typically �) and increased serum FLC with a high �/�ratio is nearly always present (54,65,66). Characteris-tics of the disulfide residues in the monoclonal lightchains may determine character of the deposits (amy-loid versus non-amyloid) (65). Hypocomplementemiamay also be found (70), especially in HCDD orL/HCDD or in the presence of a monoclonal IgGdeposit disease with a proliferative GN (67,68). Renalbiopsies often reveal a nodular intercapillary glomer-ulosclerosis (resembling a Kimmelstiel-Wilson lesionof diabetic nephropathy) with monoclonal light-chaindeposition (typically �), but the lesions can be quiteheterogeneous (69). Electron microscopy may showelectron-dense deposits in a subendothelial or in-tramembranous location in glomeruli and/or in thetubular basement membranes (sometimes resemblingdense deposit disease or MPGN type II) (70).

When untreated, the prognosis is poor and ESRDis common, especially with older age and in thepresence of diabetes (71). However, chemotherapy,including chlorambucil and prednisone, melphalanand prednisone, and perhaps bortezomide, may improvethe outcome (72). The disease recurs in the renal allograftwhen treatment for the underlying plasma cell dyscrasiadoes not precede organ transplantation (73).

Monoclonal IgG deposition can cause a prolifer-ative form of GN superficially resembling an immunecomplex GN (67,68). MIDD can be associated with

non-amyloid fibrillary deposits. These fibrils (contain-ing monoclonal Ig) often take on characteristics oforganized microtubular structures by electron micros-copy. This group of disorders is often called “immu-notactoid glomerulonephritis” and are related to thenon-MIDD disorder called “fibrillary glomerulone-phritis” (63,64) (see below). Both disorders are com-monly found in older patients.

Drug-Associated Glomerular Disease in theElderly

Adverse reactions to drugs (hypersensitivityand/or direct nephrotoxicity) are a relatively uncom-mon cause of glomerular disease. However, because ofthe more common use of drug for treatment of illnessin the elderly, they are a more common cause ofglomerular disease in this age group. NonsteroidalAnti-inflammatory agents can cause NS and acuterenal failure as a result of MCD, with or withoutinterstitial nephritis (74). They have also been impli-cated as a cause of MN and perhaps FSGS as well(75). Oral or parenteral gold therapy for rheumatoidarthritis can cause a MN (76). Mercury compounds(organic and inorganic) taken orally, injected, or ap-plied topically can cause an MN (77). Penicillamine,rifampicin, allopurinol, hydralazine, propylthiouracil,and methimazole all have been associated withANCA� crescentic GN (78). Cancer chemotherapeu-tic agents can cause glomerular lesions. Pamidronateand other bisphosphonates used for hypercalcemia ofmalignancy and osteoporosis can cause collapsingFSGS (79). �-IFN and �-IFN and lithium can causeMCS and FSGS (80–82).

Other Glomerular Diseases in the ElderlyThe spectrum of other glomerular diseases that

may develop in the elderly is quite broad and cannotbe covered comprehensively here. Only two entitiesare covered: Fibrillary GN and idiopathic nodularglomerulosclerosis (63,64,83,84).

Fibrillary GN is an uncommon disorder thattends to affect older individuals (63,64). It is charac-terized by non-amyloid (Congo Red negative) fibril-lary deposits in glomeruli that contain polyclonal oroligoclonal (but very rarely monoclonal) IgG deposits.The cause and pathogenesis are unknown. NS, hema-turia, and progressive renal failure are common, buthypocomplementemia is rare. Treatment is generallyineffective, but when crescents are abundant, aggres-

Nephrology Self-Assessment Program - Vol 10, No 1, January 2011 49

Page 62: 2011.01 - Geriatric Nephrology

sive immunosuppression and plasma exchange may bebeneficial.

Idiopathic nodular glomerulosclerosis is a re-cently described disorder that has a predilection toaffect older individuals, in particular women with astrong history of excessive cigarette smoking (83,84).The lesion is very similar, if not identical, to nodularintercapillary glomerulosclerosis that is caused by di-abetes (Kimmelstiel-Wilson lesion), except that diabe-tes and retinopathy are absent but obesity can bepresent. The cause and pathogenesis are unknown, butchronic endothelial cell injury as a result of smokinghas been suggested (84). Proteinuria and progressiverenal failure are the rule. No treatment, other thanmeticulous BP control and use of renin-angiotensininhibition, is effective.

References1. Glassock RJ. Glomerular disease in the elderly. In: Nephrology and

Geriatrics Integrated, edited by Oreopoulos D, Hazzard W, Luke R,Dordrecht, Kluwer Academic Publishers, 2000, pp 57–66

2. Faubert P, Porush J: Primary glomerular disease. In: Renal Disease inthe Elderly, 2nd Ed., New York, Marcel Dekker, Inc, 1998, pp129–174

3. Abrass CK: Glomerulonephritis in the elderly. Am J Nephrol 5:409–418, 1985

4. Jayne D: Renal vasculitis in the elderly. In: The Aging Kidney inHealth and Disease, edited by Macias Nunez JF, Cameron JS,Oreopoulos DG, New York, Springer, 2008, pp 373–384

5. Davison AM, Johnston PA: Glomerulonephritis in the elderly. Neph-rol Dial Transplant 11[Suppl 9]: 34–37, 1996

6. Glassock RJ: Glomerular disease in the elderly population. GeriatrNephrol Urol 5: 149–154, 1998

7. Vendemia F, Gesualdo L, Schena FP, D’Amico G, Renal Immuno-pathology Study Group of the Italian Society of Nephrology: Epide-miology of primary glomerulonephritis in the elderly: Report fromthe Italian Registry of Renal Biopsy. J Nephrol 14: 340–352, 2001

8. Moutzouris D-A, Herlitz L, Appel GB, Markowitz GS, FreudenthalB, Radhakrishnan J, D’Agati VD: Renal biopsy in the very elderly.Clin J Am Soc Nephrol 4: 1073–1082, 2009

9. Ferro G, Dattolo P, Negrelli S, Michelassi S, Pizzarelli F: Clinicalpathologic correlates of renal biopsy in elderly patients. Clin Nephrol65: 243–247, 2006

10. Pincon E, Rioux-Leclercq N, Frouget T, Le Pogamp P, Vigneau C:Renal biopsies after 70 years of age: a retrospective longitudinalstudy from 200 to 2007 on 150 patients in Western France. ArchGerontol Geriatr 51: e120–e124, 2010

11. Nachman PH, Glassock RJ: Glomerular, vascular, and tubulointer-stitial diseases. NephSAP 9: 119–200, 2010

12. Waldman M, Crew RJ, Valeri A, Busch J, Stokes B, Markowitz G,D’Agati V, Appel G: Adult minimal-change disease: Clinical char-acteristics, treatment and outcomes. Clin J Am Soc Nephrol 2:445–453, 2007

13. Tse K-C, Lam M-F, Yip P-S, Li F-K, Choy B-Y, Lai K-N, ChanT-M: Idiopathic minimal change nephrotic syndrome in older adults:Steroid responsiveness and pattern of relapses. Nephrol Dial Trans-plant 18: 1316–1320, 2003

14. Eguchi A, Takei T, Yoshida T, Tsuchiya K, Nitta K: Combinedcyclosporine and prednisolone therapy in adult patients with the first

relapse of minimal-change nephrotic syndrome. Nephrol Dial Trans-plant 25: 124–129, 2010

15. Cattran DC, Alexopoulos E, Heering P, Hoyer PF, Johnston A,Meyrier A, Ponticelli C, Saito T, Choukroun G, Nachman P, PragaM, Yoshikawa N: Cyclosporin in idiopathic glomerular diseaseassociated with the nephrotic syndrome: Workshop recommenda-tions. Kidney Int 72: 1429–1447, 2007

16. Haffner D, Fischer DC: Nephrotic syndrome and Rituximab: Factsand perspectives. Pediatr Nephrol 24: 1433–1438, 2009

17. Deegens JK, Wetzels JF: Membranous nephropathy in the olderadult: Epidemiology, diagnosis and management. Drugs Aging 24:717–732, 2007

18. Lefaucheur C, Stengel B, Nochy D, Martel P, Hill GS, Jacquot G,Rossert J, GN-PROGRESS Study Group. Membranous nephropathyand cancer: Epidemiologic evidence and determinants of high-riskcancer association. Kidney Int 70: 1510–1517, 2006

19. Beck LH, Salant DJ: Membranous nephropathy: Recent travels andnew roads ahead. Kidney Int 77: 765–779, 2010

20. Ohtani H, Wakui H, Komatsuda A, Okuyama S, Masai R, Maki N,Kigawa A, Sawada K, Imai H: Distribution of glomerular IgGsubclass deposits in malignancy-associated membranous nephropa-thy. Nephrol Dial Transplant 19: 574–579, 2004

21. Bjornklett R, Vikse BE, Svarstad E, Assarad K, Bostad L, LangmarkF, Iversen BM: Long-term risk of cancer in membranous nephropathypatients. Am J Kidney Dis 50: 396–403, 2007

22. Lindskog A, Ebefors K, Johansson ME, Stefansson B, Granqvist A,Arnadottir M, Berg AL, Nystrom J, Haraldsson B: Melanocortin 1receptor agonists reduce proteinuria. J Am Soc Nephrol 21: 1290–1298, 2010

23. Segarra A, Praga M, Ramos N, Polanco N, Cargol I, Gutierrez-SoliaE, Gomez MR, Montoro B, Camps J: Successful treatment of mem-branous glomerulonephritis with rituximab in calcineurin inhibitordependent patients. Clin J Am Soc Nephrol 4: 1083–1088, 2009

24. Polanco N, Gutierrez E, Covarsi A, Ariza F, Carrefio A, Vigil A,Baltar J, Fernandez-Fresnedo G, Martin C, Pons S, Lorenzo D, BernisC, Arrizabataga P, Fernandez-Jaurez G, Barrio V, Sierra M, Castel-lanos I, Espinosa M, Rivera F, Oliet A, Fernandez-Vega F, Praga M,Grupo de Estudio delas Enfermedades Glomerulares de la SociedadEspanola de Nefrologia: Spontaneous remission of nephrotic syn-drome in idiopathic membranous nephropathy. J Am Soc Nephrol 21:697–704, 2010

25. Higgins RM, Goldsmith DJ, Connolly J, Scoble JE, Hendry BM,Ackrill P, Venning MC: Vasculitis and rapidly progressive glomer-ulonephritis in the elderly. Postgrad Med J 72: 41–44, 1996

26. Uezono S, Hara S, Sato Y, Komatsu H, Ikeda N, Shimao Y, HayashiT, Asada Y, Fujimoto S, Eto T: Renal biopsy in elderly patients: Aclinicopathological analysis. Ren Fail 26: 549–555, 2006

27. Bonsib SM, Goeken JA, Kemp JD, Chandran P, Shadur C, Wilson L:Co-existent anti-neutrophil cytoplasmic antibody and anti-glomerularbasement membrane antibody associated disease: A report of sixcases. Mod Pathol 6: 526–530, 1993

28. Lindic J, Vizjak A, Ferluga D, Kovac D, Ales A, Kveder R, PonikvarR, Bren A: Clinical outcome of patients with co-existent antineutro-phil cytoplasmic antibodies and antibodies against glomerular base-ment Membrane. Ther Apher Dial 13: 278–281, 2009

29. Mukhtyar C, Guillevin L, Cid MC, Dasgupta B, de Groot K, GrossW, Hauser T, Hellmich B, Jayne D, Kallenberg CG Merkel PA,Raspe H, Salvarani C, Scott DG, Stegeman C, Watts R, Westmen K,Yaziki H, Luqmani R, European Vasculitis Study Group: EULARrecommendations for the management of primary small and mediumvessel vasculitis. Ann Rheum Dis 68: 310–317, 2009

30. Walters GD, Wills NS, Craig JC: Interventions for renal vasculitis inadults: A systematic review. BMC Nephrol 11: 12, 2010

31. Jayne DR, Gaskin G, Rasmussen N, Abramowicz D, Ferrario F,Guillevin L, Miraplex E, Savage CO, Sinico RA, Stegeman CA,Westman KW, van der Woude FJ, van Wijngaarden de Lind RA,

50 Nephrology Self-Assessment Program - Vol 10, No 1, January 2011

Page 63: 2011.01 - Geriatric Nephrology

Pusey CD, European Vasculitis Study Group: Randomized trial ofplasma exchange or high-dosage methylprednisolone as adjunctivetherapy for severe renal vasculitis. J Am Soc Nephrol 18: 2180–2188,2007

31. Stone JH, Merkel PA, Spiera R, Seo P, Langford CA, Hoffman GS,Kallenberg CG, St. Clair EW, Turkeiwicz A, Tchao NK, Webber L,Ding L, Sejismundo LP, Mieras K, Weltzenkamp D, Ikle D, Seyfert-Margolis V, Mueller M, Bruneltta P, Allen NG, Fervenza F, GeethaD, Keogh KA, Kissin EY, Monach PA, Peikert T, Stegeman C,Ytterberg SR, Specks U, RAVE-ITN Research Group. Rituximabversus cyclophosphamide for ANCA-associated vasculitis. N EnglJ Med 363: 221–232, 2010

32. Little MA, Nightingale P, Verburgh CA, Hauser T, de Groot K,Savage C, Jayne D, Hauser L, European Vasculitis Study Group:Early mortality in systemic vasculitis: Relative contributions ofadverse events and active vasculitis. Ann Rheum Dis 69: 1036–1043,2010

33. Little MA, Nazar L, Farrington K: Outcome in glomerulonephritisdue to small vessel vasculitis: Effect of functional status and non-vasculitic morbidity. Nephrol Dial Transplant 19: 356–364, 2004

34. Brijker F, Magee CC, Tervaert JW, O’Neill S, Walshe JJ: Outcomeanalysis of patients with vasculitis associated with antineutrophilcytoplasmic antibodies. Clin Nephrol 52: 344–351, 1999

35. Levy JB, Turner AN, Rees AJ, Pusey CD: Long-term outcome ofanti-glomerular basement membrane antibody disease treated withplasma exchange and immunosuppression. Ann Intern Med 134:1033–1042, 2001

36. Briganit EM, Dowling J, Finlay M, Hill PA, Jones CL, Kincaid-Smith PS, Sinclair R, McNeil JJ, Atkins RC: The incidence of biopsyproven glomerulonephritis in Australia. Nephrol Dial Transplant 16:1364–1367, 2001

37. Floege J, Eitener F: Immune modulating therapy for IgA nephropa-thy: Rationale and evidence. Semin Nephrol 28: 38–47, 2008

38. Glassock RJ: Membranoproliferative glomerulonephritis. In: Evi-dence Based Nephrology, edited by Molony DA, Craig JC, Oxford,Wiley-Blackwell, 2009, pp 183–198

39. Sethi S, Zand L, Leung N, Smith RJ, Jevremonic D, Hermann SS,Fervenza F: Membranoproliferative glomerulonephritis secondary tomonoclonal gammopathy. Clin J Am Soc Nephrol 5: 770–782, 2010

40. Rockx MA, Clark WF: Plasma exchange for treating cryoglobuline-mia: A descriptive analysis. Transfus Apher Sci 42: 247–251, 2010

41. da Silva Fucuta Pereira P, Lemos LB, de Oliveiraueharaqq SN, deSouza E, Silva AE, Ferraz MI: Long-term efficacy of rituximab inhepatitis C virus associated cryoglobulinemia. Rheum Int 30: 1515–1518, 2010

42. Moroni G, Ponticelli C: Acute post-infectious glomerulonephritis. In:Treatment of Primary Glomerulonephritis, 2nd Ed., edited by Ponti-celli C, Glassock RJ, Oxford, Oxford Medical Publishers, 2009, pp133–178

43. Haas M, Racusen LC, Bagnasco SM: IgA-dominant post-infectiousglomerulonephritis: A report of 13 cases with common ultrastructuralfeatures. Hum Pathol 39: 1309–1316, 2008

44. Nasr SH, Share DS, Vargas MT, D’Agati VD, Markowitz GS: Acutepost-staphylococcal glomerulonephritis superimposed on diabeticglomerulosclerosis. Kidney Int 71: 1317–1321, 2007

45. Rovensky J, Tuchynova A: Systemic lupus erythematosus in theelderly. Autoimmun Rev 7: 235–239, 2008

46. Lalani S, Pope J, de Leon F, Peschken C, Members of CaNIOS/1000Faces of Lupus: Clinical features and prognosis of late-onset systemiclupus erythematosus: Results from the 1000 faces of lupus study.J Rheumatol 37: 38–44, 2010

47. Boddaert J, Huong DL, Amoura Z, Weschsler B, Godeau P, Piett JC:Late-onset systemic lupus erythematosus: A personal series of 47patients and pooled analysis of 714 cases in the literature. Medicine(Baltimore) 83: 348–359, 2004

48. Glassock RJ: Treatment of severe lupus nephritis. In: Lupus Nephri-

tis, 2nd Ed., edited by Lewis E, Schwartz M, Korbet S, Oxford,Oxford Medical Publishers, 2010 (in press)

49. Stratta P, Mesiano P, Campo A, Grill A, Ferrero S, Santi S, Besso L,Mazzucco G, Rosso S, Spitale A, Fop F, Ciccione G: Life expectancyof women with lupus nephritis now approaches that of the generalpopulation. Int J Immunopathol Pharamacol 22: 1135–1141, 2009

50. Glassock RJ: Glomerular disease in the elderly. Clin Geriatr Med 25:413–422, 2009

51. Touchard G, Aucountier P, Hermine O, Ronco P, eds.: MonoclonalGammopathies and the Kidney, Dordrecht, Kluwer Academic Pub-lishers, 2003

52. Dember LM: Amyloidosis-associated kidney diseases. J Am SocNephrol 17: 3458–3471, 2006

53. Lin J, Markowitz GA, Valeri AM, Kambham N, Sherman WH, AppelGB, D’Agati VD: Renal immunoglobulin deposition diseases: Theclinical spectrum. J Am Soc Nephrol 12: 1482–1492, 2001

54. Katzmann JA: Screening panels for monoclonal Gammopathies:Time to change. Clin Biochem Rev 30: 105–111, 2009

55. Kyle RA, Therneau TM, Rajkumar SV, Larson DR, Plevak MF,Offord JR, Dispenzieri A, Katzmann JA, Mellon LJ: Prevalence ofmonoclonal gammopathy of undetermined significance. N EnglJ Med 354: 1362–1369, 2006

56. Lachmann HJ, Booth DR, Booth SE, Bybee A, Gilbertson JA,Gillmore JD, Pepys MB, Hawkins PN: Misdiagnosis of hereditaryamyloidosis as AL (primary) amyloidosis. N Engl J Med 346:1786–1791

57. Glassock RJ, Bargman J, Palmer BF, Samaniego M, Fervenza F:Nephrology quiz and questionnaire: 2009. Clin J Am Soc Nephrol 5:1141- 1160: 2010

58. Kastritis E, Wechalekar AD, Dimopoulos MA, Merlini G, HawkinsPN, Perfetti V, Gilmore JD, Palladini G: Bortezomib with or withoutdexamethasone in primary (light chain) amyloidosis. J Clin Oncol 28:1031–1037, 2010

59. Dimopoulos MA, Kastritis E, Rajkumar SV: Treatment of plasma celldyscrasias with lenalidomide. Leukemia 22: 1343–1353, 2008

60. Gertz MA, Zeldenrust SR: Treatment of immunoglobulin light chainamyloidosis. Curr Hematol Malig Rep 4: 91–98, 2009

61. Dember LM: Modern treatment of amyloidosis: Unresolved ques-tions. J Am Soc Nephrol 20: 469–472, 2009

62. Stangou AJ, Banner NR, Hendry BM, Reia M, Portmann B, WendonJ, Monaghan M, Maccarthy P, Buxton-Thomas M, Mathias CJ,Liepnieks JJ, O’Grady J, Heaton ND, Benson MD: Hereditary fibrin-ogen A alpha chain amyloidosis: Phenotypic characterization of asystemic disease and the role of liver transplantation. Blood 115:2996–3007, 2010

63. Herrera GA, Turbal-Herrera EA: Renal diseases with organizeddeposits: An algorithmic approach to classification and clinicopath-ologic diagnosis. Arch Pathol Lab Med 134: 512–531, 2010

64. Alpers CE, Kowalewska J: Fibrillary glomerulonephritis and immu-notactoid glomerulopathy. J Am Soc Nephrol 19: 34–37, 2008

65. Kaplan B, Ramirez-Alvarado M, Sikkink L, Golderman S, Dispen-zieri A, Livneh A, Gallo G: Free light chains in plasma of patientswith light chain amyloidosis and non-amyloid light chain depositiondisease: High proportion and heterogeneity of disulfide-linked mono-clonal free light chains as pathogenic features of amyloid disease.Br J Haematol 144: 705–715, 2009

66. Davern S, Tang LX, Williams TK, Macy SD, Wall JS, Weiss DT,Solomon A: Immunodiagnostic capabilities of anti-free immunoglob-ulin light chain monoclonal antibodies. Am J Clin Pathol 130:702–711, 2008

67. Nasr SH, Markowitz GS, Stokes MB, Seshan SV, Valderrama E,Appel GB, Aucouturier P, D’Agati VD: Proliferative glomerulone-phritis with monoclonal IgG deposits: A distinct entity mimickingimmune-complex glomerulonephritis. Kidney Int 65: 85–96, 2004

68. Soma J, Sato K, Sakuma T, Saito H, Sato H, Sato T, Abbas A,Aucouturier P: Immunoglobulin gamma3 heavy-chain deposition

Nephrology Self-Assessment Program - Vol 10, No 1, January 2011 51

Page 64: 2011.01 - Geriatric Nephrology

disease: Report of a case and the relationship to hypocomple-mentemia. Am J Kidney Dis 43: E10–E16, 2004

69. Godken N, Barlogie B, Liapas H: Morphologic heterogeneity of renallight-chain deposition disease. Ultrastruct Pathol 32: 17–24, 2008

70. Salant DJ, Sanchorawala V, D’Agati VD: A case of atypical lightchain deposition disease: Diagnosis and treatment. Clin J Am SocNephrol 2: 858–867, 2007

71. Stratta P, Gravellone L, Cena T, Rossi D, Gaidano G, Fenoglio R,Lazzarich E, Quaglia M, Airoldi A, Bozzola C, Monga G, Valente G,Canavese C, Magnani C: Renal outcome and monoclonal immuno-globulin deposition disease in 289 old patients with blood celldyscrasias: A single center experience. Crit Rev Oncol Hematol May28, 2010 [epub ahead of print]

72. Hassoun H, Flombaum D, D’Agati VD, Rafferty BT, Cohen A,Klimek VM, Boruchov A, Kewalramani T, Reich L, Nimer SD,Comenzo RL: High-dose melphalan and auto-SCT in patients withmonoclonal Ig deposition disease. Bone Marrow Transplant 42:405–412, 2008

73. Leung N, Lager DJ, Gertz MA, Wilson K, Kanakiriya S, Fervenza F:Long-term outcome of renal transplantation in light-chain depositiondisease. Am J Kidney Dis 43: 147–153, 2004

74. Kleinknecht D: Interstitial nephritis, the nephrotic syndrome andchronic renal failure secondary to nonsteroidal anti-inflammatorydrugs. Semin Nephrol 15: 228–235, 1995

75. Markowitz GS, Falkowitz DC, Isom R, Zaki M, Imaizumi S, AppelGB, D’Agati VD: Membranous glomerulopathy and acute interstitialnephritis following treatment with celecoxib. Clin Nephrol 59: 137–142, 2003

76. Bigazzi PE: Metals and kidney auto-immunity. Environ Health Per-spect 107[Suppl 5]: 753–765, 1999

77. Li SJ, Zhang SH, Chen HP, Zeng CH, Zheng CK, Li SH, Liu ZH:Mercury-induced membranous nephropathy: Clinical and pathologi-cal features. Clin J Am Soc Nephrol 5: 439–444, 2010

78. Chen M, Kallenberg CG: The environment, geoepidemiology andANCA-associated vasculitides. Autoimmun Rev 9: A293–A298, 2010

79. Markowitz GS, Fine PL, D’Agati VD: Nephrotic syndrome aftertreatment with pamidronate. Am J Kidney Dis 39: 1118–1122, 2002

80. Bosquet S, Descombes E, Gauthier T, Fellay G, Regamey G: Ne-phrotic syndrome during lithium therapy. Nephrol Dial Transplant12: 2728–2731, 1997

81. Kumasaka R, Nakamura N, Shirato K, Fujita T, Mrakami R, ShimadaM, Nakamura M, Osawa H, Yamada H, Okumura K: Nephroticsyndrome associated with interferon-beta-1b therapy for multiplesclerosis. Clin Exp Nephrol 10: 222–225, 2006

82. Dimitrov Y, Heibel F, Marcellin L, Chantrel F, Moulin B, Han-nadouche T: Acute renal failure and nephrotic syndrome with alphainterferon therapy. Nephrol Dial Transplant 12: 200–203, 1997

83. Li W, Verani RR: Idiopathic nodular glomerulosclerosis: A clinico-pathologic study of 15 cases. Hum Pathol 39: 1771–1776, 2008

84. Nasr SH, D’Agati VD: Nodular glomerulosclerosis in the nondiabeticsmoker. J Am Soc Nephrol 18: 2032–2036, 2007

Acute Kidney Injury

Incidence and Risk Factors for Acute KidneyInjury

It is clear that the elderly are at the very highestrisk for the development of acute kidney injury (AKI).Indeed, Feest et al. (1) demonstrated that there is athree- to eightfold progressive, age-dependent increasein the frequency of development of community-ac-quired AKI in patients who are older than 60 years. In

the past 25 years, the mean age of patients with AKIhas increased by at least 5 years and perhaps as muchas 15 years (2). Groeneveld et al. (3) demonstrated thatthe age-related yearly incidence of AKI rose from 17per million in adults who were younger than 50 yearsto 949 per million in those who were aged 80 to 89years. A 9-year prospective study in Madrid, Spain,demonstrated a 3.5-fold greater incidence of AKI inpatients who were older than 70 years (4). Ali et al. (5)demonstrated that the average age of patients withAKI in a large European cohort was 76 years. How-ever, the average age of patients with acute on chronicrenal failure was 80.5 years, and this group had a muchhigher risk for adverse outcomes (5). Hsu et al. (6)demonstrated that the incidence of AKI had increasedfrom 1996 to 2003, and this increase was most dra-matic in patients aged �80 years: An incidence of2867.5 per 100,000 person-years in 1996 to 1997rising to 4884.3 per 100,000 person-years in 2002 to2003.

Older studies that analyzed epidemiology werehampered by variable definitions of AKI. By includingsmall rises of creatinine in their criteria, the RIFLEand AKIN classifications aim at diagnosing AKI ear-lier and standardizing definitions across studies (7).The RIFLE and AKIN classification systems havenow been well validated in several large studies witha mean patient age �60 years; thus, there is goodevidence that this classification scheme should func-tion well in the elderly population (8). In a recentstudy that compared the RIFLE and AKIN classifica-tions, Joannidis et al. (9) analyzed 16,784 patientsduring the initial 48 hours of their intensive care unitstay. The incidence of AKI was found to range be-tween 28.5 and 35.5% when applying AKIN andRIFLE criteria, respectively. The mean age was 63years, and 25% of patients were older than 75 years.

Although all causes of AKI are encountered inthis age group, the frequency of prerenal (estimates of30%) and obstructive (estimates of 25%) causes isespecially prevalent in the elderly (10). Furthermore,elderly patients are more frequently subjected to inva-sive procedures, exposed to multiple (and possiblynephrotoxic) medications, and to exposed to radiocon-trast agents. The changes in drug metabolism anddisposition that occur with aging also contribute to ahigher risk for drug-induced AKI (11). Importantly,55% of all intensive care unit bed-days are occupiedby patients aged �65 years, and this is a population at

52 Nephrology Self-Assessment Program - Vol 10, No 1, January 2011

Page 65: 2011.01 - Geriatric Nephrology

high risk for AKI as a result of associated comorbidconditions such as sepsis and multisystem organ fail-ure (12). In a prospective, multicenter study of thecauses and outcomes of 103 episodes of AKI, Pascualand Liano (13) separated patients into three categoriesof age—younger than 65 years (group 1), 65 to 79years (group 2), and �80 years (group 3)—and thencompared group 1 with group 3. They found that mostcases of AKI in the elderly were caused by acutetubular necrosis, prerenal causes, and obstruction.These findings were supported by Akposso et al. (14)in their historical cohort analysis on AKI epidemiol-ogy in critically ill octogenarians. Older studies, incontrast, found that drug-related toxicity was mostcommonly involved in this age group (4).

Elderly individuals also are more likely to havechronic kidney disease (CKD), congestive heart fail-ure, hypertension, renovascular disease, and diabetesand to undergo surgery (especially cardiac and vascu-lar surgery); all of these are significant risk factors forthe development of AKI, in part because patients withthese conditions and risks are exposed to nephrotoxicradiocontrast agents, angiotensin-converting enzymeinhibitors or angiotensin receptor blockers, and non-steroidal anti-inflammatory drugs (15).

The incidence of AKI in patients who areolder than 70 years is increasing, and thispatient population is at the highest risk forthe development of AKI.

Structural and Cellular Changes in the AgingKidney

In the absence of a specific disease, the kidneyundergoes age-dependent structural, cellular, and func-tional alterations that in aggregate likely increase thesusceptibility to development of AKI in the right clinicalcontext. The aging kidney develops a significant de-crease in renal mass, functioning nephron numbers,and baseline kidney function (16–21). With renalsenescence, there is a variable but not absolute de-crease in GFR that averages approximately 1 ml/minper y beginning at approximately age 45 years (22,23).

Along with these changes, effective renal bloodflow (RBF) decreases up to 10% per decade, and thisis especially the case when assessing the ability of thekidney to vasodilate in response to intravenous amino

acids or a high-protein diet, a technique that is used toassess the functional integrity or reserve of the kidney(24). The fall in RBF is due to a higher renal vascularresistance in elderly individuals that may be related, inpart, to reduced nitric oxide production in the elderly(25). These changes in renal hemodynamics likelylead to an increased risk for AKI. For instance, mag-netic resonance imaging using changes in blood oxy-genation with an induced water diuresis in patientsaged 59 to 79 years demonstrated the inability toimprove medullary oxygenation in the older as com-pared with younger patients (26). Thus, in combina-tion with dehydration, a disturbance in autoregulatorydefense mechanisms that would normally preserveGFR and RBF (e.g., increased renal vascular resis-tance) can in the elderly kidney lead to ischemia andAKI as a result of drastic falls in RBF.

In addition to alterations in renal function andhemodynamics, aging renal cells may be more vulner-able to damaging insults as a result of changes in cellularand molecular function (summarized in Table 3). Chenet al. (34) investigated the influence of age on theresponse to ischemic injury and aimed to determinecandidate genes that mediate this differential response.The expression of 92 genes was changed by aging(either increased or decreased), including claudin 7,kidney injury molecule 1, and matrix metalloprotein-ase 7. The authors took advantage of the widelyknown observation that calorie restriction partiallyreverses susceptibility to ischemic insults in aged ratsand demonstrated that the changes in these candidategenes with aging can be reversed with calorie restric-tion. Schmitt et al. (35) also used microarray tech-niques to identify increased proximal tubular expres-sion of zinc-�(2)-glycoprotein (Zag) in older kidneys.Zag had been previously implicated in epithelial cellproliferation inhibition, and the increased expressionmay mechanistically explain some of the increasedsusceptibility of aged kidneys to nephrotoxic insults(35). The identification of these key injury-responsegenes that are altered with aging may allow for de-tailed investigations into the mechanisms underlyingthe age-related susceptibility to injury.

Calorie Restriction and SIRT1Importantly, calorie restriction suppresses age-

related oxidative stress as well as the susceptibility toischemic injury (42). The vast majority of studies onthe renal effects of calorie restriction have focused on

Nephrology Self-Assessment Program - Vol 10, No 1, January 2011 53

Page 66: 2011.01 - Geriatric Nephrology

the prevention of chronic nephrosclerosis in animalmodels (43,44). In rats, calorie restriction is partiallyprotective against ischemic insults to the kidney (34).One of the compelling findings regarding the effects ofcalorie restriction has been the association between theincreased level of sirtuins and the lifespan extension ofcalorie restriction (45). Sirtuins are members of thesilent information regulator 2 (Sir2) family, a familyof class III histone/protein deacetylases that are in-creased in expression after calorie restriction (45).There are seven mammalian sirtuins (SIRT1 through7); SIRT1 is the best studied (46). SIRT1 deacetylatesa large number of transcriptional factors and co-factorsinvolved in cell growth, differentiation, stress resis-tance, reducing oxidative damage, and metabolism(46). SIRT1 levels increase in the kidney after a24-hour fasting period, but whether SIRT1 has a directrole in renal protection is yet to be determined (47).Interestingly, a plant polyphenol, resveratrol, is a po-tent activator of SIRT1 activity and has been shown tohave renoprotective effects in several nephrotoxic andischemic model systems (37,48,49). Further work isneeded to determine whether activators of SIRT1 mayprove useful in protecting the aging kidney.

Diagnosis of AKI in the ElderlyAKI is traditionally diagnosed by an abrupt rise

in serum creatinine, with or without a decrease in urineoutput. The rate and magnitude of the increase inserum creatinine levels may be blunted in the elderlyas a result of a decrease in lean muscle mass andcreatinine generation. Thus, serum creatinine may notbe an ideal biomarker for AKI in the elderly becauseof a delay in the rate of creatinine rise as well as a

decrease in peak serum creatinine. Indeed, because ofthese factors, clinicians need to be sensitive to evensmall changes in serum creatinine in elderly patients.

Several biomarkers for the early diagnosis ofAKI have been developed to surmount the difficultieswith serum creatinine, including cystatin C, kidneyinjury molecule 1, neutrophil gelatinase–associatedlipocalin (NGAL), and IL-18 (50). Interestingly, in thecase of NGAL, the diagnostic utility of the biomarkermay be modified by age such that the test performswith a greater area under the curve in children versusolder adults and also suggests that baseline renaldisease may affect the utility of biomarkers (51). Thus,biomarkers for the diagnosis of AKI must be validatedacross a spectrum of ages, including the elderly.

Impact of AKI in the ElderlyMortality The short-term mortality of elderly pa-tients with AKI is as high as 40% depending on thesetting, definition of AKI, and specific age cutoffs(14,52). Two studies demonstrated that mortality ratesassociated with AKI have decreased despite an in-crease in comorbidity; this fall in mortality includeselderly patients (52,53). A study in Madrid also dem-onstrated that the mortality in patients who were olderthan 80 years and had AKI was not greater than thosewho were younger than 65 years (13). A retrospectivestudy of 82 patients who required dialysis after cardiacsurgery found that patients who were aged �70 yearshad a hospital mortality equal to that of patients whowere younger than 70 years (54). Importantly, thedecision to initiate dialysis in this study likely re-flected a bias toward patients with a better prognosisand thus may not accurately reflect the impact of age

Table 3. Cellular and molecular changes in the aging kidney that may increase the susceptibility to AKI

Cellular Changes Molecular Changes

Increased rates of apoptosis (27) Increased expression of messenger RNA (mRNA) and proteinsof candidate genes associated with senescence: Cell-cycleinhibitor p16INK4a, p53, cyclooxygenase 1 and 2, transforminggrowth factor �-1, and heat shock protein A5 (29)

Telomere length shortening in the renal (28) Increased expression of genes inhibiting cellular proliferation(34,35)

Decline in antioxidant defense mechanisms (30) Age-related changes in klotho expression (36)Increase in oxidant stress (31) Age-related changes in sirtuin (SIRT1) expression (37)Mitochondrial abnormalities and ATP depletion (30) Increased advanced glycation end products (AGEs) (38)Decreased in rate of cellular proliferation (32) Decreases in renal growth factors (IGF-1, EGF, VEGF) (39–41)Decrease in stem cell/progenitor cell number and

function (33)ATP, adenosine triphosphate; VEGF, vascular endothelial growth factor.

54 Nephrology Self-Assessment Program - Vol 10, No 1, January 2011

Page 67: 2011.01 - Geriatric Nephrology

alone. Despite these recent encouraging changes, theshort-term mortality associated with AKI in the elderlyremains high.

The development of AKI is also associated in-dependently with an increased risk for long-term mor-tality (55). As reviewed recently by Coca (55), evensmall changes in serum creatinine levels are associatedwith long-term death, and greater changes are associ-ated with greater risks. Although age has an effect onthe association between AKI and mortality, the impactof AKI is diluted with aging as a result of the cumu-lative effects of other, competing comorbidities (55).Risk for CKD and ESRD One of the hallmarks ofaging is impairment in the ability to repair and regen-erate injured cells. In patients exposed to nephrotoxicinsults, this impairment in repair processes can bereflected in several ways: (1) AKI may be moreprolonged as a result of impaired healing, and (2) AKImay never recover and indeed may progress to CKDand ESRD. Several clinical observations support thesepossibilities in elderly patients. Elderly patients whoexperience an episode of AKI have a 13-fold higherrelative risk for developing ESRD (this number risesto 41.2-fold when patients have baseline CKD) (56).Along those lines, a systematic review and meta-analysis of recovery of kidney function after AKI inthe elderly demonstrated that recovery after AKI isapproximately 28% less likely to occur when thepatient is older than 65 years (57). Ponte et al. (58)reviewed the long-term GFR changes after an episodeof AKI in 187 patients during a 10-year follow-upperiod. In that cohort, 19% of patients showed pro-gressive declines in renal function, and a regressionmodel identified age, comorbid conditions, dischargeestimated GFR, and follow-up time as independentpredictors of long-term renal function (58). Most re-cently, a Canadian study of 3769 adults who had AKIand required in-hospital dialysis demonstrated thatpatients who were aged �65 years had a significantlyhigher risk for ESRD than patients who were youngerthan 65 years after an episode of AKI (59). Ishani et al.(60) studied a large cohort of patients who were olderthan 67 years and had experienced AKI and demon-strated that the hazard ratio for developing ESRD was41.2 in patients with AKI and CKD relative to thosewithout CKD and 13 for patients with AKI and with-out CKD. However, these studies cannot demonstratewhether the relationship between AKI and CKD/ESRD is causal or related to residual confounding. In

any case, data such as these highlight the need forpreventive strategies and close monitoring of elderlypatients after AKI.Quality of Life Few studies have specifically ad-dressed the quality-of-life outcomes after a seriousoccurrence of AKI (usually requiring dialysis). Evenfewer have included very elderly patients. A generalconclusion of the majority of these studies is that manypatients experience some limitation in physical func-tioning after serious AKI but despite this are stillsatisfied with their overall condition (61–63). In fact,Gopal et al. (63) studied 35 patients (mean age 58.0years) 2.5 years after an episode of AKI that requireddialysis, and 86.5% were satisfied with their healthdespite that 41.9% of patients could not walk morethan 200 m. Importantly, 94.5% of this cohort believedthat their treatment was worthwhile, and 91.2% wouldundergo the same treatment again if needed (63). Datasuch as these may be helpful in the decision-makingprocess to initiate dialysis therapy in the elderly withAKI.

Management of AKI in the ElderlyThere are few effective strategies to treat estab-

lished AKI, with the exception of renal replacementtherapy. The indications and use of renal replacementtherapy in the elderly are not different from other agegroups because the outcomes with these modalities inthe elderly do not seem to be different from other agegroups (55). However, decisions to proceed with ag-gressive support modalities should include a carefulassessment of comorbid conditions, prognosis formeaningful recovery, and quality-of-life issues. Theseissues are often much more salient in the elderly andwill require thoughtful decision making in conjunctionwith the patient, family, and other caregivers.

Given the limited therapeutic options for AKIand the significant impact of AKI on outcomes, it isimperative that preventive strategies be used wheneverpossible. Several of these preventive strategies arelisted in Table 4. Key to these strategies is earlydetection of patients at risk. Given that a major riskfactor for AKI is impaired baseline renal function(CKD), careful assessment of baseline renal functionin the elderly at the time of hospital admission orexposure to potential risk is critical. In this regard,serum creatinine levels in the elderly may be mislead-ing and give the false sense of relatively normal renalfunction as a result of diminished muscle mass and

Nephrology Self-Assessment Program - Vol 10, No 1, January 2011 55

Page 68: 2011.01 - Geriatric Nephrology

creatinine generation rate. The use of regression equa-tions such as the Modification of Diet in Renal Disease(MDRD) or Chronic Kidney Disease Epidemiology(CKD-EPI) formulas are discussed elsewhere.

Given limited therapeutic options for AKIand the significant impact of AKI on out-comes, it is imperative that preventivestrategies be used whenever possible.

References1. Feest TJ, Round A, Hamad S: Incidence of severe acute renal failure

in adults: Results of a community-based study. BMJ 306: 481–489,1993

2. Turney JH, Marshall DH, Brownjohn AM, Ellis CM, Parsons FM:The evolution of acute renal failure. QJM 74: 83–89, 1990

3. Groeneveld AB, Tran DD, Van der Meulen J, Nauta JJ, Thijs LG:Acute renal failure in the intensive care unit: Predisposing, compli-cating factors affecting outcome. Nephron 59: 602–607, 1991

4. Pascual J, Orofino L, Liano F, Marcen R, Naya MT, Orte L, OrtunoJ: Incidence and prognosis of acute renal failure in older patients.J Am Geriatr Soc 38: 25–32, 1990

5. Ali T, Khan I, Simpson W, Prescott G, Townsend J, Smith W,

Macleod A: Incidence and outcome in acute kidney injury: A com-prehensive population-based study. J Am Soc Nephrol 18: 1292–1298, 2007

6. Hsu C-Y, McCullough CE, Fan D, Ordonez JD, Chertow GM, GoAS: Community-based incidence of acute renal failure. Kidney Int72: 208–212, 2007

7. Bagshaw SM, George C, Bellomo R: A comparison of the RIFLE andAKIN criteria for acute kidney injury in critically ill patients. NephrolDial Transplant 23: 1569–1574, 2008

8. Lopes JA, Fernandes P, Jorge S, Goncalves S, Alvarez A, Costa eSilva Z, Franca C, Prata MM: Acute kidney injury in intensive careunit patients: A comparison between the RIFLE and the AcuteKidney Injury Network classifications. Crit Care 12: R110, 2008

9. Joannidis M, Metnitz B, Bauer P, Schusterschitz N, Moreno R,Druml W, Metnitz PG: Acute kidney injury in critically ill patientsclassified by AKIN versus RIFLE using the SAPS 3 database.Intensive Care Med 35: 1692–1702, 2009

10. Macias-Nunez JF, Lopez-Novoa JM, Martinez-Maldonado M: Acuterenal failure in the aged. Semin Nephrol 16: 330–342, 1996

11. Jerkic M, Vojvodic S, Lopez-Novoa JM: The mechanism of in-creased renal susceptibility to toxic substances in the elderly. Int UrolNephrol 32: 539–547, 2001

12. Angus DC, Barnato AE, Linde-Zwirble WT, Weissfeld LA, WatsonRS, Rickert T, Rubenfeld GD, Robert Wood Johnson FoundationICU End-Of-Life Peer Group: Use of intensive care at the end of lifein the United States: An epidemiologic study. Crit Care Med 32:638–643, 2004

13. Pascual J, Liano F: Causes and prognosis of acute renal failure in thevery old. Madrid Acute Renal Failure Study Group. J Am Geriatr Soc46: 721–725, 1998

14. Akposso K, Hertig A, Couprie R, Flahaut A, Alberti C, Karras GA,Haymann JP, Costa De Beauregard MA, Lahlou A, Rondeau E, SraerJD: Acute renal failure in patients over 80 years old: 25-years’experience. Intensive Care Med 26: 400–406, 2000

15. Rosner MH: Acute kidney in the elderly: Pathogenesis, diagnosis andtherapy. Aging Health 5: 635–646, 2009

16. Choudhury D, Raj DSC, Palmer BF, Levi M: Effect of aging on renalfunction and disease. In: The Kidney, 6th Ed., edited by Brenner BM,Philadelphia, Saunders, 2000, pp. 2187–2216

17. Esposito C, Plati A, Mazzullo T, Fasoli G, De Mauri A, Grosjean F,Mangione F, Castoldi F, Serpieri N, Cornacchia F, Dal Canton A:Renal function and functional reserve in healthy elderly individuals.J Nephrol 20: 617–625, 2007

18. Wald F: The weight of normal adult human kidneys and its variabil-ity. Arch Pathol Lab Med 23: 493–500, 1937

19. Tauchi H, Tsuboi K, Okutomi J: Age changes in the human kidney ofthe different races. Gerontologia 17: 87–97, 1971

20. Goyal VK: Changes with age in the human kidney. Exp Gerontol 17:321–331, 1982

21. Kaplan C, Pasternack B, Shah H, Gallo G: Age-related incidence ofsclerotic glomeruli in human kidneys. Am J Pathol 80: 227–234,1975

22. Lindeman RD, Tobin J, Shock NW: Longitudinal studies on the rateof decline in renal function with age. J Am Geriatr Soc 33: 278–285,1985

23. Rowe JW, Andres R, Tobin JD, Norris AH, Shock NW: The effect ofage on creatinine clearance in men: A cross-sectional and longitudi-nal study. J Gerontol 31: 155–163, 1976

24. Fliser D, Zeier M, Nowack R, Ritz E: Renal functional reserve inhealthy elderly subjects. J Am Soc Nephrol 3: 1371–1377, 1993

25. Tan D, Cernadas MR, Aragoncillo P, Castilla MA, Alvarez ArroyoMV, Lopez Farre AJ, Casado S, Caramelo C: Role of nitric oxide-related mechanisms in renal function in ageing rats. Nephrol DialTransplant 13: 594–600, 1998

26. Prasad PV, Epstein FH: Changes in renal medullary pO2 duringwater diuresis as evaluated by blood oxygenation level-dependent

Table 4. General approaches to the prevention of AKI

Recognition of riskearly recognition of at-risk patients (use eGFR to detect

CKD)recognition of high-risk clinical settings (cardiovascular

surgery, ICU patients)use of risk scoring systems to estimate occurrence of

AKI in given settingAvoidance of nephrotoxin exposure

recognition of potential nephrotoxinsavoidance of concomitant use of multiple nephrotoxins�se of lowest dose and for shortest time periodif applicable, monitoring of drug levelsfrequent monitoring of renal functionmaintain euvolemia, hemodynamic instabilitydrug-specific measures (liposomal amphotericin, once-

daily aminoglycoside dosing)Avoidance of nosocomial infectionsExtracellular fluid expansion (especially for prophylaxis

of contrast exposure; maintain adequate urine output,stable hemodynamics)

Avoid agents that impair renal blood flow autoregulation(NSAIDS, ACEI, ARBs)

Use of computer surveillance systemsidentify high-risk patients and medicationsdetermine correct medication dose for eGFRdetect small changes in creatinine and alert caregivers

eGFR, estimated GFR; NSAID, nonsteroidal anti-inflammatory drug; ACEI,angiotensin-converting enzyme inhibitor; ARB, angiotensin receptor blocker.

56 Nephrology Self-Assessment Program - Vol 10, No 1, January 2011

Page 69: 2011.01 - Geriatric Nephrology

magnetic resonance imaging: Effects of aging and cyclooxygenaseinhibition. Kidney Int 55: 294–303, 1999

27. Qiao X, Chen X, Wu D, Ding R, Wang J, Hong Q, Shi S, Li J, XieY, Lu Y, Wang Z: Mitochondrial pathway is responsible for aging-related increase of tubular cell apoptosis in renal ischemia/reperfu-sion injury. J Gerontol A Biol Sci Med Sci 60: 830–839, 2005

28. Melk A, Ramassar V, Helms LM, Moore R, Rayner D, Solez K,Halloran PF: Telomere shortening in kidneys with age. J Am SocNephrol 11: 44–51, 2000

29. Melk A, Schmidt BM, Takeuchi O, Sawitzki B, Rayner DC, HalloranPF: Expression of p16INK4a and other cell cycle regulator andsenescence associated genes in aging human kidney. Kidney Int 65:510–519, 2004

30. Miura K, Goldstein RS, Morgan DG, Pasino DA, Hewitt WR, HookJB: Age-related differences in susceptibility to renal ischemia in rats.Toxicol Appl Pharmacol 87: 284–292, 1987

31. Beckman KB, Ames BN: The free radical theory of aging matures.Physiol Rev 78: 547–581, 1998

32. Ding G, Franki N, Kapasi AA, Reddy K, Gibbons N, Singhal PC:Tubular cell senescence and expression of TGF-beta1 and p21(WAFI/CIPI) in tubulointerstitial fibrosis of aging rats. Exp MolPathol 70: 43–53, 2001

33. Thum T, Hoeber S, Froese S, Klink I, Stichtenoth DO, Galuppo P,Jakob M, Tsikas D, Anker SD, Poole-Wilson PA, Borlak J, Ertl G,Bauersachs J: Age-dependent impairment of endothelial progenitorcells is corrected by growth-hormone-mediated increase of insulin-like growth factor-1. Circ Res 100: 434–443, 2007

34. Chen G, Bridenbaugh EA, Akintola AD: Increased susceptibility ofaging kidney to ischemic injury: Identification of candidate geneschanged during aging, but corrected by caloric restriction. Am JPhysiol Renal Physiol 293: F1272–F1281, 2007

35. Schmitt R, Marlier A, Cantley LG: Zag expression during agingsuppresses proliferation after kidney injury. J Am Soc Nephrol 19:2375–2383, 2008

36. Sugiura H, Yoshida T, Tsuchiya K, Mitobe M, Nishimura S, ShirotaS, Akiba T, Nihei H: Klotho reduces apoptosis in experimentalischemic acute renal failure. Nephrol Dial Transplant 20: 2636–2645, 2005

37. Do Amaral CL, Francescato HD, Coimbra TM, Costas RS, Darin JD,Antunes LM, Bianchi Mde L: Resveratrol attenuated cisplatin-in-duced nephrotoxicity in rats. Arch Toxicol 82: 363–370, 2008

38. Kassi E, Papvassiliou AG: Could glucose be a proaging factor? J CellMol Med 11: 1194–1198, 2008

39. Xu X, Bennett SA, Ingram RL, Sonntag WE: Deceases in growthhormone receptor signal transduction contribute to the decline ininsulin-like growth factor 1 gene expression with age. Endocrinology136: 4551–4557, 1995

40. Tran KT, Rusu SD, Satish L, Wells A: Aging-related attenuation ofEGF receptor signaling is mediated in part by increased proteintyrosine phosphatase activity. Exp Cell Res 289: 359–367, 2003

41. Chou JS, Reiser IW, Porush JG: Aging and urinary excretion ofepidermal growth factor. Am Clin Lab Sci 27: 116–122, 1997

42. Yu BP: Aging and oxidative stress: Modulation by dietary restriction.Free Radic Biol Med 21: 651–668, 1996

43. Wyndham J, Everitt A, Everitt S: Effect of isolation and foodrestriction begun at 50 days on the development of age-associatedrenal disease in the male Wistar rat. Arch Gerontol Geriatr 2:317–332, 1983

44. Tucker S, Mason R, Beauchene R: Influence of diet and feedrestriction on kidney function of aging male rats. J Gerontol 31:264–270, 1976

45. Allard JS, Perez E, Zou S, de Cabo R: Dietary activators of Sirt1. MolCell Endocrinol 299: 58–63, 2009

46. Dali-Youcef N, Lagouge M, Froelich S, Koehl C, Schoonjans K,Auwerx J: Sirtuins: The “magnificent seven,” function, metabolismand longevity. Ann Med 39: 335–345, 2007

47. Cohen HY, Miller C, Bitterman KJ, Wall NR, Hekking B, Kessler B,Howitz KT, Gorospe M, de Cabo R, Sinclair DA: Calorie restrictionpromotes mammalian cell survival by inducing the SIRT1 deacety-lase. Science 305: 390–392, 2004

48. de Jesussoaresqq T, Volpini RA, Francescato HD, Costa RS, da SilvaCG, Coimbra TM: Effects of resveratrol on glycerol-induced renalinjury. Life Sci 81: 647–656, 2007

49. Sebai H, Ben-Attia M, Sani M, Aouani E, Ghanem-Boughanmi N:Protective effect of resveratrol on acute endotoxemia-induced neph-rotoxicity in rat through nitric oxide independent mechanism. FreeRadic Res 42: 913–920, 2008

50. Rosner MH: Urinary biomarkers for the detection of renal injury. AdvClin Chem 49: 73–97, 2009

51. Haase M, Bellomo R, Devarajan P, Schlattmann P, Haase-Fielitz A:Accuracy of neutrophil gelatinase-associated lipocalin (NGAL) indiagnosis and prognosis in acute kidney injury: A systematic reviewand meta-analysis. Am J Kidney Dis 54: 1012–1024, 2009

52. Xue JL, Daniels F, Star RA, Kimmel PL, Eggers PW, Molitoris BA,Himmelfarb J, Collins AJ: Incidence and mortality of acute renalfailure in Medicare beneficiaries, 1992 to 2001. J Am Soc Nephrol 17:1135–1142, 2006

53. Waikar SS, Curhan GC, Wald R, McCarthy EP, Chertow GM:Declining mortality in patients with acute renal failure: 1988 to 2002.J Am Soc Nephrol 17: 1143–1150, 2006

54. Van Den Noortgate N, Mouton V, Lamot C, Van Nooten G, DhondtA, Vanholder R, Afschrift M, Lameire N: Outcome in a post-cardiacsurgery population with acute renal failure requiring dialysis: Doesage make a difference? Nephrol Dial Transplant 18: 732–736, 2003

55. Coca SG: Acute kidney injury in elderly persons. Am J Kidney Dis56: 122–131, 2010

56. Ishani A, Xue JL, Himmelfarb J, Eggers PW, Kimmel PL, MolitorisBA, Collins AJ: Acute kidney injury increases risk of ESRD amongthe elderly. J Am Soc Nephrol 20: 223–228, 2009

57. Schmitt R, Coca S, Kanbay M, Tinetti ME, Cantley LG, Parikh CR:Recovery of kidney function after acute kidney injury in the elderly:A systematic review and meta-analysis. Am J Kidney Dis 52: 262–271, 2008

58. Ponte B, Felipe C, Muriel A, Tenorio MT, Liano F: Long-termfunctional evolution after an acute kidney injury: A 10-year study.Nephrol Dial Transplant 23: 3859–3866, 2008

59. Wald R, Quinn RR, Luo J, Li P, Li P, Scales DC, Mamdani MM, RayJG, University of Toronto Acute Kidney Injury Research Group:Chronic dialysis and death among survivors of acute kidney injuryrequiring dialysis. JAMA 302: 1179–1185, 2009

60. Ishani A, Xue JL, Himmelfarb J, Eggers PW, Kimmel PL, MolitorisBA, Collins AJ: Acute kidney injury increases risk of ESRD amongelderly. J Am Soc Nephrol 20: 223–228, 2009

61. Noble JS, Simpson K, Allison ME: Long-term quality of life andhospital mortality in patients treated with intermittent or continuoushemodialysis for acute renal and respiratory failure. Ren Fail 28:323–330, 2006

62. Ahlstrom A, Tallgreen M, Peltonen S, Rasanen P, Pettila V: Survivaland quality of life of patients requiring acute renal replacementtherapy during intensive care. Intensive Care Med 31: 1222–1228,2005

63. Gopal I, Bhonagiri S, Ronco C, Bellomo R: Out of hospital outcomeand quality of life in survivors of combined acute multiple organ andrenal failure treated with continuous venovenous hemofiltration/hemodiafiltration. Intensive Care Med 23: 766–772, 1997

Therapeutic Options for Older Individuals withCKD

Traditionally, renal replacement therapy foradults with chronic kidney disease (CKD), regardless

Nephrology Self-Assessment Program - Vol 10, No 1, January 2011 57

Page 70: 2011.01 - Geriatric Nephrology

of age, has comprised the three modalities transplan-tation, hemodialysis (HD), and peritoneal dialysis(PD). More recently, however, there is an increasedappreciation that nondialysis renal care is also animportant therapeutic option that should be disclosedto all patients and their families (1,2).

Older patients with advanced stages of CKDshould have a baseline comprehensive geri-atric assessment done as part of their pre-dialysis care in a multidisciplinary clinic set-ting to identify geriatric syndromes early.Predialysis education should include de-tailed information about nondialysis care,particularly for vulnerable seniors.

With this in mind, particularly in older individ-uals, it is helpful to think of therapy as being eitherhome based or in-center based rather than being mo-dality specific. Home-based therapies are preferred (3)and include treatments such as kidney transplantation,nocturnal HD, intermittent daily or thrice-weekly HD,PD, and nondialysis renal care. Collectively, thesemodalities promote independence and personal auton-omy across a range of abilities and, with the exclusionof transplantation, across a range of medical condi-tions. The choice of treatment, although often left to

the patient and his or her family, is largely influencedby the individual’s comorbidities and any resultingfunctional and cognitive impairments. Early referraland assessment in a predialysis multidisciplinary clinicsetting are associated with improved outcomes. Al-though the treatments themselves are directed at themanagement of renal disease and ESRD-related symp-toms, patients who are on dialysis experience an in-creased burden of nonrenal symptoms. These symp-toms, together with morbidity and mortality data, arediscussed in more detail in the subsequent subsections.

Geriatric Syndromes in Elderly Patients withCKD

Within geriatric medicine, it is well recognizedthat disease, coupled with social and environmentalfactors, predisposes to a host of common syndromes.These syndromes include accidental falls, dementiaor cognitive impairment, functional dependency, andfrailty (see Table 5 for common definitions). Multiplestudies have demonstrated high rates of frailty, falls,functional impairment, and cognitive impairment indialysis patients (4–10). Frailty, for example, is seenmore commonly in those with CKD than in those withnormal renal function (15 versus 6%, respectively) (5),whereas dialysis patients have been shown to experi-ence one of the highest prevalence rates for frailty inany single population (78.8% of those older than 80

Table 5. Common definitions for terms commonly used in geriatric medicine

Syndrome Common Definitions

Frailty (108) The presence of three of five criteria: Unintentional weight loss, self-reported exhaustion, slowgait speed, weakness (measured using a hand grip), and low physical activity

Falls (109) Defined as an event that results in the patient’s coming to rest inadvertently on the ground orother lower level. Injurious falls are defined as those that cause minor (e.g., cuts, bruises) ormajor injuries (e.g., fractures, loss of consciousness, hospitalization, death).

Dementia (110) A decline in memory from previously higher levels of functioning together with one of thefollowing manifestations: Aphasia, apraxia, agnosia, or disturbances in executivefunctioning. A diagnosis of dementia cannot be made if deficits occur exclusively during thecourse of a delirium. Impairment must be severe enough to affect the patient’s ability tofunction in his or her work, personal, or social environment. Milder degrees of impairment,which may not affect day-to-day living, are sometimes termed mild cognitive impairment.

Functional impairmentand disability (111)

Defined as a restriction in the ability to perform basic actions in daily life, such toileting andbathing, walking, transferring, feeding and control of both bowel and bladder functions(known as activities of daily living), and activities required for independent householdmanagement, such as meal preparation, transportation, telephone use, shopping,housecleaning, laundry, and management of medications and finances (known asinstrumental activities of daily living). When functional limitations affect the whole body,disability occurs, reflecting difficulty, limitation, or inability to perform in social roles.

58 Nephrology Self-Assessment Program - Vol 10, No 1, January 2011

Page 71: 2011.01 - Geriatric Nephrology

years were considered frail) (9). Older dialysis patientsexperience more accidental falls than would be ex-pected in age-matched populations (6,11,12), have anoverall lower level of independent functioning(10,13,14), and have increased burden of cognitiveimpairment (4,7,8). As with frailty, falls were associ-ated with increased mortality over time (9,15). Muchof the functional decline occurs around the time ofdialysis initiation (16,17), with additional declinelikely with each subsequent hospitalization (18). In-tervention and prevention of these clinical syndromeslikely are best approached by increased awareness andthe introduction of routine comprehensive geriatricassessments (19).

Renal Replacement Therapies for OlderIndividualsHome-Based Therapies: Home HD, PD, andNocturnal Dialysis As with younger patients, bothHD and PD can be offered to patients in their homesetting, provided that they have access to clean waterand a basic level of hygiene. PD is the more com-monly used home dialysis modality (in patients aged�65 years, PD utilization rates are 18% in Canada[20], 6% in the United States [21], and 16% in theUnited Kingdom [22]), although there has been anincrease in the number of older patients being acceptedfor home nocturnal dialysis programs and home HDprograms in recent years. Initial concerns that olderpatients on PD still experience an increased risk forperitonitis have not been borne out in large observa-tional studies (23,24). PD peritonitis rates have de-creased in the past decade, particularly in patients aged�70 years. This observation is attributed to advancesin PD connectology and increased use of topical anti-bacterial ointments.

Both patients and their families face considerablechallenges when considering home dialysis as a long-term therapy. These include barriers within theirhomes, such as storage space and electrical wiring, andthose arising from their social circumstances and alsodifficulties related directly to their own physical andmental well-being. Although many of the challengesfaced are similar for both HD and PD, barriers to homedialysis have been most thoroughly studied from thehome PD perspective. In a prospective cohort study of497 Canadian patients of all ages, only 22% of patientshad absolute medical or social contraindications toPD, suggesting that the majority of patients in our

clinical practice are eligible for home dialysis therapy(25). Of those who were believed to be suitable forPD, the majority (63%) had at least one physical orcognitive barrier to PD, with the most commonlyappreciated barriers being decreased strength and mo-bility, decreased manual dexterity, and visual loss(25). Older female patients were most likely to havemultiple reasons precluding PD use, whereas thosewith family support were most likely to receive PDdespite having one or more barriers. Supportive careand assisted dialysis programs (discussed in the latersections) may help overcome many of these barriers.Hospital- or Center-Based Therapy: HD and Noc-turnal Dialysis In-center HD is the most commonlyused treatment for older patients with ESRD. Appre-ciating that, in 2003 alone, �13,000 patients aged�80 years were started on dialysis in the United States(26), this is an area with maximal growth, and evensmall improvements in the use of home dialysis wouldhave a significant impact on these numbers. Althoughin-center therapies include intermittent HD, intermit-tent PD, and more recently nocturnal dialysis, inter-mittent PD is rarely recommended for long-term use.

In the Dialysis Outcomes and Practice PatternsStudy (DOPPS), the mean age of participants on HDincreased over time in all parts of the world, with datafrom DOPPS III showing that anywhere between 18 and40% of prevalent patients were aged �75 years (B.Canaud, L. Tong, F. Tentori, T. Akiba, T. Akizawa, R.L.Pisoni, J. Bommer, and F.K. Port, submitted). Europeancountries such as France, Belgium, and Sweden seem tohave higher prevalent rates than the United States forpatients aged �75 years, perhaps reflective of the lon-gevity seen in Europe at large. Although there is clearlyvariability in how dialysis care is delivered, the careprovided to older patients mostly reflects the variablepractice patterns seen across the world. For example,although the relative use of semipermanent dialysis-tunneled catheters is highest in individuals aged �75years in Europe and Australia/New Zealand (Eur/ANZ;relative risk 1.6-fold higher usage when compared withyounger patients), overall catheter usage is still lowerthan that seen in North America (22 and 29% in Eur/ANZ and North America, respectively) (B. Canaud, L.Tong, F. Tentori, T. Akiba, T. Akizawa, R.L. Pisoni, J.Bommer, and F.K. Port, submitted).

Age-specific problems include the high rate offistulas that do not mature (27), the high rate ofhospitalizations (28), and the observation that older

Nephrology Self-Assessment Program - Vol 10, No 1, January 2011 59

Page 72: 2011.01 - Geriatric Nephrology

patients are more likely to have low serum albumenand low serum phosphate levels, suggesting a higherrate of malnutrition (B. Canaud, L. Tong, F. Tentori,T. Akiba, T. Akizawa, R.L. Pisoni, J. Bommer, andF.K. Port, submitted). DOPPS data have shown thatolder in-center HD patients are at increased risk fordepression and adynamic bone disease (defined asparathyroid hormone levels �150 pg/ml) comparedwith those younger than 75 years (B. Canaud, L. Tong,F. Tentori, T. Akiba, T. Akizawa, R.L. Pisoni, J.Bommer, and F.K. Port, submitted). Dialysis prescrip-tions and guideline adherence rates seem to be similaracross all age groups.

Nocturnal HD is an emerging dialysis modality.It is not widely funded and therefore is used only inselect centers. As a result, there is only limited expe-rience with performing in-center nocturnal HD, par-ticularly in older patients (29). In the future, weanticipate that this may be more commonly used innursing home settings, where patients are alreadyresident, and the additional cost associated with over-night staffing is less of an issue.

Assistive Programs and Dialysis in NursingHomes Older patients are more likely than youngerpatients to require help with personal care or managingtheir dialysis or require transfer to a nursing homewhen on dialysis. Of those who do not require full carebut need some assistance, visiting nurses can betrained to provide dialysis-specific help (assisted dial-ysis), particularly for patients who are on home-basedPD (30–34). These visiting nurses help with bagexchanges, exit-site care, monitoring of BP andweights, and connections or disconnections to thecycler machine (30). Although nursing salaries addsignificantly to the cost of providing PD at home,these costs are variable and often still substantially lessthan providing in-center HD care (35).

Patients who have acute kidney injury and sub-sequently require nursing home care and those whoinitiate dialysis while residing in a nursing home haveguarded outcomes (16,36). A recent study showed thatindividuals who initiated dialysis while residing in anursing home had poor survival and almost invariablyexperienced accelerated functional decline (16). Thosewho transfer to a nursing home, when well establishedon dialysis, fare better than those who initiate dialysiswhile in a nursing home despite similar attendantcomorbidities, suggesting that the process of dialysis

initiation is when elderly individuals are at their mostvulnerable (36).

The provision of on-site dialysis care for patientswho reside in a nursing home may be limited by thecost-effectiveness of treating only small numbers ofpatients at any one time and by the experience andwillingness of nursing home staff to assist with dial-ysis care. Notwithstanding these issues, nursing homedialysis units have been effectively run (37–40). Casereports of on-site HD units, staffed by experienceddialysis technicians under the supervision of an off-site nephrologist (37), have demonstrated feasibility.Similarly, PD services have been provided by a ded-icated dialysis nurse (41,42) or by nursing home staffafter extra training (39,43). Refresher courses andongoing education for the nursing home is vital tomaintaining skill competency, and although peritonitisrates tend to be higher in nursing home residents, thisis likely due, in part, to the patients’ high levels ofconcomitant comorbidity.Nondialysis Care as an Active Treatment Strat-egy There is an increasing appreciation that long-term dialysis therapy is a burdensome treatment.Dietary restrictions, the time commitment for dial-ysis therapy, and the fluctuations in the feeling ofwell-being have a great impact on an individualpatient’s quality of life (QoL). In addition, non–ESRD-specific symptoms such as pain, fatigue, anddecreased strength are common, particularly inolder individuals (44). Despite high-quality dialysiscare and guideline adherence, many of the non–ESRD-specific symptoms persist even after therapy is estab-lished, reducing the overall QoL for the individual ondialysis. Recognizing this, there is a strong trend forthose involved in geriatric nephrology care to ad-vocate for improved access and increased use ofnondialysis renal care (1,2).

Nondialysis renal care has been offered in anumber of centers across the world, but it is notwidespread or often promoted as a “treatment” strat-egy (45–50). A set of interventions and medicationscan be used to minimize symptoms and to promotegood living rather than lengthy living (discussed in thePalliative Care and Geriatric Treatment of Patientswith Advanced Chronic Kidney Disease segment).Centers that have well-developed programs havefound that when presented as an alternative treatmentstrategy to patients and families, a substantial propor-tion choose not to undergo dialysis. In one series,

60 Nephrology Self-Assessment Program - Vol 10, No 1, January 2011

Page 73: 2011.01 - Geriatric Nephrology

patients underwent multidisciplinary team assessmentas part of their routine predialysis care (45). The teamcollectively discussed concerns and made recommen-dations favoring either dialysis or nondialysis care.During a period of 4 years, 19% of patients evaluatedwere advised to consider nondialysis care. Of thosewho were advised to consider nondialysis care, only26% opted to undergo dialysis (although a substantialproportion of both those who wanted dialysis andthose who opted for no dialysis either did not progressor died before they would have had a need for dialy-sis). Although initial data suggested that the survivalafter dialysis initiation was similar to the survival ofthose who opted for nondialysis care (approximately9-month survival), this was likely limited by themethod used to determine an equivalent time pointfrom which survival was measured in the two groups(45). Subsequent studies, which used different meth-ods to calculate a putative dialysis date, suggested amodest survival benefit with dialysis that may bepartly counterbalanced by an increased rate of hospi-talizations and reduced likelihood of dying at homewith family or friends (46,47). Of interest, in the mostrecent study of 29 patients who opted for nondialysiscare, a number of patients who did not undergo dial-ysis survived a number of months after their putativedialysis date, suggesting that the putative dialysis startis likely not predictable from a serum creatinine esti-mate or eGFR (51). In addition, factors such as ageand comorbidity burden are poor surrogates for iden-tifying patients whom the multidisciplinary team be-lieve are best suited to nondialysis care (47,52). Thereader is referred to the Palliative Care and GeriatricTreatment of Patients with Advanced Chronic KidneyDisease for further discussion of nondialysis medicaltherapy.Transplantation Patients who are aged �65 yearsare the least likely of all age groups to undergo kidneytransplantation, partly because of medical unsuitabilityand the ongoing debate regarding the use of scarceorgans in those with limited lifespan. There is, how-ever, increasing debate about how much the underuseof transplantation is attributable to subtle physicianand patient biases (28,53). Older patients are lessfrequently placed on the waiting list and are morelikely to be offered kidneys of extended-donor criteria.In a small cohort of 113 patients who initiated dialysis,Kiberd et al. (54) suggested that age was one of themost common reasons for nonreferral of patients who

seemed otherwise suitable for transplantation, whereasdata from Scotland suggested that age was an impor-tant determinant of transplant waiting list status (55).In the United States, older Caucasian patients are athigher risk for death while on the waiting list thanthose who are younger (56), reflecting the increasedtime to an available organ and the relatively highburden of comorbidity (28). One third of all kidneytransplants in individuals aged �70 years in theUnited States were kidneys of extended-donor criteria(57). Overall survival outcomes seem good with or-gans of extended-donor criteria; however, there isconcern that, over time, the more commonplace use oforgans of extended-donor criteria may result in de-creased graft survival. In Europe, a formal programpromoting the use of organs from older, marginaldonors in recipients aged �65 years (the so-called“old for old” program) has seen good outcomes and anincrease in the numbers of kidney transplantation sur-geries done; however, the absolute number of organsprocured still remains less than that in other parts ofthe world (58,59).

Guidelines for transplantation evaluation aresimilar regardless of age (60,61). Some advisory bod-ies suggest more aggressive screening for cancers,although this is not widespread practice. After trans-plantation, immunosuppression protocols are similarto that for younger individuals, with much variabilityacross centers and across the world (62–64). Acuterejection tends to be less frequent in most but not allstudies, whereas morbidity and mortality associatedwith sepsis are 20 to 40% higher than in youngerpatients (65–68).

Outcomes with and without Renal ReplacementTherapiesSurvival In comparison with dialysis, a functioningkidney transplant offers improved QoL at a lower cost.Mechanistic comparisons of patients who are on dial-ysis and those who undergo transplantation demon-strated that transplantation also affords a better overallsurvival than dialysis, despite increased mortality risksat the time of the transplant surgery. In the UnitedStates, 1-, 3-, and 5-year patient survival with trans-plantation is 90.8, 77.0, and 62.1% for those aged �65years (28). When compared with dialysis patients onthe waiting list, the adjusted relative risk (RR) fordeath is 0.46 (95% confidence interval [CI] 0.34 to0.61). In the initial perioperative period, patients are at

Nephrology Self-Assessment Program - Vol 10, No 1, January 2011 61

Page 74: 2011.01 - Geriatric Nephrology

threefold higher risk for death; however, this risk fallssteadily, and by 125 days, the risks are equivalent. Asexpected, the relative benefits of transplantation overcontinued dialysis decrease as age increases such thatindividuals aged 70 to 74 years have a 42% lowermortality (RR 0.58; 95% CI 0.52 to 0.65), whereasthose aged �75 years have a 33% lower risk (RR 0.67;95% CI 0.53 to 0.86) (57,69). Ideally, older patientsshould undergo transplantation preemptively or within2 years of dialysis initiation, not only to maximizesurvival and QoL gains but also to provide a reason-able cost-effective treatment. Delays beyond 2 yearsare associated with drastically increased costs. Forexample, for a patient aged 75 years, the cost perquality-adjusted life year goes from $99,593 to$305,017 if the time on the waiting list increases from2 to 4 years (70). Both registry reports and single-center case reports suggested that death-censored graftsurvival is compatible with, if not superior to, that seenin younger patients (71–76). Recent data showed 83.0,74.1, and 64.1% graft survival at 1, 3, and 5 years,respectively, with a deceased-donor organ and 94.3,88.8, and 72.3% with living-donor organs (77).

With dialysis, 1-year survival, for those whoinitiate dialysis in the United States when aged �75years is 54%; in Canada, 1-, 3-, and 5-year survivalrates are 69.0, 36.7, and 20.3%, respectively (26,78).International comparisons suggest that the unadjustedmortality rate is lowest in Japan and highest in NorthAmerica; however, further interpretation is limited bylarge differences in acceptance rates and the preva-lence of cardiovascular disease (B. Canaud, L. Tong,F. Tentori, T. Akiba, T. Akizawa, R.L. Pisoni, J.Bommer, and F.K. Port, submitted). Most survivaldata are derived from national renal registries andtherefore criticized for having limited comorbidityinformation. There are also widespread differences incriteria used to determine which patients are acceptedonto dialysis programs that may lead to selectionbiases and survival differences. Because cohorts arebased on registry information, they are subject to all ofthe weaknesses associated with using nonrandomized,observational data.

A large body of literature compares survival onHD and PD using various methods (69,79–90). Thereis much controversy around the interpretation of theresults. Those who favor HD often interpret the data asshowing that PD has poorer survival, whereas propo-nents of PD cite the data as showing superior survival

with PD in the first 1 to 2 years, with decreasedsurvival (often attributed to loss of residual renalfunction) only in later years. Despite the limitationsand varying results, most recent high-quality studiessuggested that the benefits or risks attributable todialysis modality may depend on how long a patienthas been treated for ESRD (85,86,90). Collectively,the data, particularly those specific to individuals aged�65 years, suggested that mortality is similar for thefirst 12 to 24 months but approximately 20% higher inthe second or third year in patients who are maintainedon PD (85,86,90). Interpretations vary, but in theopinion of this author, when considered together withthe relatively modest life expectancy associated withdialysis in older age groups, the impact of thesedifferences is small and may amount to a gain in lifeof only 1 to 2 months.

Survival is significantly affected by late refer-ral for nephrology assessment, higher comorbidity,and functional status (26,28,48). In many olderpatients, mortality seems to be highest at the onsetof dialysis, with North American studies showing a24 to 26% mortality rate in the first 3 months invulnerable groups of patients (16,17). In their sem-inal paper, Kurella Tamura et al. (16) followed�3702 nursing home residents who initiated dialy-sis. Fewer than half survived �9 months, with a42% 1-year survival. In a recent DOPPS study ofpatients aged �75 years at the time of dialysisinitiation, high numbers of concomitant comorbidconditions predicted poorer outcomes, although theauthors did not find diabetes to be associated withincreased mortality (B. Canaud, L. Tong, F. Tentori,T. Akiba, T. Akizawa, R.L. Pisoni, J. Bommer, andF.K. Port, submitted).Quality of Life Previous studies that followed pa-tients through the transition from dialysis to transplan-tation showed that kidney transplantation offers supe-rior QoL compared with dialysis (91). Values aresimilar to that of the general public. In contrast, dialysispatients have significantly lower scores on well-beingand QoL assessments. Three studies focused on QoLevaluation of older individuals (83,92,93). All sug-gested that the main reason for poor QoL scores is dueto the changes in the physical well-being, with mark-edly less impact on mental health scores. Data from391 prevalent HD patients who were aged �70 yearsand recruited to the Hemodialysis (HEMO) studyshowed similar physical and mental component scores

62 Nephrology Self-Assessment Program - Vol 10, No 1, January 2011

Page 75: 2011.01 - Geriatric Nephrology

on the SF-36 questionnaire to that of younger partic-ipants (93). Mental scores were similar to that of thegeneral population (mean score 50 � 10), whereasphysical scores were lower. Few changes (either dec-rements or improvements) were seen in QoL scoresover time. The North Thames Dialysis Study of prev-alent patients who were older than 70 years alsoshowed that patients had lower overall scores com-pared with age-matched norms and that this waslargely due to changes in physical well-being (physicalcomponent scores 34 � 11 versus 41 � 12 and mentalcomponent scores 51 � 11 versus 53 � 9, respec-tively, for dialysis patients compared with UK generalpopulation norms) (83,92). Data from a recent DOPPSis the first to suggest that the QoL of older HDpatients, as measured by the Kidney Disease Qualityof Life Survey (KDQOL), is lower in those aged �75years than in younger patients (B. Canaud, L. Tong, F.Tentori, T. Akiba, T. Akizawa, R.L. Pisoni, J. Bom-mer, and F.K. Port, submitted). This may in part relateto the fact that this DOPPS included incident patientsrather than prevalent patients.

Modality-specific QoL studies are often con-founded by the characteristics of the patients them-selves. HD patients are often believed to have ahigher burden of disease and therefore can be ex-pected to have poorer QoL. In the BroadeningOptions for Long-term Dialysis in the Elderly(BOLDE) study, comparisons were made aftermatching prevalent patients who were aged �65years and on PD with those who were on HD usingdemographic criteria. Matching was not done oncomorbidity. QoL scores seemed higher (better) inthose who were on PD even after correction fordifferences in comorbidity (94). Although PD pa-tients also showed significantly lower illness intru-sion and depression burden, it is unclear whetherthere is residual confounding even after correctionfor differences in comorbidity, and the authors onlyconcluded that PD is associated with at least as gooda QoL as HD.Associated Morbidity Many cross-sectional stud-ies of prevalent patients have suggested that olderdialysis patients have a high burden of nonrenalsymptoms and impairments. Hospitalizations arecommon, with US Renal Data System data showinga mean of two hospitalizations per year for thoseaged �65 years (28). In addition to the impact ofillness and the cost to the health care systems,

hospitalizations in and of themselves are associatedwith an increased risk for physical dependence,particularly for older patients (95–98). In a single-center cohort study of prevalent dialysis patients,fewer than 5% of patients were fully independent inall aspects of personal function. In the same studycohort, 95.6% had visual acuity levels less thanage-expected values, suggesting a high rate of sen-sory impairment (10,99). Geriatric syndromes suchas recurrent accidental falls and cognitive impair-ment are common, with prevalence estimates sug-gesting that up to 37% of all HD patients, regardlessof age, have severe cognitive impairment (7,100).Almost 80% of HD patients who were studied aspart of the US Renal Data System Dialysis Mortalityand Morbidity Study (DMMS) Wave 2 and wereaged �80 years more fulfilled the criteria for clin-ical frailty, whereas �90% were noted to have lowscores on the RAND-36 physical function score(9,100). Frailty was associated with increased mor-tality and hospitalization even after adjustment forcommonly measured characteristics such as age,gender, and comorbidity. Using a modified frailtyphenotype score, which incorporated low physicalfunctioning, inactivity, and undernutrition, the in-creased hazard of mortality was estimated at 1.87per unit increase in frailty score (95% CI 1.59 to2.20).

Much of this morbidity seems to occur aroundthe time of dialysis initiation. In an administrativedatabase study of 3702 patients who resided in anursing home and initiated dialysis, there was anaccelerated rate of functional decline shortly beforeand continuing after dialysis initiation (16). Moreconcerning was the observation that many patientsdied and few, if any, regained their predialysisfunctional level even after 6 or 12 months. Althoughthis study was limited to individuals who alreadyhad some degree of disability that required nursinghome care, functional decline at the time of dialysisis likely not limited to only frail, vulnerable elders.In a single-center observational study of healthyoctogenarians, �30% of individuals experiencedsignificant functional decline in the first 6 monthsafter dialysis start (17). Unlike those who resided ina nursing home, however, the number of individualswho remained at home (or who died before func-tional decline) remained stable after this period.

Nephrology Self-Assessment Program - Vol 10, No 1, January 2011 63

Page 76: 2011.01 - Geriatric Nephrology

Miscellaneous Issues Relating to theTherapeutic Management of CKD in OlderPatients

Dialysis guidelines rarely distinguish betweengoals of care for older patients and those for youngerpatients. Vascular access guidelines encourage the useof an arteriovenous fistula as the vascular access ofchoice even in older individuals. However, some (butnot all) studies suggested that older patients have highrates of poor fistula maturation. Because older patientshave lower life expectancy, not all patients may ben-efit from undergoing fistula creation (101). Definitiverecommendations cannot be made, but it is appropriateto consider the anticipated life expectancy of thepatient when referring for access creation.

The Renal Physicians Association and the Amer-ican Society of Nephrology advocate for all patients tohave clearly documented advance directives regardlessof age (102). This is imperative for older patients, whoare at increased risk for disabling events. Although itis not uncommon for physicians to feel inadequatelytrained to facilitate palliative care and advance careplanning interviews, it is an important part of provid-ing comprehensive care to the older patient. Thereader is referred to an article by Davison et al. (103)for an example of questions that may be used wheninterviewing patients.

Unintentional weight loss is a striking predictorof outcome in both dialysis and nondialysis patients.Thus, recent studies showing dietary manipulation asan effective method to delay dialysis need to bevalidated in larger populations. Data showing thatsupplementation with bicarbonate may delay renalprogression are promising (104,105). Although thesetwo studies included a limited number of older patientsand therefore must be interpreted with caution, a lowrate of fluid overload or malnutrition was seen, sug-gesting that it may be a simple and cost-effectivetreatment for CKD management. Dietary protein re-striction has also been shown to be a cost-effectiveway to postpone dialysis (106,107). In a small, un-blinded, randomized, controlled trial, 56 patients wererandomly assigned to a vegan diet with a calorie intakeof 35 kcal/kg and protein intake of 0.3g/kg body wtand compared with patients who were starting dialysis.Patients were followed for 48 months, during whichthey did not experience a fall in serum albumin levels,protein catabolic rate, or body mass index (107).Although promising, further studies that confirm that

dietary restriction over the long term does not result inweight loss or malnutrition or predispose to frailty areneeded.

The significantly increased incidence of coloncancer in older individuals is important when treat-ing older patients with CKD. Clinicians should havea high index of suspicion for occult colon cancer inolder individuals when they note unintentionalweight loss or when patients require frequent intra-venous iron infusions (including maintenancedoses). It is often useful to refer to their advancedirectives to evaluate whether colonoscopy is war-ranted under these circumstances.

References1. Arnold RM, Zeidel ML: Dialysis in frail elders: A role for palliative

care. N Engl J Med 361: 1597–1598, 20092. Jassal SV, Watson D: Doc, don’t procrastinate: Rehabilitate, palli-

ate, and advocate. Am J Kidney Dis 55: 209–212, 20103. Ahmed S, Addicott C, Qureshi M, Pendleton N, Clague JE, Horan

MA: Opinions of elderly people on treatment for end-stage renaldisease. Gerontology 45: 156–159, 1999

4. Kurella M, Chertow GM, Luan J, Yaffe K: Cognitive impairment inchronic kidney disease. J Am Geriatr Soc 52: 1863–1869, 2004

5. Shlipak MG, Stehman-Breen C, Fried LF, Song X, Siscovick D,Fried LP, Psaty BM, Newman AB: The presence of frailty in elderlypersons with chronic renal insufficiency. Am J Kidney Dis 43:861–867, 2004

6. Desmet C, Beguin C, Swine C, Jadoul M: Falls in hemodialysispatients: Prospective study of incidence, risk factors, and compli-cations. Am J Kidney Dis 45: 148–153, 2005

7. Cook WL, Tomlinson G, Donaldson M, Markowitz SN, Naglie G,Sobolev B, Jassal SV: Falls and fall-related injuries in older dialysispatients. Clin J Am Soc Nephrol 1: 1197–1204, 2006

8. Murray AM, Tupper DE, Knopman DS, Gilbertson DT, PedersonSL, Li S, Smith GE, Hochhalter AK, Collins AJ, Kane RL: Cogni-tive impairment in hemodialysis patients is common. Neurology 67:216–223, 2006

9. Johansen KL, Chertow GM, Jin C, Kutner NG: Significance offrailty among dialysis patients. J Am Soc Nephrol 18: 2960–2967,2007

10. Cook WL, Jassal SV: Functional dependencies among the elderly onhemodialysis. Kidney Int 73: 1289–1295, 2008

11. Cook WL, Jassal SV: Prevalence of falls amongst seniors main-tained on hemodialysis. Int Urol Nephrol 37: 649–652, 2005

12. Dukas LC, Schacht E, Mazor Z, Stahelin HB: A new significant andindependent risk factor for falls in elderly men and women: A lowcreatinine clearance of less than 65 ml/min. Osteoporos Int 16:332–338, 2005

13. Ifudu O, Paul H, Homel P, Friedman EA: Predictive value offunctional status for mortality in patients on maintenance hemodi-alysis. Am J Nephrol 18: 109–116, 1998

14. Fried L, Lee JS, Shlipak MG, Chertow GM, Green C, Ding J, HarrisT, Newman AB: Chronic kidney disease and functional limitation inolder people: Health, Aging and Body composition study. J AmGeriatr Soc 54: 750–756, 2006

15. Li M, Tomlinson G, Naglie G, Cook WL, Jassal SV: Geriatriccomorbidities, such as falls, confer an independent mortality risk toelderly dialysis patients. Nephrol Dial Transplant 23: 1396–1400,2008

16. Kurella Tamura M, Covinsky KE, Chertow GM, Yaffe K, Landefeld

64 Nephrology Self-Assessment Program - Vol 10, No 1, January 2011

Page 77: 2011.01 - Geriatric Nephrology

CS, McCulloch CE: Functional status of elderly adults before andafter initiation of dialysis. N Engl J Med 361: 1539–1547, 2009

17. Jassal SV, Chiu E, Hladunewich MA: Loss of independence inpatients starting dialysis at 80 years of age or older. N Engl J Med361: 1612–1613, 2009

18. Lo D, Chiu E, Jassal SV: A prospective pilot study to measurechanges in functional status associated with hospitalization in el-derly dialysis-dependent patients. Am J Kidney Dis 52: 956–961,2008

19. American Society of Nephrology: Geriatric Nephrology Curriculum2009. Available at: www.asn-online.org/education_and_meetings/geriatrics. Accessed September 15, 2010

20. Canadian Institute for Health Information: Treatment of End-StageOrgan Failure in Canada, 1999 to 2008—CORR 2010 AnnualReport, Ottawa, Ontario, Canada: Canadian Institute for HealthInformation, 2010

21. US Renal Data System: 2009 ADR v3 Reference Tables page 85.Available at: www.usrds.org/2009/fb/INDEX_REF.HTML. Ac-cessed May 2010

22. The UK Renal Registry: UK ESRD incident rates in 2008: Nationaland centre-specific analyses. Available at: www.renalreg.com/Report-Area/Report%202009/Chap03_Renal09_web.pdf. AccessedMay 2010

23. Nessim SJ, Bargman JM, Austin PC, Nisenbaum R, Jassal SV:Predictors of peritonitis in patients on peritoneal dialysis: Results ofa large, prospective Canadian database. Clin J Am Soc Nephrol 4:1195–1200, 2009

24. Nessim SJ, Bargman JM, Austin PC, Story K, Jassal SV: Impact ofage on peritonitis risk in peritoneal dialysis patients: An era effect.Clin J Am Soc Nephrol 4: 135–141, 2009

25. Oliver MJ, Garg AX, Blake PG, Johnson JF, Verrelli M, ZachariasJM, Pandeya S, Quinn RR: Impact of contraindications, barriers toself-care and support on incident peritoneal dialysis utilization.Nephrol Dial Transplant 25: 2737–2744, 2010

26. Kurella M, Covinsky KE, Collins AJ, Chertow GM: Octogenariansand nonagenarians starting dialysis in the United States. Ann InternMed 146: 177–183, 2007

27. Lok CE, Oliver MJ, Su J, Bhola C, Hannigan N, Jassal SV:Arteriovenous fistula outcomes in the era of the elderly dialysispopulation. Kidney Int 67: 2462–2469, 2005

28. US Renal Data System: 2010 Atlas of CKD & ESRD. Available at:www.usrds.org/. Accessed November 4, 2010

29. Bugeja A, Dacouris N, Thomas A, Marticorena R, McFarlane P,Donnelly S, Goldstein M: In-center nocturnal hemodialysis: An-other option in the management of chronic kidney disease. ClinJ Am Soc Nephrol 4: 778–783, 2009

30. Oliver MJ, Quinn RR, Richardson EP, Kiss AJ, Lamping DL,Manns BJ: Home care assistance and the utilization of peritonealdialysis. Kidney Int 71: 673–678, 2007

31. Povlsen JV, Ivarsen P: Assisted automated peritoneal dialysis(AAPD) for the functionally dependent and elderly patient. PeritDial Int 25[Suppl 3]: S60–S63, 2005

32. Castrale C, Evans D, Verger C, Fabre E, Aguilera D, RyckelynckJP, Lobbedez T: Peritoneal dialysis in elderly patients: Report fromthe French Peritoneal Dialysis Registry (RDPLF). Nephrol DialTransplant 25: 255–262, 2010

33. Povlsen JV, Ivarsen P: Assisted peritoneal dialysis: Also for the latereferred elderly patient. Perit Dial Int 28: 461–467, 2008

34. Lobbedez T, Moldovan R, Lecame M, Hurault DL, El Haggan W,Ryckelynck JP: Assisted peritoneal dialysis: Experience in a Frenchrenal department. Perit Dial Int 26: 671–676, 2006

35. Dratwa M: Costs of home assistance for peritoneal dialysis: Resultsof a European survey. Kidney Int Suppl S72–S75, 2008

36. Thakar CV, Quate-Operacz M, Leonard AC, Eckman MH: Out-comes of hemodialysis patients in a long-term care hospital setting:A single-center study. Am J Kidney Dis 55: 300–306, 2010

37. Reddy NC, Korbet SM, Wozniak JA, Floramo SL, Lewis EJ:Staff-assisted nursing home haemodialysis: Patient characteristicsand outcomes. Nephrol Dial Transplant 22: 1399–1406, 2007

38. Taskapan H, Tam P, LeBlanc D, Ting RH, Nagai GR, Chow SS,Fung J, Ng PS, Sikaneta T, Roscoe J, Oreopoulos DG: Peritonealdialysis in the nursing home. Int Urol Nephrol 42: 545–551, 2010

39. Wang T, Izatt S, Dalglish C, Bargman J, Jassal S, Vas S, OreopoulosD: Peritoneal dialysis in a nursing home: Limited survival expecta-tions. Clin Nephrol 60: 373–374, 2003

40. Anderson JE, Kraus J, Sturgeon D: Incidence, prevalence, andoutcomes of end-stage renal disease patients placed in nursinghomes. Am J Kidney Dis 21: 619–627, 1993

41. Anderson JE, Sturgeon D, Lindsay RM, Schiller A: Use of contin-uous ambulatory peritoneal dialysis in a nursing home: Patientcharacteristics, technique success and survival predictors. Am JKidney Dis 16: 137–141, 1990

42. Anderson JE, Kraus J, Sturgeon D: Incidence, prevalence, andoutcomes of end-stage renal disease patients placed in nursinghomes. Am J Kidney Dis 21: 619–627, 1993

43. Carey HB, Chorney W, Pherson K, Finkelstein FO, Kliger AS:Continuous peritoneal dialysis and the extended care facility. Am JKidney Dis 37: 580–587, 2001

44. Leinau L, Murphy TE, Bradley E, Fried T: Relationship betweenconditions addressed by hemodialysis guidelines and non-ESRD-specific conditions affecting quality of life. Clin J Am Soc Nephrol4: 572–578, 2009

45. Smith C, Silva-Gane M, Chandna S, Warwicker P, Greenwood R,Farrington K: Choosing not to dialyse: Evaluation of plannednon-dialytic management in a cohort of patients with end-stage renalfailure. Nephron Clin Pract 95: c40–c46, 2003

46. Murtagh FE, Marsh JE, Donohoe P, Ekbal NJ, Sheerin NS, HarrisFE: Dialysis or not? A comparative survival study of patients over75 years with chronic kidney disease stage 5. Nephrol Dial Trans-plant 22: 1955–1962, 2007

47. Carson RC, Juszczak M, Davenport A, Burns A: Is maximumconservative management an equivalent treatment option to dialysisfor elderly patients with significant comorbid disease? Clin J AmSoc Nephrol 4: 1611–1619, 2009

48. Joly D, Anglicheau D, Alberti C, Nguyen AT, Touam M, GrunfeldJP, Jungers P: Octogenarians reaching end-stage renal disease:Cohort study of decision-making and clinical outcomes. J Am SocNephrol 14: 1012–1021, 2003

49. Smith C, Silva-Gane M, Chandna S, Warwicker P, Greenwood R,Farrington K: Choosing not to dialyse: Evaluation of plannednon-dialytic management in a cohort of patients with end-stage renalfailure. Nephron Clin Pract 95: c40–c46, 2003

50. De Biase V, Tobaldini O, Boaretti C, Abaterusso C, Pertica N,Loschiavo C, Trabucco G, Lupo A, Gambaro G: Prolonged conser-vative treatment for frail elderly patients with end-stage renaldisease: The Verona experience. Nephrol Dial Transplant 23: 1313–1317, 2008

51. Carson RC, Juszczak M, Davenport A, Burns A: Is maximumconservative management an equivalent treatment option to dialysisfor elderly patients with significant comorbid disease? Clin J AmSoc Nephrol 4: 1611–1619, 2009

52. Chandna SM, Schulz J, Lawrence C, Greenwood RN, Farrington K:Is there a rationale for rationing chronic dialysis? A hospital basedcohort study of factors affecting survival and morbidity. BMJ 318:217–223, 1999

53. Cecka JM: Kidney transplantation in the United States. Clin Transpl1–18, 2008

54. Kiberd B, Boudreault J, Bhan V, Panek R: Access to the kidneytransplant wait list. Am J Transplant 6: 2714–2720, 2006

55. Oniscu GC, Schalkwijk AA, Johnson RJ, Brown H, Forsythe JL:Equity of access to renal transplant waiting list and renal transplan-tation in Scotland: Cohort study. BMJ 327: 1261, 2003

Nephrology Self-Assessment Program - Vol 10, No 1, January 2011 65

Page 78: 2011.01 - Geriatric Nephrology

56. Schold J, Srinivas TR, Sehgal AR, Meier-Kriesche HU: Half ofkidney transplant candidates who are older than 60 years now placedon the waiting list will die before receiving a deceased-donortransplant. Clin J Am Soc Nephrol 4: 1239–1245, 2009

57. Rao PS, Merion RM, Ashby VB, Port FK, Wolfe RA, Kayler LK:Renal transplantation in elderly patients older than 70 years of age:Results from the Scientific Registry of Transplant Recipients.Transplant 83: 1069–1074, 2007

58. Collini A, Kalmar P, Dhamo A, Ruggieri G, Carmellini M: Renaltransplant from very old donors: How far can we go? Transplant 87:1830–1836, 2009

59. Frei U, Noeldeke J, Machold-Fabrizii V, Arbogast H, Margreiter R,Fricke L, Voiculescu A, Kliem V, Ebel H, Albert U, Lopau K,Schnuelle P, Nonnast-Daniel B, Pietruck F, Offermann R, Persijn G,Bernasconi C: Prospective age-matching in elderly kidney trans-plant recipients: A 5-year analysis of the Eurotransplant SeniorProgram. Am J Transplant 8: 50–57, 2008

60. The UK Renal Registry. Available at: www.renalreg.com. AccessedMay 2010

61. Knoll G, Cockfield S, Blydt-Hansen T, Baran D, Kiberd B, Lands-berg D, Rush D, Cole E: Canadian Society of Transplantation:Consensus guidelines on eligibility for kidney transplantation.CMAJ 173: S1–S25, 2005

62. Schold JD, Srinivas TR, Kayler LK, Meier-Kriesche HU: Theoverlapping risk profile between dialysis patients listed and notlisted for renal transplantation. Am J Transplant 8: 58–68, 2008

63. Meier-Kriesche HU, Friedman G, Jacobs M, Mulgaonkar S,Vaghela M, Kaplan B: Infectious complications in geriatric renaltransplant patients: Comparison of two immunosuppressive proto-cols. Transplant 68: 1496–1502, 1999

64. Meier-Kriesche HU, Kaplan B: Immunosuppression in elderly renaltransplant recipients: Are current regimens too aggressive? DrugsAging 18: 751–759, 2001

65. Dharnidharka VR, Agodoa LY, Abbott KC: Risk factors for hospi-talization for bacterial or viral infection in renal transplant recipi-ents: An analysis of USRDS data. Am J Transplant 7: 653–661,2007

66. Snyder JJ, Israni AK, Peng Y, Zhang L, Simon TA, Kasiske BL:Rates of first infection following kidney transplant in the UnitedStates. Kidney Int 75: 317–326, 2009

67. Meier-Kriesche HU, Ojo A, Hanson J, Cibrik D, Lake K, AgodoaLY, Leichtman A, Kaplan B: Increased immunosuppressive vulner-ability in elderly renal transplant recipients. Transplant 69: 885–889, 2000

68. Meier-Kriesche HU, Ojo AO, Hanson JA, Kaplan B: Exponentiallyincreased risk of infectious death in older renal transplant recipients.Kidney Int 59: 1539–1543, 2001

69. Wolfe RA, Ashby VB, Milford EL, Ojo AO, Ettenger RE, AgodoaLY, Held PJ, Port FK: Comparison of mortality in all patients ondialysis, patients on dialysis awaiting transplantation, and recipientsof a first cadaveric transplant. N Engl J Med 341: 1725–1730, 1999

70. Jassal SV, Krahn MD, Naglie G, Zaltzman JS, Roscoe JM, Cole EH,Redelmeier DA: Kidney transplantation in the elderly: A decisionanalysis. J Am Soc Nephrol 14: 187–196, 2003

71. Gill J, Bunnapradist S, Danovitch GM, Gjertson D, Gill JS, CeckaM: Outcomes of kidney transplantation from older living donors toolder recipients. Am J Kidney Dis 52: 541–552, 2008

72. Foley DP, Patton PR, Meier-Kriesche HU, Li Q, Shenkman B,Fujita S, Reed A, Hemming AW, Kim RD, Howard RJ: Long-termoutcomes of kidney transplantation in recipients 60 years of age andolder at the University of Florida. Clin Transpl 101–109, 2005

73. Moore PS, Farney AC, Hartmann EL, Rogers J, Doares W, Gaut-reaux MD, Iskandar SS, Hairston G, Adams PL, Stratta RJ: Expe-rience with deceased donor kidney transplantation in 114 patientsover age 60. Surgery 142: 514–523, 2007

74. Kauffman HM, McBride MA, Cors CS, Roza AM, Wynn JJ: Early

mortality rates in older kidney recipients with comorbid risk factors.Transplant 83: 404–410, 2007

75. Moore PS, Farney AC, Hartmann EL, Rogers J, Doares W, Gaut-reaux MD, Iskandar SS, Hairston G, Adams PL, Stratta RJ: Expe-rience with deceased donor kidney transplantation in 114 patientsover age 60. Surgery 142: 514–523, 2007

76. USRDS 2004 annual data report. Am J Kidney Dis 45: 8–280, 200577. Sener A, Schweitzer EJ, Munivenkatappa R, Cooper M, Bartlett ST,

Philosophe B, Barth RN: Deceased-donor renal transplantation inthe geriatric population demonstrates equal graft survival comparedwith younger recipients. Transplant 87: 1549–1554, 2009

78. Jassal SV, Trpeski L, Zhu N, Fenton SSA, Hemmelgarn BR:Changes in survival over the years 1990–1999 for elderly patientsinitiating dialysis. CMAJ 177: 1033–1038, 2007

79. Wolfe RA, Port FK, Hawthorne VM, Guire KE: A comparison ofsurvival among dialytic therapies of choice: In-center hemodialysisversus continuous ambulatory peritoneal dialysis at home. Am JKidney Dis 15: 433–440, 1990

80. Nelson CB, Port FK, Wolfe RA, Guire KE: Comparison of contin-uous ambulatory peritoneal dialysis and hemodialysis patient sur-vival with evaluation of trends during the 1980s. J Am Soc Nephrol3: 1147–1155, 1992

81. Lowrie EG, Huang WH, Lew NL: Death risk predictors amongperitoneal dialysis and hemodialysis patients: A preliminary com-parison. Am J Kidney Dis 26: 220–228, 1995

82. Collins AJ, Weinhandl E, Snyder JJ, Chen SC, Gilbertson D:Comparison and survival of hemodialysis and peritoneal dialysis inthe elderly. Semin Dial 15: 98–102, 2002

83. Harris SA, Lamping DL, Brown EA, Constantinovici N: Clinicaloutcomes and quality of life in elderly patients on peritoneal dialysisversus hemodialysis. Perit Dial Int 22: 463–470, 2002

84. Vonesh EF, Snyder JJ, Foley RN, Collins AJ: The differentialimpact of risk factors on mortality in hemodialysis and peritonealdialysis. Kidney Int 66: 2389–2401, 2004

85. Vonesh EF, Snyder JJ, Foley RN, Collins AJ: Mortality studiescomparing peritoneal dialysis and hemodialysis: What do they tellus? Kidney Int Suppl S3–S11, 2006

86. Weinhandl ED, Foley RN, Gilbertson DT, Arneson TJ, Snyder JJ,Collins AJ: Propensity-matched mortality comparison of incidenthemodialysis and peritoneal dialysis patients. J Am Soc Nephrol 21:499–506, 2010

87. Sanabria M, Munoz J, Trillos C, Hernandez G, Latorre C, Diaz CS,Murad S, Rodriguez K, Rivera A, Amador A, Ardila F, Caicedo A,Camargo D, Diaz A, Gonzalez J, Leguizamon H, Lopera P, MarinL, Nieto I, Vargas E: Dialysis outcomes in Colombia (DOC) study:A comparison of patient survival on peritoneal dialysis vs hemodi-alysis in Colombia. Kidney Int Suppl S165–S172, 2008

88. Tse KC, Lui SL, Lo WK: Comparison of long-term survival (be-yond 12 years) in patients on peritoneal dialysis and on hemodial-ysis. Perit Dial Int 23[Suppl 2]: S104–S108, 2003

89. Liem YS, Wong JB, Hunink MG, de Charro FT, Winkelmayer WC:Comparison of hemodialysis and peritoneal dialysis survival in theNetherlands. Kidney Int 71: 153–158, 2007

90. McDonald SP, Marshall MR, Johnson DW, Polkinghorne KR:Relationship between dialysis modality and mortality. J Am SocNephrol 20: 155–163, 2009

91. Laupacis A, Keown P, Pus N, Ferguson B, Wong C, Muirhead N: Astudy of the quality of life and cost-utility of renal transplantation.Kidney Int 50: 235–242, 1996

92. Lamping DL, Constantinovici N, Roderick P, Normand C, Hender-son L, Harris S, Brown E, Gruen R, Victor C: Clinical outcomes,quality of life, and costs in the North Thames Dialysis Study ofelderly people on dialysis: A prospective cohort study. Lancet 356:1543–1550, 2000

93. Unruh ML, Newman AB, Larive B, Dew MA, Miskulin DC, GreeneT, Beddhu S, Rocco MV, Kusek JW, Meyer KB: The influence of

66 Nephrology Self-Assessment Program - Vol 10, No 1, January 2011

Page 79: 2011.01 - Geriatric Nephrology

age on changes in health-related quality of life over three years in acohort undergoing hemodialysis. J Am Geriatr Soc 56: 1608–1617,2008

94. Brown EA, Johansson L, Farrington K, Gallagher H, Sensky T,Gordon F, Silva-Gane M, Beckett N, Hickson M: BroadeningOptions for Long-term Dialysis in the Elderly (BOLDE): Differ-ences in quality of life on peritoneal dialysis compared to haemo-dialysis for older patients. Nephrol Dial Transplant 25: 3755–3763,2010

95. Covinsky KE, Palmer RM, Counsell SR, Pine ZM, Walter LC,Chren MM: Functional status before hospitalization in acutely illolder adults: Validity and clinical importance of retrospective re-ports. J Am Geriatr Soc 48: 164–169, 2000

96. Fried TR, Bradley EH, Williams CS, Tinetti ME: Functional dis-ability and health care expenditures for older persons. Arch InternMed 161: 2602–2607, 2001

97. Covinsky KE, Palmer RM, Fortinsky RH, Counsell SR, Stewart AL,Kresevic D, Burant CJ, Landefeld CS: Loss of independence inactivities of daily living in older adults hospitalized with medicalillnesses: Increased vulnerability with age. J Am Geriatr Soc 51:451–458, 2003

98. Gill TM, Allore HG, Holford TR, Guo Z: Hospitalization, restrictedactivity, and the development of disability among older persons.JAMA 292: 2115–2124, 2004

99. Chiu E, Markowitz SN, Cook WL, Jassal SV: Visual impairment inelderly patients receiving long-term hemodialysis. Am J Kidney Dis52: 1131–1138, 2008

100. Murray AM: Cognitive impairment in the aging dialysis and chronickidney disease populations: An occult burden. Adv Chronic KidneyDis 15: 123–132, 2008

101. Wright S, Danziger J: American Society of Nephrology GeriatricCurriculum: Chapter 21: Vascular access for hemodialysis in theelderly. Available at: www.asn-online.org/education_and_meetings/geriatrics/OnlineGeriatricsCurriculum.pdf. Accessed May 28, 2009

102. Renal Physicians Association, American Society of Nephrology:RPA/ASN position on quality of care at the end of life. DialTransplant 26: 776, 778–780, 782–783, 1997

103. Davison SN, Torgunrud C: The creation of an advance care plan-ning process for patients with ESRD. Am J Kidney Dis 49: 27–36,2007

104. Cianciaruso B, Pota A, Pisani A, Torraca S, Annecchini R, Lom-bardi P, Capuano A, Nazzaro P, Bellizzi V, Sabbatini M: Metaboliceffects of two low protein diets in chronic kidney disease stage 4–5:A randomized controlled trial. Nephrol Dial Transplant 23: 636–644, 2008

105. Brito-Ashurst I, Varagunam M, Raftery MJ, Yaqoob MM: Bicar-bonate supplementation slows progression of CKD and improvesnutritional status. J Am Soc Nephrol 20: 2075–2084, 2009

106. Scalone L, Borghetti F, Brunori G, Viola BF, Brancati B, Sottini L,Mantovani LG, Cancarini G: Cost-benefit analysis of supplementedvery low-protein diet versus dialysis in elderly CKD5 patients.Nephrol Dial Transplant 25: 907–913, 2010

107. Brunori G, Viola BF, Parrinello G, De Biase V, Como G, Franco V,Garibotto G, Zubani R, Cancarini GC: Efficacy and safety of avery-low-protein diet when postponing dialysis in the elderly: Aprospective randomized multicenter controlled study. Am J KidneyDis 49: 569–580, 2007

108. Fried LP, Tangen CM, Walston J, Newman AB, Hirsch C, Gottdie-ner J, Seeman T, Tracy R, Kop WJ, Burke G, McBurnie MA: Frailtyin older adults: Evidence for a phenotype. J Gerontol A Biol Sci MedSci 56: M146–M156, 2001

109. Buchner DM, Hornbrook MC, Kutner NG, Tinetti ME, Ory MG,Mulrow CD, Schechtman KB, Gerety MB, Fiatarone MA, Wolf SL:Development of the common data base for the FICSIT trials. J AmGeriatr Soc 41: 297–308, 1993

110. American Psychological Association: Diagnostic and Statistical

Manual of Mental Disorders, 4th Ed., Text Revision, Washington,DC, American Psychological Association, 2000

111. Tawney KW: Disablement and rehabilitation in end-stage renaldisease. Semin Dial 16: 447–452, 2003

Palliative Care and Geriatric Treatment ofPatients with Advanced Chronic Kidney Disease

This section reviews recent developments in thefield of palliative care as it relates to treatment of thegeriatric renal patient across the advanced chronickidney disease (CKD) and dialysis illness trajectory.Updated areas presented include the simultaneous pal-liative care model, geriatric decision making, nondi-alysis medical therapy (NDT), and symptom burdenwith a comment on opioids. Many critical issues inpalliative medicine lack high-quality evidence (1),making development of renal palliative guidelines andcollaborative research important priorities (2).

The most common renal patient that a newlytrained nephrologist will treat is an older patient witha spectrum of geriatric syndromes and comorbidities,who will be an unlikely transplant patient. Thesepatients whose goals are independence, functionality,and quality of life (QoL) have unmet palliative careneeds (3,4) that are magnified by geriatric susceptibil-ity factors such as cognitive and functional impair-ments, frailty, pain, increasing dependence at home,falls, and the realities of transitions of care that willhave an impact on what is ultimately a provisionalexistence (5). An integrated, individually tailored ge-riatric palliative care model with geriatric end points(6) that moves beyond traditional renal targets and abetter understanding of how geriatric syndromes, eachwith multiple causative factors and interacting patho-genetic pathways, are assessed and treated (7) areneeded to provide optimal geriatric renal care. It is notclear whether this will improve survival or other endpoints, but meeting current Kidney Disease OutcomesQuality Initiative (KDOQI) guidelines has not in theaggregate slowed the daunting mortality figures (8) orimproved the symptom burden in the geriatric CKD/dialysis population (9,10).

There is a serious effort to upgrade prognosticmodels in patients with ESRD (11–13). It is alsoappreciated that prognosis in the individual patient isultimately uncertain, including those who opt forNDT, leading to “prognostic ambiguity” which has aneffect on communication, decision making, and goalsof care. This uncertainty, however, should not limit thenecessity to discuss openly the implications of life

Nephrology Self-Assessment Program - Vol 10, No 1, January 2011 67

Page 80: 2011.01 - Geriatric Nephrology

either on dialysis or with NDT and describe physicaland psychosocial outcomes clearly and honestly. Prog-nostic difficulty may also partially explain the latereferral to and inadequate use of hospice in ESRD(14). The 2010 revised Shared Decision-Making in theAppropriate Initiation of and Withdrawal from Dialy-sis (15), which has been rigorously updated, addressesmany issues that renal patients face and is the mostcurrent evidence-based American reference regardingdialysis decision issues across all renal domains, in-cluding the intensive care unit (16). In a study thatexamined end-of-life (EOL) dialysis decision making(17), awareness and use of the first edition of thisdocument was independently associated with greaternephrologist preparedness, although only 39% ofthose surveyed felt well prepared. These physicianswere older, practiced longer, used time-limited trialsmore often, and had more experience with dialysiswithdrawal. This underscores the point that implemen-tation of palliative care and geriatric medicine educa-tion at the renal fellowship level is timely and thedevelopment of integrated subspecialty renal-pallia-tive care geriatric fellowships is warranted.

Palliative CarePalliative care is a form of treatment that strives

to match medical care to patient goals, relieve pain,and improve QoL for people with chronic or life-threatening illnesses regardless of prognosis (18,19).Palliative care was initially associated with EOL andhospice care but has evolved by necessity into asimultaneous or shared care model (18,20) appropriateat the beginning of any serious illness administeredsimultaneously with life-sustaining or curative therapies.The relative weight placed on each part (curative/restor-ative and palliative care) in this model (Figure 11)changes over time as a function of disease progressionand treatment burdens, functional and frailty status,and patient and family goals and needs. Eventually,restorative care is fully replaced by palliative care.Major health organizations have articulated consensusstatements to clarify the reach of palliative care builton the core concept of addressing all sources and typesof suffering through patient-family–based care. Thedefinition and goal of palliative care put forth by the2009 National Consensus Project for Quality PalliativeCare “is to prevent and relieve suffering and to supportthe best possible QoL for patients and their families,regardless of the stage of the disease or the need for

other therapies. Palliative care expands traditional dis-ease-model medical treatments to include the goals ofenhancing QoL for patient and family, optimizingfunction, helping with decision making, and providingopportunities for personal growth. As such, it can bedelivered concurrently with life-prolonging care or asthe main focus of care” (21).

The World Health Organization also affirms thesimultaneous palliative care model, stating that “it isapplicable early in the course of illness, in conjunctionwith other therapies that are intended to prolong life,such as chemotherapy or radiation therapy, and in-cludes those investigations needed to better understandand manage distressing clinical complications” (22). Aset of palliative care principles include treating painand other distressing symptoms; affirming life butaccepting dying as a normal process; intending neitherto hasten nor to postpone death; integrating psycho-logical and spiritual aspects of patient care; offering asupport system to help patients live as actively aspossible until death and assist the family during thepatient’s illness and in their own bereavement; andusing a team approach to achieve these goals with theunderlying goal of enhancing QoL and positively in-fluencing the course of illness (22).

Finally, the Centers for Medicare and MedicaidServices (CMS) stated that “palliative care meanspatient- and family-centered care that optimizes QoLby anticipating, preventing, and treating suffering.Palliative care throughout the continuum of illness

Di M difi Th /Disease Modifying Therapy/Cura�ve or restora�ve intent

Life closure

i iPallia�ve Care Death &

Bereavement

DiagnosisHospice

Dialysis therapy prolong life/improve func�on non-restora�ve non dialysis medical therapy

Time line

Figure 11. Simultaneous care model. See text for explana-tion. Originally published in A Model to Guide HospicePalliative Care (Ferris FD, Balfour HM, Bowen K, FarleyJ, Hardwick M, Lamontagne C, Lundy M, Syme A, West P.A Model to Guide Hospice Palliative Care. Ottawa, ON:Canadian Hospice Palliative Care Association, 2002);adapted by Meier et al. in reference 20. Figure has beenadapted from reference 20. Adapted with permission fromthe Canadian Hospice Palliative Care Association.

68 Nephrology Self-Assessment Program - Vol 10, No 1, January 2011

Page 81: 2011.01 - Geriatric Nephrology

involves addressing physical, intellectual, emotional,social, and spiritual needs and facilitating patient au-tonomy, access to information, and choice” (23). Thisseems incongruous with the Hospice Medicare benefitrequiring �6 months estimated survival for inclusion.Efforts are ongoing to modify this with a palliativecare benefit independent of prognosis (24).

The palliative care umbrella encompasses twoseparate, nonsynonymous but fluid subdivisions (seeFigure 15): (1) Non-hospice palliative care (e.g., treat-ment of symptoms, initiation of advance care planning[ACP], psychosocial support), which is not tied toEOL or dying, is independent of prognosis, is appro-priate at any point for a patient who has a seriousillness and is receiving life-prolonging treatment, and(2) hospice palliative care, which is tied to prognosisand provided at the EOL and for which there is nogood single evidence-based definition (1).

The simultaneous care model of palliativecare is an appropriate template for theevaluation and treatment of dialysis pa-tients who undergo targeted medical ther-apies to address their myriad needs.

Practically, palliative care can be classified intoconventional care that is provided by professionalswho are not specifically trained in palliative care butwho may have received education in basic manage-ment and specialized care or specialist palliative carethat is provided by palliative medicine–trained profes-sionals who may also be certified by the AmericanBoard of Internal Medicine certified subspecialty. Pal-liative care services are offered through the followingvenues: Institution-based (hospital or nursing home)palliative care consultation or inpatient (non-hospiceor hospice-based) palliative care units; outpatient pal-liative care clinics for patients after hospital dischargeor referred from other subspecialties; consultativecommunity palliative care programs that help homehealth agencies manage patients who are not yet onhospice; and hospice care provided in the home, nursinghome, residential facility, or an inpatient unit (21).

Palliative Care in Medical PracticeThere is evidence that palliative care consulta-

tion improves patient care. A VA study (25) comparedpalliative consultation and usual medical care in the

last month of life. A retrospective telephone survey of524 family survivors used a scoring system that ex-amined key areas of palliative care intervention in ninedomains: Patient well-being and dignity, adequacy ofcommunication, respect for treatment preferences,emotional and spiritual support, management of symp-toms, access to the inpatient facility of choice, carearound the time of death, access to home care services,and access to benefits and services after the patient’sdeath. A total of 56% of patients received a palliativecare consultation and most commonly within the lastweeks of life (mean and median 14 days; interquartilerange 3 to 24). Using multivariable linear regression,the most significant benefits of palliative care consul-tation were better overall patient and family satisfac-tion with improvements in emotional and spiritualsupport, information and communication, care at timeof death, access to home services, and well-being anddignity. No unwanted interventions were undertaken.Patients were more likely to receive treatments thatthey wanted and be better treated for symptoms of painand posttraumatic stress disorder. Earlier consultationalso led to higher scores for communication and emo-tional support. The likelihood of getting a consultationwas significant only with a cancer diagnosis (P �0.001), and no renal patients were included in thestudy.

There has been an important perception change,and it is now widely recognized that patients withnon-cancer diagnoses such as advanced renal failure(2), long-term dialysis (26), heart failure (18,27,28),and end-stage pulmonary disease (29) are also candi-dates for a simultaneous palliative care model ap-proach.

In a study (30) of 203 chronically critically illmechanically ventilated patients in a respiratory careunit, limitation of life-sustaining therapies was mostinfluenced by palliative care consultation (adjustedodds ratio 40.9; 95% confidence interval [CI] 13.1 to127.4; P � 0.0001) and previously appointed healthcare proxy (adjusted odds ratio 6.7; 95% CI 2.3 to20.0; P � 0.00060).

A systematic review of EOL palliative care in-terventions (1) found strong evidence for the use ofopioids in cancer pain and for dyspnea in chronicobstructive pulmonary disorder and the use of multi-component interventions to improve continuity inheart failure. Moderate evidence supported ACP ledby skilled facilitators and interventions to alleviate

Nephrology Self-Assessment Program - Vol 10, No 1, January 2011 69

Page 82: 2011.01 - Geriatric Nephrology

caregiver burden. There was no evidence for the use ofopioids in non-cancer pain or management of dyspneain advanced heart failure. In addition, no precise def-inition of EOL was found but instead a spectrumincluding clinician assessment of “active dying,” sur-vival prediction rules, prognostic tools, and the sur-prise question, “Would it be a surprise if this patientwere to die within 6 mo?” which was recently studiedin a cohort of dialysis patients (11). The authorspointed out that waiting for near certainty of EOLwould fail to identify most dying people, so palliativecare should be a part of the therapy of any patient withserious illness. This study underscores the importanceof symptom management research in non-cancer ad-vanced illness and the need for skill sets to delivereffective ACP. It also suggests that careful use ofphrases such as end of life, dying, and “actively dying”must be framed in an ongoing supportive communi-cation process that learns who the patient is, what hisor her goals are, what he or she wants to know, andwhat his or her support system is.

Geriatric Palliative CareGeriatric palliative care (31) is distinct in its

focus on identifying and managing geriatric syn-dromes, coordinating ACP, and organizing care in avariety of long-term care settings (home care, adultday care, residential and assisted living, nursing home)for geriatric patients who transition through thechronic illness model of slow decline, increasing de-pendency, and mental incapacity.

Geriatric palliative medicine, according to a con-sensus definition issued by the European Union Geri-atric Medical Society (32), “is the medical care andmanagement of older patients with health-relatedproblems and progressive, advanced disease for whichthe prognosis is limited and the focus of care is QoL.”Their set of principles includes combining geriatricmedicine and palliative care concepts; a focus oncomprehensive geriatric assessment; recognition ofunique features of symptom and disease presentationin the elderly; management of pain and other symp-toms; integration of social, spiritual, psychological,and environmental aspects; safe drug prescribing; atailored multidisciplinary approach for both patientand family; an emphasis on autonomy, shared deci-sion-making, and acknowledgment of ethical issues;good communication skills; needs assessment of olderpatients and their families across all settings (home,

long-term care, hospices, and hospital); special atten-tion to transitions within and between settings of care;and a support system to help families cope during thepatient’s terminal phase of care. This alliance permitsthe merging of expertise (33) not only to meet theneeds of the elderly patient with chronic progressivelyburdensome disease but also to provide geriatric med-ical subspecialties with appropriately tailored pallia-tive care tools.

Palliative Care in Advanced CKD and DialysisSeveral recent reviews on renal palliative care

(26,34–39) are good examples of the simultaneouscare model in non-cancer progressive chronic illness.Principles include the following (34,37,40,41):

Y an agreed-on management plan to optimize QoL andrelieve suffering

Y offered simultaneously with all other appropriate med-ical therapy

Y not synonymous with EOL or hospice careY not just the absence of dialysis provisionY suitable in dialysis patients, tooY appropriate in all patients with serious illness

Components of renal palliative care include

Y ACP (42): A process of ongoing communication toupdate prognosis/goals of care/preferences as the tra-jectory of decline progresses and EOL issues becomemore prominent

Y pain and symptom managementY patient and family support to:

Y create a sense of control over the patient’s healthcare

Y relieve potential burdens on loved oneY strengthen interfamily relationships

Y Hospice referral when appropriate (�6 months esti-mated survival)

Geriatric renal palliative care incorporates geri-atric principles of the interdisciplinary team through aholistic approach. Active medical treatment of ad-vanced renal and dialysis complications is continuedsimultaneously with evaluation and treatment of geri-atric syndromes and symptoms (e.g., pain, depression,fatigue, insomnia, pruritus, constipation) to maximizefunction and QoL, avoid unnecessary hospitalizations,and eventually allow a dignified death in the place ofthe patient’s choosing. The “family meeting” thatunites patients and family members with their healthcare team (including subspecialists) is a major vehiclefor palliative care delivery. It is a forum for informa-tion sharing, exploration of patient and family hopes

70 Nephrology Self-Assessment Program - Vol 10, No 1, January 2011

Page 83: 2011.01 - Geriatric Nephrology

and expectations, clarification of current status, andelaboration of a care strategy to aim for realistic goals.Besides nephrologist unpreparedness (17), symptomsare unrecognized (43) and undertreated (44). EffectiveACP is unevenly practiced. This was shown in asurvey study (4) of EOL preferences of 584 renalpatients (40.8% predialysis, 38% hemodialysis, 12.5%peritoneal dialysis, and 8.7% transplant) with amean � SD age of 68.16 � 14.40. Although 84.6% ofpatients felt informed about their medical conditionand prognosis, 90.4% reported that their nephrologisthad not discussed prognosis or EOL issues in the pastyear. Only 17.9% believed that their health wouldworsen during the ensuing year; 69% did not knowwhat palliative care was. The most notable findingswere that 61% of patients regretted their decision tostart dialysis, and it was made by others in 65.8% ofcases (physician 51.95%; family 13.90%). Finally,besides family and friends, patients relied significantlyon the renal team for psychosocial support, medicalinformation, and decision making. Within the limita-tions of this specific population (majority Caucasianand high education) and the use of surveys, the dis-connect between what patients perceived and wantedand what they received is dramatically illustrated.

Preferences of older patients may also changeover time. In an observational cohort study (45), pref-erences for life-sustaining therapy of 189 community-dwelling patients who were aged �60 years and hadadvanced cancer, heart failure, or chronic obstructivepulmonary disease were evaluated at least every 4months for up to 2 years. Participants were askedwhether they would undergo high-burden therapy dur-ing an acute medical event to avoid death but risk atradeoff with an impaired health state. During thestudy, 35% had an inconsistent flip-flop preferencetrajectory. This proportion increased to 48 and 49%when the participants were specifically asked whetherthey would accept a physical or cognitive disability.Participants with non-cancer diagnoses were morelikely to have inconsistent preferences over time whencompared with cancer patients, which could reflecttheir variable health trajectories, limited information,or messages received from their health care providers.These findings demonstrate the complexity of ad-vanced illness in the elderly and may explain thereluctance of some patients to discuss the dialysisquestion, ambivalence in pursuing access, and presen-

tation to the hospital for emergent dialysis and perma-cath insertion.

Geriatric Medical Decision MakingStudies (8) have documented the dismal survival

in dialysis patients who are older than 80: Average1-year mortality of 46%. Geriatric susceptibility fac-tors associated with increased mortality include ad-vancing age (versus 80 to 84 years; 85 to 89 years RR1.22 [95% CI 1.20 to 1.24], �90 years RR 1.56 [95%CI 1.51 to 1.61]), serum albumin concentration �35g/L (RR 1.28 [95% CI 1.25 to 1.30]), nonambulatorystatus (RR 1.54 [95% CI 1.49 to 1.58]), congestiveheart failure (RR 1.21 [95% CI 1.19 to 1.23]), under-weight (RR 1.20 [95% CI 1.18 to 1.23]), and numberof comorbid conditions (versus 0 to 1; 2 to 3 RR 1.31[95% CI 1.28 to 1.33], �4 RR 1.68 [95% CI 1.64to 1.72].

The median survival after dialysis initiation inthese patient is approximately one sixth that of age-matched nondialysis population with the following agecategory survival figures: 65 to 79 years, 24.9 months(interquartile range [IQR] 8.3 to 51.8 months); 80 to84 years, 15.6 months (IQR 4.8 to 35.5 months); 85 to89 years, 11.6 months (IQR 3.7 to 28.5 months); and90 years, 8.4 months (IQR 2.8 to 21.3 months). It isimportant to note the IQRs, so although these figuresare useful for general statements about prognosis, theyshould be taken in the context of the patient’s prefer-ences and goals.

Another risk factor for poor outcome on dialysiscould be called “nursing home” ESRD syndrome,which is associated with functional decline and signifi-cant mortality. Kurella et al. (46), in a retrospectivestudy, compared functional status defined by the abilityto perform activities of daily living (ADLs; eating, dress-ing, toileting, maintaining personal hygiene, walking,transferring from a chair to a standing position, andchanging positions in bed) in 3702 nursing homepatients before initiation of and during 1 year oflong-term dialysis (95% hemodialysis; 5% perito-neal dialysis) over 1 year using the Minimum DataSet-Activities of Daily Living (MDS-ADL), a stan-dardized nursing home assessment tool. At 3 and 12months after starting dialysis, functional status waspreserved in only 39 and 13%, respectively, withmortality rates of 24, 41, and 58% at 3, 6, and 12months. After adjustment for demographic and clin-ical factors, older age, dementia, hospitalization at

Nephrology Self-Assessment Program - Vol 10, No 1, January 2011 71

Page 84: 2011.01 - Geriatric Nephrology

dialysis initiation, and serum albumin �3.5 g/dlwere independently associated with a lower odds ofa preserved functional status after 12 months ofdialysis. The authors concluded that dialysis initia-tion in nursing home patients is associated withsubstantial functional deterioration and mortality.This may not dissuade a patient or family frominitiating dialysis, but realistic goals must be part ofthe decision-making process.

These studies reinforce what is widely knownin the geriatric literature, namely that functionalstatus is as useful as comorbidity in risk assessment.It can improve mortality prediction in older patientsand lead to more accurate risk adjustment (47,48).

Comprehensive geriatric assessment (CGA) isused to define overall health status, identify geriat-ric syndromes, formulate individualized diagnosticinterventions, and classify patients into functionalage categories (healthy, vulnerable, frail) that haveprognostic and therapeutic value (49).

Geriatric syndromes are powerful predictors ofadverse outcomes, including mortality, hospitaliza-tion, nursing home placement, and hip fractures. CKD,uremia, and dialysis accelerate these outcomes, espe-cially the expression and progression of frailty inpredisposed patients (50–53). Some form of geriatricdeficit screening should be part of every dialysisdecision paradigm.

One useful screening test is the Vulnerable El-ders Survey-13 (VES-13), a 13-item self-administered,function-based tool (www.rand.org/health/projects/acove/survey.html) with a maximum score of 10points that can identify older community-dwellingindividuals who are at risk for adverse outcomes. Ascore of �3 identifies vulnerable adults (54) and theneed for further investigation.

This was shown in an observational longitudinalstudy of 420 community-dwelling older adults, inwhich those with a VES-13 score of �3 were followedfor a mean of 11 months (8 to 14 months). VES-13scores predicted death and functional decline (P �0.001; area under the curve 0.66). The estimatedcombined risk for death and decline with VES-13score of 3 was 23%, which increased to 60% with ascore of 10. Comorbidity was not a significant predic-tor after controlling for VES-13 score (55).

Models developed in geriatric oncology (49,56–59) use modified geriatric assessment tools to evaluatethe risks and benefits of therapy in elderly cancer

patients and the appropriateness of modified treatmentprotocols. The VES-13 was used in a group of elderlypatients who had prostate cancer and were receivingchemotherapy and compared with a CGA (54). Fiftypercent of patients had impairment by VES-13 (�3)compared with 60% of patients with impairment ontwo CGAs. Reliability (Pearson c coefficient 0.92),sensitivity (72.7%), and specificity (85.7%) for geri-atric deficits were highly predictive for identifyingimpairment. Patient with a positive VES-13 performedsignificantly worse on evaluations of ADLs (P �001), physical performance (P � 0.002), comorbidity(P � 0.004), and cognitive impairment (P � 0.003).

This has not been formally studied in advancedCKD/dialysis, although dialysis patients with a diag-nosis of frailty variously defined have a higher all-cause mortality and hospitalization rate. This wasillustrated in the Dialysis Morbidity and MortalityWave 2 Study (50), which found an overall prevalenceof frailty of 67.7% in a group of 2275 adult dialysispatients with 78.8% in those who were older than 80years old and 66.4% in those aged 50 to 60 years.Frailty was independently associated with a higher riskfor death (adjusted hazard ratio [HR] 2.24; 95% CI1.60 to 3.15) and a combined outcome of death orhospitalization (adjusted HR 1.63, 95% CI 1.41 to1.87). Although frailty is a predictor for death, themore important implication is that frailty creates aself-propagating “cycle of frailty” (60) (Figure 12) thatleads to increasing disability and dependence fed byrecurrent (hazards of) hospitalizations and acute med-ical (sentinel) events that converge into a mountingburden of geriatric and nursing home syndromes tocreate the well-described ESRD disease trajectory(35,61).

The elderly patient who has CKD and is beingevaluated for dialysis may range from a “healthy”cognitively intact person who can verbalize his or herpreferences to a “frail” nursing home patient withdecisional incapacity and no stated previous wishes,health care proxy, or surrogates. Because geriatricpatients with CKD are a heterogeneous group, someform of geriatric assessment is part of the initialdecision-making process. For the non-geriatriciannephrologist, the Get Up And Go Test (from sittingposition, stand without using arms for support, andwalk 10 ft/3 m and back as quickly as possible) and theRapid Chair Rise (stand up from a seated position in ahardback chair with arms folded) can be considered. In

72 Nephrology Self-Assessment Program - Vol 10, No 1, January 2011

Page 85: 2011.01 - Geriatric Nephrology

this case, physical frailty is defined as scoring �10 sfor the go test and/or an inability to rise from the chairwithout using the arms. A moderately frail patientwould be unable to complete either test, whereasseverely frail is defined as inability to complete both(62). These tests with other screening tools such as theVES-13 can be used as part of a “staging the aging”classification (49) to stratify the patient into the fol-lowing categories:

Y Healthy/usual: This is the most optimal dialysis patientwho might also be a transplant candidate. Geriatricscreening might show a VES-13 of 0 to 2, negativefrailty testing, normal walking speed and chair test,intact ADLs and instrumental activities of daily living(iADLs), absent geriatric syndromes or limiting comor-bidities, and no polypharmacy (fewer than five medi-cations).

Y Vulnerable: This is a more typical dialysis candidate.VES-13 might range from 3 to 6, frailty syndrometesting would be positive for one to two of five com-ponents (63); there could be one ADL and/or iADLdeficit, some geriatric syndrome(s) (falls, dementia,depression, delirium), one to three comorbidities, andpolypharmacy of five to eight medications. Geriatricintervention plans and palliative care evaluation (e.g.,rehabilitation, pain and nonpain symptom control,treatment of cognitive deficits and depression, limiting

polypharmacy, preventing falls, instituting home ser-vices, ACP) may slow the progression of geriatricsusceptibility factors that will adversely affect progno-sis, QoL, and the dialysis experience and begin theparticipation of families in shared decision making astransitions occur,

Y Frail: This is a suboptimal dialysis candidate andshould be considered for an NDT plan or a time-limiteddialysis trial with well-defined functional and QoLgoals and time limits if possible. Geriatric screeningwould yield poor scores with a VES-13 score of �7 to10, positive frailty syndrome testing (three or more offive components [63]), two or more ADL or iADLdeficits, multiple geriatric syndromes, and four or morecomorbidities. Final decisions will hinge on patientpreferences either directly or through substituted judg-ment if decision making is impaired and QoL andcontextual issues. Palliative care intervention can arrangea “family meeting” to explore these preferences and otherconcerns of those involved in the decision making.

The four-topics method (64) modified for thegeriatric renal patient is a useful template to addressthe main components of a dialysis discussion in theelderly and is outlined in Figure 13 (65). Each topic isframed by underlying ethical principles and their as-sociated clinical counterparts. Although topic 2 takesprecedence, the more topics that can be fully exploredand discussed, the better informed and shared decisionswill be.

Functional age and frailty are necessaryfactors to assess in the dialysis decisionprocess.

Using the four-topics method and the RenalPhysicians Association guidelines (15) for shareddecision making, is the patient a candidate fordialysis or NDT? If the patient is found to befunctionally “frail” with geriatric susceptibility fac-tors for adverse outcomes and limited survival onthe basis of age and comorbidities than from amedical intervention standpoint, then that patient isa suboptimal candidate for dialysis. If that patient isdeemed “vulnerable” or “healthy,” then renal re-placement therapy (RRT) or perhaps transplantationwould be a choice. Exploration of the remainingtopics will help guide the dialysis decision to eitherdialysis or ND).

Walkingspeed

Disability

Dependency

• Dialysis/access issues• Surgery/amputations• Infections• Cardiovascular events

DiseaseEnvironmentMedications Chronic undernutrition

Weight loss

CYCLE OF FRAILTYpropagates ESRDdisease trajectory

Resting metabolic rate

Total energyexpenditureLow activity

Poor endurance

Activity

Sarcopeniashrinking

Insulinsensitivity

OsteopeniaVO2 max

DiseaseMedications

Aging-related changes

• Recurrent (hazards of) hospitalizations• Geriatric syndromes• “Nursing home” ESRD syndrome

Strength and powerWeaknessSlowness

Immobilization

Falls and injuries

Impairedbalance

Figure 12. Cycle of frailty. See text for explanation.Modified from reference 60 (Fried LP, Hadley EC, Wal-ston JD, Newman AB, Guralnik JM, Studenski S, HarrisTB, Ershler WB, Ferrucci L: From bedside to bench:Research agenda for frailty. Sci Aging Knowledge Envi-ron (31): pe24, 2005). Reprinted with permission fromAAAS.

Nephrology Self-Assessment Program - Vol 10, No 1, January 2011 73

Page 86: 2011.01 - Geriatric Nephrology

Nondialysis Medical Renal TherapyNDT, also called “maximum conservative

therapy,” “maximum conservative management”(66), or “active medical management without dial-ysis” (15), refers to the care of patients who haveadvanced CKD and who after an appropriate deci-sion-making process make an informed choice toforgo initiation of RRT and opt for medical man-agement.

There is more recognition of and experience withNDT in the United Kingdom than in the United States,where the focus until recently has been on RRT as thede facto treatment of choice, hence the common des-ignations “refusing dialysis” instead of “forgoing di-alysis” and “exercising informed consent to forgodialysis or opting for NDT.”

The 2008 United Kingdom Renal Associationguidelines (67) for a nondialysis treatment state,“Nondialytic treatment of patients with establishedrenal failure should be regarded as a specific man-agement option and not as ‘no treatment’ with thegoals to prolong survival if possible and optimizeQoL. It includes regular follow-up, a clear treatmentplan with timely arrangements for palliative and endof life care in close consultation with patients andtheir families.”

The Renal Physicians Association 2010 guide-lines state that “medical management without dialysisis an acceptable alternative that may better achieve

patients’ goals of care. It is active treatment whichentails advance care planning, implementation of pa-tients’ goals, and management of anemia, bone dis-ease, fluid balance, acidosis, and BP.” They suggestthat consideration be given to this therapy option inpatients who have stage 5 CKD; are older than 75years; have a high level of comorbidity, signifi-cantly impaired functional status, and serum albu-min �2.5 g/dl; and clinicians’ response of, “No, Iwould not be surprised,” to the surprise question(11,15), “Would I be surprised if this patient died inthe next year?”

As previously discussed, studies of NDT sug-gested that survival may not be significantly differentin selected subgroups (multiple comorbid conditions,ischemic heart disease) between those on long-termdialysis compared with patients who have stage 5CKD and are medically treated without RRT (68).Using a multidisciplinary team approach, fewer hos-pitalizations and more patient deaths at home may bepossible (69). This will provide a more humane anddignified EOL experience for the frail geriatric patientand his or her family (70).

In a retrospective cohort study (71) of conserva-tively treated patients with stage 5 CKD that examinedsurvival defined as starting from the first known timepoint of stage 5 CKD, the overall median patientsurvival was 21 months. Patients who were known toa nephrologist before reaching stage 5 CKD survived

Four Topics Method Adapted to Dialysis Decisions

1. Medical indica�ons for interven�on 2. Pa�ent preferences

What is func�onal age of this pa�ent orIs this pa�ent “healthy” “vulnerable” “frail”?What are the survival data?What are the geriatric suscep�bility factors?Are nursing home pa�ents different?

Based on the above:

Is the pa�ent a candidate for dialysis or non dialysismedical therapy?

Establish “big picture” goals (e.g. “pain free”, stay athome, live as long as possible)Explore pa�ent’s personal narra�veEngage the family

higher prevalence of cogni�ve dysfunc�on andinability to make decisionssubs�tuted judgment will be more common

Be prepared that:preferences may change w/�me and new eventssome pa�ents not able to decide or express theirpreferencessome want to receive limited or no informa�on anddelegate to others

3. Quality of life 4. Contextual features

No universal metricPersonal value judgmentSome objec�ve criteria (end stage demen�a, cachexia,advanced cancer) but families may not see it that waySignificant symptom burdenTime limited trial to assess if QOL acceptable

Is family suppor�ve of pa�ent’s decision?Are there conflicts between family members?Are descrip�ons of pa�ent wishes consistent?What is cultural, ethnic or religious belief background?Is there conflict among healthcare providers(“mixed messages”)or between the family and them?

Figure 13. Four-topics method for geriatric dialysis decision making. Adapted from references 64 and 65. See text forexplanation.

74 Nephrology Self-Assessment Program - Vol 10, No 1, January 2011

Page 87: 2011.01 - Geriatric Nephrology

longer (median 32 months) than those who presentedwith stage 5 CKD (15 months; P � 0.025). Serumalbumin �35 g/L was associated with greater survival,but other biochemical parameters, comorbidity grade,and age did not predict survival. The authors con-cluded that risk factors for survival may be differentbetween the dialysis- and nondialysis-treated popula-tion. Median survival estimates in other NDT studiesrange from 6.3 (72) to 13.9 (66) to 23.4 (70) months.In one study, the median time from a GFR of 15ml/min to death was 588 days (range 67 to 2528 days)for patients on dialysis and 540 days (range 4 to 2193days) for those managed with NDT (68), again illus-trating that prognostic information must be used care-fully. Also, earlier renal referral may increase thechances of longer survival in the NDT population.

Nephrologists should have competency in dis-cussing the NDT option with patients and familiesusing palliative care concepts. Patients will requireactive and increasing multilevel interventions. TheNDT trajectory is not well characterized, but one studyfound a period of relative stability with acceleration ofsymptoms in the last month similar to cancer (73).

NDT programs are being developed and assessedwith quality measures. A retrospective study (2) of aUK outpatient renal palliative service reviewed demo-graphics and interventions for 36 NDT patients whohad ESRD and were being medically treated in acombined “low-clearance clinic” and palliative careclinic during a 1-year period. The palliative care clinicteam consisted of a palliative medicine physician anda dedicated renal nurse who evaluated patients jointlyevery 2 weeks as needed. Patient status was reviewedevery 4 to 6 weeks by a combined renal/palliative team(a nephrologist, a palliative medicine doctor, two renalnurses, a social worker, and a counselor). Referralswere from the low-clearance clinic (defined by a GFRof �25 ml/min) after the dialysis decision process wascompleted, the patient opted for NDT, and symptomsor other needs developed. Thirty-two (88.9%) patientswere older than 70 years with 3 older than 90 year.Non-symptom interventions included community pal-liative specialist care team referral in the patient’slocal area with liaison updates; ongoing decision-making review (one patient switched to dialysis); andfamily communication involving ongoing explanationof the NDT choice, prognosis, trajectory, future care,preferences, and planning (nursing home, future ap-propriate testing and acute treatment, other support ser-

vices). Common complications of ESRD included fluidoverload (31%) treated with diuretics (no ultrafiltrationmentioned), anemia (53%) treated with erythropoietin-stimulating agents and intravenous iron, BP medicationadjustment (36%), and CKD mineral bone manage-ment (39%). Selected symptoms that required inter-vention included pain (55%); nausea, vomiting, andconstipation (22%); sleep disturbances (17%); andpruritus (11%). The authors stressed several points:Lack of documentation of when to withdraw renalinterventions so that decisions could be individual andbenefit-burden driven; many interventions revolvedaround information dissemination, family communica-tion, EOL planning, and community liaison services toget these patient back home; and clinic time was a onlya small part of the delivered care that also includedfrequent home visits, telephone support, and activepalliative liaison. The authors concluded that the rangeand complexity of care in this population are substan-tial and that more research is needed. Their version ofNDT, a conservative pathway treatment paradigm, ispresented in Figure 14.

Symptom burden is significant and un-dertreated in both dialysis patients andpatients with advanced CKD.

Symptom BurdenSeveral studies have documented the high symp-

tom burden across the advanced CKD and dialysisexperience (10,74). A cross-sectional study (10) ex-amined entry symptom prevalence in 66 patients withstage 5 CKD in an NDT treatment plan (conservativemanagement pathway) using the Memorial SymptomAssessment Scale–Short Form (MSAS-SF-32 symp-toms) with additional renal symptoms (muscle cramps,dry skin, muscle soreness, headaches, bone or jointpain, chest pain, restless legs) identified in the DialysisSymptom Index and a MSAS-SF validated in dialysispatients (75). The mean � SD age was 82.0 � 6.6years with a mean GFR of 11.2 � 2.8 ml/min. Themean number of symptoms was 11.58 � 5.2 with anadditional 2.77 � 1.7 renal symptoms. Symptom bur-den and QoL scores approached those in patients withadvanced cancer (76).

Another study (77) evaluated symptoms in 55elderly patients with a mean estimated GFR of 12.75

Nephrology Self-Assessment Program - Vol 10, No 1, January 2011 75

Page 88: 2011.01 - Geriatric Nephrology

ml/min (range 3 to 30 ml/min) using a patient-com-pleted tool to assess the presence and severity of 15symptoms during the 3 days before the test. Prevalentsymptoms included weakness (75%), poor mobility(75%), poor appetite (58%), pain (56%), pruritus(56%), and dyspnea (49%). The mean number ofsymptoms per patient was 6.8 (range 1 to 14) andfrequently reported as moderate, severe, or over-whelming. No significant association was demon-strated between the number of symptoms experiencedand either severity of renal disease or comorbidityburden. This suggests that some symptoms are non–organ based and a function of geriatric syndromes.

In a systematic review of dialysis patients (9), 60studies (59 long-term dialysis; one dialysis discontin-uation) were analyzed for symptom prevalence. Al-lowing for differences in symptom definition, severity,and point in time of assessment, approximately 50% ormore of patients experienced fatigue/tiredness, pruri-tus, constipation, anorexia, pain, and sleep distur-bances. In the one cited prospective study of 79 pa-tients who discontinued dialysis, symptoms present in

the last 24 hours included pain (42%), agitation (30%),myoclonus (28%), dyspnea (25%), fatigue (25%), diar-rhea (14%), and nausea (13%). More intense symptoms(�3 on a scale of 1 to 5) included pain (n � 5),myoclonus (n � 4), dyspnea (n � 3), and fatigue (n � 3).

A retrospective pilot study (78) examined dialy-sis discontinuation in 35 dialysis patients who alsoreceived palliative care. Sixty percent were male witha mean age of 70 years (range 32 to 92 years) andmean and median dialysis vintage of 18 and 11months, respectively (range 1 month to 7 years).The mean survival time was 10 days (range 1 to 48).The decision to withdraw was made by 51.5% of thepatients on their own and 45.7% by the family and/ornephrologist (no information for one patient). Reasonsincluded chronic progressive deterioration (66%),acute medical/surgical events (17%), failed trial(11%), and technical difficulties (6%). Thirty-fourpercent had cognitive impairment at the time of with-drawal. The most frequently reported distressingsymptoms were confusion/agitation, pain, and dys-pnea. Palliative care intervention decreased symptoms

Figure 14. Nondialysis medical therapy. The multidisciplinary nature and active communication components of nondialysismedical therapy are illustrated in this figure. Adapted from reference 2 (Murtagh FE, Murphy E, Shepherd KA, Donohoe P,Edmonds PM: End-of-life care in end-stage renal disease: Renal and palliative care. Br J Nurs 15: 8–11, 2006).

76 Nephrology Self-Assessment Program - Vol 10, No 1, January 2011

Page 89: 2011.01 - Geriatric Nephrology

in the last 24 hours for pain, agitation, and dyspnea,although 24% experienced unrelieved symptoms and17% had unrelieved psychological distress. Two pa-tients underwent ultrafiltration for a total of four treat-ments. Opioids were used in 97% of patients for pain,dyspnea, or both, of which 59% received hydromor-phone and 28% received morphine. No serious ad-verse effects were described for those who receivedmorphine. Benzodiazepines (commonly midazolamand lorazepam) were used in 91% of patients forindications of restlessness/agitation, twitching/myoclo-nus, or seizure-like activity. Patients who survivedlonger had more severe symptoms, especially pruritus,terminal delirium, nausea, and twitching. A total of88% of patients died in the hospital, 9% in an inpatientpalliative unit or hospice, and 3% in a nursing home.Greater than 90% of patients and families felt ade-quately supported by the health care team and eachother. The authors pointed out the unique nature ofdialysis withdrawal with its abrupt transition and pre-dictable short survival and rightly emphasized theimportance of open, honest, and early discussions withthose involved. All of these studies draw attention tothe need for early symptom assessment, proactive andvigorous symptom management, and expertise in drugprescribing to balance relief with avoidance of toxic-ity. Collaboration with palliative care colleagues opti-mizes this process.

Despite a large symptom burden, good symp-tom management studies are lacking in the renalpopulation. Recommendations exist, but there is apaucity of evidence-based consensus guidelines(79 – 85). The World Health Organization (WHO)analgesic ladder is effective for dialysis patientswith both nociceptive and neuropathic pain (86).Older renal patients who receive opioids needsmaller initial dosages and longer dosing intervals.Opioids are dosed according to GFR because parentand active metabolites can accumulate. Dialyzableopioids include morphine and hydromorphone andtheir metabolites. Methadone and fentanyl are re-ported to have inactive metabolites and are notdialyzable, but some dosage reduction is recom-mended (84). Opioid glucuronide metabolites areresponsible for adverse effects such as respiratorydepression, oversedation, myoclonus, and toxic ag-itation, and the half-life may be longer than theparent compound (84). For this reason, morphineshould be avoided because morphine-6-glucuronide

accumulates even though it is dialyzed. Recentpharmacokinetic dialysis studies of hydromorphoneand its active metabolite, hydromorphone-3-G(H3G) (85), showed that hydromorphone did notsubstantially accumulate, and although H3G in-creased between treatments, it was effectively re-moved by dialysis. This is welcome informationconcerning hydromorphone use in dialysis patientsand also provides a way to treat toxicity. There willbe different thresholds for opioid toxicity betweenpatients with advanced CKD (lower), dialysis pa-tients (higher), and those who discontinue dialysis(lowest). The main message is careful dosing andvigilant monitoring. Composite graded opioid choicesbased on the literature might include “cautious” useof tramadol and oxycodone for WHO step 2 mod-erate pain, “acceptable” use of fentanyl and metha-done, “careful” use of hydromorphone, and “cau-tious” use of oxycodone for WHO step 3 severepain. Opioids to be avoided on the basis of theliterature and case reports of serious adverse effectsinclude morphine, codeine, dihydrocodeine, pro-poxyphene, and meperidine. One cautionary note:Although methadone is probably the most compat-ible renal opioid available, its use should be super-vised by a pain or palliative medicine specialistbecause of its potency and complex pharmacokinet-ics (84).

A published evidence-based consensus guidelineexists for managing symptoms in dying patients whoare undergoing NDT or discontinuing dialysis anduses the Liverpool Care Pathway for the Dying Patientmodified for renal patients (87). Treatment recommen-dations are given for symptoms associated with “ac-tive dying,” meaning the last days of life. Because onecriterion for initiating this pathway is the inability toswallow, the group recommends that medications begiven by the subcutaneous route. Because this is aUK-based guideline, some of the medications used aredifferent from those available in the United States.General starting recommendations include haloperidolat 50%, the usual dosage for uremia-related nauseaand vomiting; glycopyrrolate at 50%, the normal dos-age for respiratory secretions; midazolam at a reduceddosage and increased dosing interval for terminal de-lirium; and fentanyl for pain and dyspnea switching toalfentanil if toxicity or tolerance develops, with “shortterm” use of hydromorphone or oxycodone if no otherchoices are available.

Nephrology Self-Assessment Program - Vol 10, No 1, January 2011 77

Page 90: 2011.01 - Geriatric Nephrology

Summary and ConclusionIt is clear that nephrologists will care for increasing

numbers of elderly patients who have varying combina-tions of geriatric syndromes and comorbid conditions,have advanced CKD or are on dialysis, come from thecommunity or nursing homes or will enter the renalsystem during a hospitalization, and are candidates forgeriatric palliative care. Given their uncertain progno-sis, high symptom burden, and mounting needs, afresh perspective that moves away from just EOL careand hospice timing and toward a simultaneous pallia-tive care model will better serve the needs of thispopulation. Whether patients opt for NDT or continueor forgo dialysis, the system in place before and afterthe dialysis decision or any significant medical deci-sion will determine the quality of care and the ability tomatch care to patient goals. A geriatric renal palliativecare action plan is presented in Figure 15. The next stepis to undertake intervention studies to accumulate a goodevidence base for clinical guidelines.

Finally, language matters with geriatric patients.It should focus on the needs of elderly patients andfamilies as they perceive them: Relief, practical help,support, hope, and comfort. Unless the patient timepoint is located at or near the hospice transition sectionof the simultaneous care model (Figure 11), prematureintroduction of EOL, dying, and bereavement lan-guage may initially be irrelevant and perhaps eventhreatening and influence the ability to establish goodcommunication and an empathetic forum for shareddecision making. Some older patients do not want an“open awareness” of death (88). Careful phraseologysuch as “to forgo” (to let pass; to do without) or“informed consent/right to forgo” instead of “to with-

hold/withdraw” or “refuse” dialysis; “nondialysismedical therapy” instead of “conservative therapy” toemphasize active intervention; “suboptimal” or “non-ideal” instead of “poor” dialysis candidate; and torefocus or re-orient hope or goals to optimize function,minimize discomfort, maximize time at home withfamily will create an environment in which the simul-taneous care model of renal palliative care can beimplemented.

References1. Lorenz KA, Lynn J, Dy SM, Shugarman LR, Wilkinson A, Mularski

RA, Morton SC, Hughes RG, Hilton LK, Maglione M, Rhodes SL,Rolon C, Sun VC, Shekelle PG: Evidence for improving palliativecare at the end of life: A systematic review. Ann Intern Med 148:147–159, 2008

2. Murtagh FE, Murphy E, Shepherd KA, Donohoe P, Edmonds PM:End-of-life care in end-stage renal disease: Renal and palliative care.Br J Nurs 15: 8–11, 2006

3. Fitzsimons D, Mullan D, Wilson JS, Conway B, Corcoran B, Demp-ster M, Gamble J, Stewart C, Rafferty S, McMahon M, MacMahon J,Mulholland P, Stockdale P, Chew E, Hanna L, Brown J, Ferguson G,Fogarty D: The challenge of patients’ unmet palliative care needs inthe final stages of chronic illness. Palliat Med 21: 313–322, 2007

4. Davison SN: End-of-life care preferences and needs: Perceptions ofpatients with chronic kidney disease. Clin J Am Soc Nephrol 5:195–204, 2010

5. Frankel VE: Man’s Search for Meaning, Revised and Updated Ed.,New York, Pocket Books, 1984

6. O’Hare AM: The management of older adults with a low eGFR:Moving toward an individualized approach. Am J Kidney Dis 53:925–927, 2009

7. Inouye SK, Studenski S, Tinetti ME, Kuchel GA: Geriatric syn-dromes: Clinical, research, and policy implications of a core geriatricconcept. J Am Geriatr Soc 55: 780–791, 2007

8. Kurella M, Covinsky KE, Collins AJ, Chertow GM: Octogenariansand nonagenarians starting dialysis in the United States. Ann InternMed 146: 177–183, 2007

9. Murtagh FE, Addington-Hall J, Higginson IJ: The prevalence ofsymptoms in end-stage renal disease: A systematic review. AdvChronic Kidney Dis 14: 82–99, 2007

10. Murtagh FE, Addington-Hall JM, Edmonds PM, Donohoe P, Carey I,Jenkins K, Higginson IJ: Symptoms in advanced renal disease: Across-sectional survey of symptom prevalence in stage 5 chronickidney disease managed without dialysis. J Palliat Med 10: 1266–1276, 2007

11. Moss AH, Ganjoo J, Sharma S, Gansor J, Senft S, Weaner B, DaltonC, MacKay K, Pellegrino B, Anantharaman P, Schmidt R: Utility ofthe “surprise” question to identify dialysis patients with high mortal-ity. Clin J Am Soc Nephrol 3: 1379–1384, 2008

12. Cohen LM, Ruthazer R, Moss AH, Germain MJ: Predicting six-month mortality for patients who are on maintenance hemodialysis.Clin J Am Soc Nephrol 5: 72–79, 2010

13. Couchoud C, Labeeuw M, Moranne O, Allot V, Esnault V, Frimat L,Stengel B, French Renal Epidemiology and Information Network(REIN) registry: A clinical score to predict 6-month prognosis inelderly patients starting dialysis for end-stage renal disease. NephrolDial Transplant 24: 1553–1561, 2009

14. Murray AM, Arko C, Chen SC, Gilbertson DT, Moss AH: Use ofhospice in the United States dialysis population. Clin J Am SocNephrol 1: 1248–1255, 2006

15. Renal Physicians Association: Shared Decision-Making in the Ap-

simultaneous pallia�ve care model

non-hospice-based pallia�ve care hospice-based palliative care

“before decision” plan

geriatric decision-making

‘func�onal age”

frailty syndrome

dialysis decision

dialysis therapy

non-dialysis medical therapy

“a�er decision” plan

Has func�onal status changed?

Have geriatric syndromes progressed?

Have goals of care changed?

•improve QOL/treat sources of suffering•pa�ent and family support•prognosis uncertain or measured in years

•group decision•burdens >benefits•prognosis based (< 6 months)

Figure 15. Geriatric renal palliative care and dialysis deci-sion plan. Comprehensive and updated care before and afterthe dialysis decision is an integral aspect of the action plan.The role of nonhospice palliative care is highlighted.

78 Nephrology Self-Assessment Program - Vol 10, No 1, January 2011

Page 91: 2011.01 - Geriatric Nephrology

propriate Initiation of and Withdrawal from Dialysis, 2nd Ed.,Rockville, Renal Physicians Association, 2010

16. Patel SS, Holley JL: Withholding and withdrawing dialysis in theintensive care unit: Benefits derived from consulting the RenalPhysicians Association/American Society of Nephrology clinicalpractice guideline, shared decision-making in the appropriate initia-tion of and withdrawal from dialysis. Clin J Am Soc Nephrol 3:587–593, 2008

17. Davison SN, Jhangri GS, Holley JL, Moss AH: Nephrologists’reported preparedness for end-of-life decision-making. Clin J Am SocNephrol 1: 1256–1262, 2006

18. Selman LE, Beattie JM, Murtagh FE, Higginson IJ: Palliative care:Based on neither diagnosis nor prognosis, but patient and familyneed. Commentary on Chattoo and Atkin. Soc Sci Med 69: 154–157,discussion 158–159, 2009

19. Meier DE, Lim B, Carlson MD: Raising the standard: Palliative carein nursing homes. Health Aff (Millwood) 29: 136–140, 2010

20. Meier DE, Isaacs SL, Hughes RG, eds. Palliative Care: Transform-ing The Care of Serious Illness. Robert Wood Johnson FoundationSeries on Health Policy, San Francisco, Jossey-Bass, 2010, p. 60

21. The National Consensus Project for Quality Palliative Care: ClinicalPractice Guidelines for Quality Palliative Care, 2nd Ed., 2009.Available at: http://nationalconsensusproject.org/. Accessed May 15,2010

22. World Health Organization: WHO Definition of Palliative Care,2008. Available at: www.who.int/cancer/palliative/definition/en/.Accessed April 20, 2010

23. Centers for Medicare & Medicaid Services (CMS), HHS: Medicareand Medicaid programs: Hospice conditions of participation. Finalrule. Fed Regist 73: 32087–32220, 2008

24. Huskamp HA, Stevenson DG, Chernew ME, Newhouse JP: A newMedicare end-of-life benefit for nursing home residents. Health Aff(Millwood) 29: 130–135, 2010

25. Casarett D, Pickard A, Bailey FA, Ritchie C, Furman C, Rosenfeld K,Shreve S, Chen Z, Shea JA: Do palliative consultations improvepatient outcomes? J Am Geriatr Soc 56: 593–599, 2008

26. Cohen LM, Moss AH, Weisbord SD, Germain MJ: Renal palliativecare. J Palliat Med 9: 977–992, 2006

27. Harding R, Moss AH, Weisbord SD, Germain MJ: Provision ofpalliative care for chronic heart failure inpatients: How much do weneed? BMC Palliat Care 8: 8, 2009

28. Adler ED, Goldfinger JZ, Kalman J, Park ME, Meier DE: Palliativecare in the treatment of advanced heart failure. Circulation 120:2597–2606, 2009

29. Reinke LF, Engelberg RA, Shannon SE, Wenrich MD, Vig EK, BackAL, Curtis JR: Transitions regarding palliative and end-of-life care insevere chronic obstructive pulmonary disease or advanced cancer:Themes identified by patients, families, and clinicians. J Palliat Med11: 601–609, 2008

30. Camhi SL, Mercado AF, Morrison RS, Du Q, Platt DM, August GI,Nelson JE: Deciding in the dark: Advance directives and continuationof treatment in chronic critical illness. Crit Care Med 37: 919–925,2009

31. Kapo J, Morrison LJ, Liao S: Palliative care for the older adult. JPalliat Med 10: 185–209, 2007

32. Pautex S, Curiale V, Pfisterer M, Rexach L, Ribbe M, Van DenNoortgate N: A common definition of geriatric palliative medicine.J Am Geriatr Soc 58: 790–791, 2010

33. Arnold RM, Jaffe E: Why palliative care needs geriatrics. J PalliatMed 10: 182–183, 2007

34. Brown EA, Chambers EJ, Eggeling C: Palliative care in nephrology.Nephrol Dial Transplant 23: 789–791, 2008

35. Holley JL: Palliative care in end-stage renal disease: Illness trajec-tories, communication, and hospice use. Adv Chronic Kidney Dis 14:402–408, 2007

36. Brown EA: Epidemiology of renal palliative care. J Palliat Med 10:1248–1252, 2007

37. Holley JL: Palliative care in end-stage renal disease: Focus onadvance care planning, hospice referral, and bereavement. Semin Dial18: 154–156, 2005

38. Jablonski A: Palliative care: Misconceptions that limit access forpatients with chronic renal disease. Semin Dial 21: 206–209, 2008

39. Noble H: Supportive and palliative care for the patient with end-stagerenal disease. Br J Nurs 17: 498–504, 2008

40. Moss AH, Holley JL, Davison SN, Dart RA, Germain MJ, Cohen L,Swartz RD: Palliative care. Am J Kidney Dis 43: 172–173, 2004

41. Holley JL, Davison SN, Moss AH: Nephrologists’ changing practicesin reported end-of-life decision-making. Clin J Am Soc Nephrol 2:107–111, 2007

42. Davison SN, Torgunrud C: The creation of an advance care planningprocess for patients with ESRD. Am J Kidney Dis 49: 27–36, 2007

43. Weisbord SD, Fried LF, Mor MK, Resnick AL, Unruh ML, PalevskyPM, Levenson DJ, Cooksey SH, Fine MJ, Kimmel PL, Arnold RM:Renal provider recognition of symptoms in patients on maintenancehemodialysis. Clin J Am Soc Nephrol 2: 960–967, 2007

44. Claxton RN, Blackhall L, Weisbord SD, Holley JL: Undertreatmentof symptoms in patients on maintenance hemodialysis. J Pain Symp-tom Manage 39: 211–218, 2010

45. Fried TR, O’Leary J, Van Ness P, Fraenkel L: Inconsistency overtime in the preferences of older persons with advanced illness forlife-sustaining treatment. J Am Geriatr Soc 55: 1007–1014, 2007

46. Kurella Tamura M, Covinsky KE, Chertow GM, Yaffe K, LandefeldCS, McCulloch CE: Functional status of elderly adults before andafter initiation of dialysis. N Engl J Med 361: 1539–1547, 2009

47. Lee SJ, Go AS, Lindquist K, Bertenthal D, Covinsky KE: Chronicconditions and mortality among the oldest old. Am J Public Health98: 1209–1214, 2008

48. Lee SJ, Lindquist K, Segal MR, Covinsky KE: Development andvalidation of a prognostic index for 4-year mortality in older adults.JAMA 295: 801–808, 2006

49. Rodin MB, Mohile SG: A practical approach to geriatric assessmentin oncology. J Clin Oncol 25: 1936–1944, 2007

50. Johansen KL, Chertow GM, Jin C, Kutner NG: Significance of frailtyamong dialysis patients. J Am Soc Nephrol 18: 2960–2967, 2007

51. Fried LF, Lee JS, Shlipak M, Chertow GM, Green C, Ding J, HarrisT, Newman AB: Chronic kidney disease and functional limitation inolder people: Health, aging and body composition study. J AmGeriatr Soc 54: 750–756, 2006

52. Li M, Tomlinson G, Naglie G, Cook WL, Jassal SV: Geriatriccomorbidities, such as falls, confer an independent mortality risk toelderly dialysis patients. Nephrol Dial Transplant 23: 1396–1400,2008

53. Covinsky KE, Eng C, Lui LY, Sands LP, Yaffe K: The last 2 yearsof life: Functional trajectories of frail older people. J Am Geriatr Soc51: 492–498, 2003

54. Mohile SG, Bylow K, Dale W, Dignam J, Martin K, Petrylak DP,Stadler WM, Rodin M: A pilot study of the vulnerable elderssurvey-13 compared with the comprehensive geriatric assessment foridentifying disability in older patients with prostate cancer whoreceive androgen ablation. Cancer 109: 802–810, 2007

55. Min LC, Elliott MN, Wenger NS, Saliba D: Higher vulnerable elderssurvey scores predict death and functional decline in vulnerable olderpeople. J Am Geriatr Soc 54: 507–511, 2006

56. Basso U, Monfardini S: Multidimensional geriatric evaluation inelderly cancer patients: A practical approach. Eur J Cancer Care(Engl) 13: 424–433, 2004

57. Balducci L, Yates J: General guidelines for the management of olderpatients with cancer. Oncology (Williston Park) 14: 221–227, 2000

58. Balducci L, Extermann M, Carreca I: Management of breast cancer inthe older woman. Cancer Control 8: 431–441, 2001

59. Walter LC, Covinsky KE: Cancer screening in elderly patients: A

Nephrology Self-Assessment Program - Vol 10, No 1, January 2011 79

Page 92: 2011.01 - Geriatric Nephrology

framework for individualized decision making. JAMA 285: 2750–2756, 2001

60. Fried LP, Hadley EC, Walston JD, Newman AB, Guralnik JM,Studenski S, Harris TB, Ershler WB, Ferrucci L: From bedside tobench: Research agenda for frailty. Sci Aging Knowledge Environ(31): pe24, 2005

61. Murtagh FE, Murphy E, Sheerin NS: Illness trajectories: An impor-tant concept in the management of kidney failure. Nephrol DialTransplant 23: 3746–3748, 2008

62. Gill TM, Baker DI, Gottschalk M, Peduzzi PN, Allore H, Byers A: Aprogram to prevent functional decline in physically frail, elderlypersons who live at home. N Engl J Med 347: 1068–1074, 2002

63. Fried LP, Tangen CM, Walston J, Newman AB, Hirsch C, GottdienerJ, Seeman T, Tracy R, Kop WJ, Burke G, McBurnie MA, Cardio-vascular Health Study Collaborative Research Group: Frailty in olderadults: Evidence for a phenotype. J Gerontol A Biol Sci Med Sci 56:M146–M156, 2001

64. Jonsen AR, Seigler M, Winslade WJ: Clinical Ethics, 6th Ed., NewYork, McGraw Hill, 2006

65. Swidler M: Dialysis decisions in the elderly patient with advanced CKDand the role of nondialytic therapy. In: American Society of NephrologyGeriatric Nephrology Curriculum, edited by Oreopoulos MD, WigginsJ, eds., San Luis Obispo, American Society of Nephrology, 2009,chapter 37, p. 8. Available at: http://www.asn-online.org/education_and_meetings/geriatrics/Chapter37.pdf. Accessed May 4, 2010

66. Carson RC, Juszczak M, Davenport A, Burns A: Is maximumconservative management an equivalent treatment option to dialysisfor elderly patients with significant comorbid disease? Clin J Am SocNephrol 4: 1611–1619, 2009

67. UK Renal Association, 2007. Available at: http://www.renal.org/Libraries/Old_Guidelines/Module_1_-_Chronic_Kidney_Disease_CKD_-_4th_Edition.sflb.ashx. Accessed June 7, 2010

68. Murtagh FE, Marsh JE, Donohoe P, Ekbal NJ, Sheerin NS, HarrisFE: Dialysis or not? A comparative survival study of patients over 75years with chronic kidney disease stage 5. Nephrol Dial Transplant22: 1955–1962, 2007

69. De Biase V, Tobaldini O, Boaretti C, Abaterusso C, Pertica N,Loschiavo C, Trabucco G, Lupo A, Gambaro G: Prolonged conser-vative treatment for frail elderly patients with end-stage renal disease:The Verona experience. Nephrol Dial Transplant 23: 1313–1317,2008

70. Wong CF, McCarthy M, Howse ML, Williams PS: Factors affectingsurvival in advanced chronic kidney disease patients who choose notto receive dialysis. Ren Fail 29: 653–659, 2007

71. Ellam T, El-Kossi M, Prasanth KC, El-Nahas M, Khwaja A: Con-servatively managed patients with stage 5 chronic kidney disease:Outcomes from a single center experience. QJM 102: 547–554, 2009

72. Smith C, Da Silva-Gane M, Chandna S, Warwicker P, Greenwood R,Farrington K: Choosing not to dialyse: evaluation of planned non-dialytic management in a cohort of patients with end-stage renalfailure. Nephron Clin Pract 95: C40–C46, 2003

73. Murtagh FE, Addington-Hall J, Edmonds P, Donohoe P, Carey I,Jenkins K, Higginson IJ: Symptoms in the month before death for

stage 5 chronic kidney disease patients managed without dialysis.J Pain Symptom Manage 40: 342–352, 2010

74. Davison SN: The prevalence and management of chronic pain inend-stage renal disease. J Palliat Med 10: 1277–1287, 2007

75. Weisbord SD, Fried LF, Arnold RM, Rotondi AJ, Fine MJ, LevensonDJ, Switzer GE: Development of a symptom assessment instrumentfor chronic hemodialysis patients: The Dialysis Symptom Index. JPain Symptom Manage 27: 226–240, 2004

76. Saini T, Murtagh FE, Dupont PJ, McKinnon PM, Hatfield P, Saun-ders Y: Comparative pilot study of symptoms and quality of life incancer patients and patients with end stage renal disease. Palliat Med20: 631–636, 2006

77. Murphy EL, Murtagh FE, Carey I, Sheerin NS: Understandingsymptoms in patients with advanced chronic kidney disease managedwithout dialysis: Use of a short patient-completed assessment tool.Nephron Clin Pract 111: c74–c80, 2009

78. Chater S, Davison SN, Germain MJ, Cohen LM: Withdrawal fromdialysis: A palliative care perspective. Clin Nephrol 66: 364–372,2006

79. Davison SN: Pain in hemodialysis patients: Prevalence, cause, sever-ity, and management. Am J Kidney Dis 42: 1239–1247, 2003

80. Douglas C, Murtagh FE, Chambers EJ, Howse M, Ellershaw J:Symptom management for the adult patient dying with advancedchronic kidney disease: A review of the literature and development ofevidence-based guidelines by a United Kingdom Expert ConsensusGroup. Palliat Med 23: 103–110, 2009

81. Murtagh FE, Addington-Hall JM, Donohoe P, Higginson IJ: Symp-tom management in patients with established renal failure managedwithout dialysis. EDTNA ERCA J 32: 93–98, 2006

82. Murtagh FE, Chai MO, Donohoe P, Edmonds PM, Higginson IJ: Theuse of opioid analgesia in end-stage renal disease patients managedwithout dialysis: Recommendations for practice. J Pain Palliat CarePharmacother 21: 5–16, 2007

83. Dean M: Opioids in renal failure and dialysis patients. J PainSymptom Manage 28: 497–504, 2004

84. Niscola P, Scaramucci L, Vischini G, Giovannini M, Ferrannini M,Massa P, Tatangelo P, Galletti M, Palumbo R: The use of majoranalgesics in patients with renal dysfunction. Curr Drug Targets 11:752–758, 2010

85. Davison SN, Mayo PR: Pain management in chronic kidney disease:The pharmacokinetics and pharmacodynamics of hydromorphoneand hydromorphone-3-glucuronide in hemodialysis patients. J OpioidManag 4: 335–336, 339–344, 2008

86. Barakzoy AS, Moss AH: Efficacy of the World Health Organizationanalgesic ladder to treat pain in end-stage renal disease. J Am SocNephrol 17: 3198–3203, 2006

87. The Marie Curie Palliative Care Institute: Liverpool Care Pathwayfor the Dying Patient. Available at: www.mcpcil.org.uk/liverpool-care-pathway/. Accessed June 1, 2010

88. Gott M, Small N, Barnes S, Payne S, Seamark D: Older people’sviews of a good death in heart failure: Implications for palliative careprovision. Soc Sci Med 67: 1113–1121, 2008

80 Nephrology Self-Assessment Program - Vol 10, No 1, January 2011

Page 93: 2011.01 - Geriatric Nephrology

Nephrology Self-Assessment Program

Examination QuestionsInstructions to obtain 8 AMA PRA Category 1 CreditsTM

Date of Original Release: January 2011Examination Available Online: on or before Monday, January 10, 2011Audio Files Available: No audio files for this issue

CME Credit Eligible Through: December 31, 2011

Answers: Correct answers with explanations will be posted on the ASN website in January 2012 when the issue is archived.UpToDate Links Active: January and February 2011

Core Nephrology question links active: No core questions for this issue

Target Audience: Nephrology Board and recertification candidates, practicing nephrologists, and internists.

Method of Participation:● Read the syllabus that is supplemented by original articles in the reference lists, and complete the online self-assessment

examination.● Examinations are available online only after the first week of the publication month. There is no fee. Each participant is

allowed two attempts to pass the examination (�75% correct).● Upon completion, review your score and incorrect answers.● Your CME certificate can be printed immediately after completion.● Answers and explanations are provided with a passing score and/or after the second attempt.● CME Credit will be posted to your transcript within 48 hours after checking the attestation box.

Instructions to Access the Online Examination and Evaluation:● Go to the ASN website: www.asn-online.org.● Click the CME tab at the top of the homepage.● Click the ASN CME Center button on the left side of the page.● Click on to the ASN CME Center icon.● Login to the ASN website.● Select Claim Credits for the NephSAP topic-activity you would like to complete.● Complete the NephSAP examination.● Complete the evaluation.● Enter the number of CME credits commensurate with your participation in the activity.● Check the box attesting that you have completed this activity.● You can print your CME certificate immediately.● CME credit will be posted to your transcript within 48 hours.● View or print your full transcript anytime at “My CME Center.”

Instructions to Obtain American Board of Internal Medicine (ABIM) Maintenance of Certification(MOC) Points:

Each issue of NephSAP provides 10 MOC points. Respondents must meet the following criteria:● Be certified by ABIM in internal medicine and/or nephrology and must be enrolled in the ABIM–MOC program

via the ABIM website (www.abim.org).● Take the self-assessment examination within the timeframe specified in this issue of NephSAP.● Designate the issue for MOC points by clicking on the MOC link on the CME certificate page after passing the examination.

You will be leaving the ASN site and transferring the information directly to the ABIM in real-time.● Provide your ABIM Certificate ID number and your date of birth.● You will receive a confirmation message from the ABIM indicating the receipt of your information.

MOC points will be applied to only those ABIM candidates who have enrolled in the program. It is your responsibility to completethe ABIM MOC enrollment process.

Nephrology Self-Assessment Program - Vol 10, No 1, January 2011

82

Page 94: 2011.01 - Geriatric Nephrology

Volume 10, Number 1, January 2011—Geriatric Nephrology

1. A 78-year-old man with a short duration (3years) of diabetes that required oral hypoglyce-mic agents is found to have stage 3 chronickidney disease (CKD) with proteinuria.

Which ONE of the following statements re-garding treatment of this patient is MOSTCORRECT?

A. Post hoc analysis of the Reduction ofEndpoints in NIDDM with the Angioten-sin II Antagonist Losartan (RENAAL)study suggests that this patient shouldreceive angiotensin II (AngII) antagonisttherapy.

B. Recent data have shown that physiciansprescribe AngII inhibitor or angiotensin-converting enzyme inhibitor in similar pa-tients with proteinuria the majority of thetime.

C. Although AngII antagonist therapy in suchpatients in the RENAAL study reducedprogression of decline in GFR, reductionin ESRD was not seen.

D. Although AngII antagonist therapy is ef-fective in similar patients, it is signifi-cantly more effective in patients who areyounger than 50 years.

2. You are referred a 76-year-old man with diabe-tes and stage 3 CKD (estimated GFR [eGFR] 30ml/min per 1.73 m2). The patient inquires aboutthe value of maintaining “tight” glycemic con-trol as suggested by his primary care physicianbecause he does not wish to adhere to a rigorousregimen.

Current studies would suggest which ONEthe following statements is MOST COR-RECT advice to this patient?

A. Tight glycemic control has been shown toreduce the risk for developing more ad-vanced CKD in patients with stage 3 CKDand diabetes.

B. If he strives to achieve tight glycemiccontrol with a hemoglobin A1c target of6%, then he can expect more frequent hy-poglycemic episodes.

C. The best approach would be the combineduse of chlorpropamide and glyburide.

D. You should advise him to pursue tightglycemic control despite his preference.

3. Which ONE of the following pathophysiologicor pathologic changes of diabetic nephropathyin the kidney is MOST inconsistent with thepathologic/pathophysiologic changes found inthe nondiabetic aging kidney?

A. Mesangial matrix expansion

B. Thickening of the glomerular basementmembrane

C. Diffuse global glomerular sclerosis

D. Glomerular hyperfiltration

4. A 74-year-old woman is seen by her familyphysician for fatigue, a dry cough, and low-grade fever lasting 2 weeks. She was givenbroad-spectrum antibiotics but failed to im-prove. A urinalysis showed 3� protein and 3�blood, and serum creatinine was 3.9 mg/dl. BPwas 139/68 mmHg. She was referred to a neph-rologist, who performed additional laboratorystudies and renal biopsy. The renal biopsy con-tained eight glomeruli, all of which showednecrotizing and diffuse crescentic lesions, andthe immunofluorescence showed a linear patternof IgG deposits. An ANCA test was stronglypositive for anti-myeloperoxidase antibody andan anti–glomerular basement membrane anti-body was negative. A chest x-ray was unremark-able. A repeat serum creatinine was 5.1 mg/dl.

Which ONE of the following treatment regi-mens would you recommend be startednow?

Nephrology Self-Assessment Program - Vol 10, No 1, January 2011

83

Page 95: 2011.01 - Geriatric Nephrology

A. Hemodialysis (HD) using a central venouscatheter

B. Intravenous methylprednisolone, oral cy-clophosphamide, and plasma exchange

C. Intravenous methylprednisolone and ritux-imab

D. Intravenous methylprednisolone and intra-venous cyclophosphamide

E. Intravenous methylprednisolone and oralmycophenolate mofetil

� 5. An 80-year-old man has sudden onset of severelower extremity edema and weight gain (6 kg in10 days). He has been taking allopurinol forgout prophylaxis, celocoxib for pain, losartanfor hypertension, and metformin for type 2 dia-betes. He has smoked half a pack of cigarettesdaily for the past 60 years. His BP is 160/90mmHg. Other than the pitting lower extremityedema, the physical examination, including aretinal examination, is negative. A chest x-rayshows mild emphysematous changes. A urinal-ysis reveals 4� protein and 1� blood. Serumcreatinine is 3.6 mg/dl, and serum albumin is 2.1g/dl. The erythrocyte sedimentation rate is 90mm/h (Westergren). Blood glucose is 140 mg/dl(nonfasting). Electrolytes show a sodium (Na)of 132 mEq/L, potassium (K) of 5.5 mEq/L,bicarbonate (HCO3) of 20 mEq/L, and chloride(Cl) of 98 mEq/L. A renal biopsy is performed.

Which ONE of the following lesions isMOST likely to be present in the renal bi-opsy?

A. Minimal Change Disease with interstitialnephritis

B. Amyloidosis

C. Crescentic glomerulonephritis

D. Membranous nephropathy

E. IgA nephropathy

6. A 71-year-old man is found to have proteinuriaand stage 3 CKD and is referred to you forfurther evaluation. Until 3 months ago, he feltquite well but has since been easily fatigued. Hehas lost approximately 3 kg (6.6 lbs) in weightto approximately 70 kg (154 lb). Other than“baby” aspirin and a � blocker (for a myocardial

infarction 4 years ago) and an occasional non-steroidal anti-inflammatory drug for “osteoar-thritis,” he takes no medication. His BP is150/88 mmHg. His physical examination is un-remarkable. Laboratory serum studies reveal thefollowing: Creatinine 1.9 mg/dl, hemoglobin10.9 g/dl, albumin 3.6 g/dl, globulin 3.4 g/dl,uric acid 7.0 mg/dl, and calcium 10.2 mg/dl.Serum electrolytes were as follows: Na 135mEq/L, K 4.9 mEq/L, Cl 100 mEq/L, and HCO3

26 mEq/L. Urine protein-creatinine ratio was1000 mg/g, serum C3 was 56 mg/dl, fluorescentanti-nuclear antibody was 1:80, rheumatoid fac-tor was weakly reactive, and hepatitis B/C se-rology was negative.

Which ONE of the following laboratorytests should you order now?

A. Serum protein electrophoresis

B. Serum quantitative Igs

C. Serum free light chains

D. Urinary electrophoresis

E. Anti–double-stranded DNA antibody

7. A 76-year-old man develops the insidious onsetof mild pedal edema over several months. Anevaluation by his family physician revealed pro-teinuria. He is referred to you for further inves-tigation. Other than mild fatigue and leg cramps,he has felt well. He takes no medication otherthan multivitamins and an occasional nonsteroi-dal anti-inflammatory drug for “aches andpains.” He has been a heavy smoker. His BP is160/88 mmHg, and the physical examinationreveals 1 to 2� pitting ankle edema but isotherwise unremarkable. A urinalysis reveals3� proteinuria. Further laboratory serum studiesshow the following: Creatinine 1.3 mg/dl, he-moglobin 11.3 g/dl, albumin 2.0 g/dl, and glob-ulins 4.0 g/dl. Serum electrolytes were as fol-lows: Na 134 mEq/dl, K 4.1 mEq/L, Cl 100Eq/L, HCO3 28 mEq/L. C3 was 120 mg/dl,hepatitis B/C serology was negative, and urineprotein-creatinine ratio was 12,000 mg/g. A re-nal biopsy reveals membranous nephropathy,stage 3.

Which ONE of the following additionalstudies should be conducted now?

84 Nephrology Self-Assessment Program - Vol 10, No 1, January 2011

Page 96: 2011.01 - Geriatric Nephrology

A. A bone marrow biopsy

B. A renal venous angiogram

C. A computerized tomography of the chest

D. A serum protein electrophoresis

E. A radioactive thyroid scan

� 8. A 66-year-old woman is found to have protein-uria, hematuria, stage 3 CKD, and hypertensionby her family physician. She is referred forfurther evaluation. Her BP is 168/98 mmHg, andher physical examination reveals 1� pedaledema. No skin rashes are present. Urinary pro-tein-creatinine ratio is 4800 mg/g, serum creat-inine is 1.6 mg/dl, and serum albumin is 3.0g/dl. Serum C3 is 8.2 mg/dl, and serum C4 is 11mg/dl. The fluorescent anti-nuclear antibody isweakly reactive at a titer of 1:80. A renal biopsywas performed and reveals “membranoprolif-erative glomerulonephritis.” No glomeruli werepresent in the specimen submitted for electronmicroscopy, but a single glomerulus showedstrong IgG and C3 deposits in a “granular pat-tern” with both � and � light chains in themesangium and capillary walls in the immuno-fluorescence microscopy study.

Which ONE of the following laboratorytests should be performed now?

A. An assay for serum C3 nephritic factor

B. An assay for serum complement factor Hlevels

C. A serum free light chain assay

D. A serum cryoglobulin assay

E. A serum anti–double-stranded DNA anti-body assay

� 9. An 89-year-old man is admitted to the intensivecare unit with pneumonia and sepsis. His base-line serum creatinine is 0.9 mg/dl, and on hos-pital day 1, his creatinine is noted to be 1.5mg/dl. During the next 3 days, he developsoliguria and requires institution of continuousvenovenous hemofiltration in addition to me-chanical ventilation and vasopressor support.During the next 10 days, he slowly improvesand is weaned off mechanical ventilation andvasopressor support. However, his oliguria con-tinued, and he is dialysis dependent. His family

is concerned about his continued need for dial-ysis and asks about the prognosis for his renalfunction.

Which ONE of the following statements isMOST CORRECT?

A. He will be dialysis dependent for the restof his life.

B. He should have gradual return of renalfunction to his baseline over the next fewdays.

C. As compared with younger patients, thelikelihood of renal recovery is approxi-mately 30% lower in elderly patients.

D. Renal recovery may take longer in elderlypatients, but he should eventually be dialy-sis independent.

10. An 86-year-old man with a history of dementiais cared for at home by his daughter. His base-line serum creatinine 2 months ago was 2.3mg/dl. Over the past few days, he has com-plained of some difficulty sleeping, and hisdaughter has given him diphenhydramine 25 mgat bedtime. However, he is now brought to theemergency department with altered mental sta-tus and complaints of lower abdominal pain. BPis 178/90 mm Hg, pulse is 70, and he has atemperature of 38°C. Physical examination re-veals a confused man who is thrashing around inbed. Abdominal examination is notable for dif-fuse tenderness and lower abdominal fullness.Laboratory examination reveals blood urea ni-trogen (BUN) of 120 mg/dl, creatinine of 9.8mg/dl, and K of 5.4 mmol/L.

Which ONE of the following is the nextMOST appropriate step in the care of thispatient?

A. Electrocardiogram

B. Foley catheter placement

C. Renal ultrasound

D. Digital rectal examination

E. HD

�11. A 78-year-old man with a 14-year history ofhypertension, type 2 diabetes, ischemic cardio-myopathy, and CKD is evaluated for treatmentof pain in his left knee. BP is 132/80 mmHg, and

Nephrology Self-Assessment Program - Vol 10, No 1, January 2011 85

Page 97: 2011.01 - Geriatric Nephrology

pulse is 70 per minute. Because of tendernessand effusion in the left knee joint, the patient isprescribed naproxen 500 mg twice daily. After12 days of therapy, the patient reports dyspnea,increased swelling in the lower extremities, andfatigue. BP is elevated to 178/102 mmHg, andlaboratory evaluation reveals a BUN level of 71mg/dl (baseline 38 mg/dl) and a serum creati-nine concentration of 4.3 mg/dl (baseline 2.1mg/dl). Urinalysis revealed no cellular elementsand no proteinuria. Spot urine sodium was �20mEq/L.

Which ONE of the following is the MOSTlikely mechanism by which naproxencaused acute kidney injury in this patient?

A. Drug reaction causing allergic interstitialnephritis

B. Acute papillary necrosis with renal ob-struction

C. Acute tubular necrosis from drug-inducednephrotoxicity

D. Hemodynamic-mediated renal insufficiency

12. A 79-year-old patient who recently started HDfor ESRD secondary to renovascular diseaseapproaches you to discuss his potential for kid-ney transplantation. He has not approached hisfamily to consider living donation. His clinicalhistory includes hypertension, mild angina-likesymptoms, and hyperlipidemia. He has smokedone pack of cigarettes per day for years. In yourregion, the average time for patients with hisblood type to be on the waiting list is 5 yearsbefore transplantation.

Which ONE of the following statementswould MOST accurately inform the patientwith regard to his candidacy for renaltransplantation?

A. He is unlikely to gain any benefit fromtransplantation because of an increasedrisk for death in the first 18 months post-operatively among patients aged �75years.

B. He would likely benefit from transplanta-tion, but emphasize that living donation ishis best option.

C. The American Society of Transplantation

recommendations suggest that he shouldundergo a stringent evaluation for occultcancers.

D. He should not seek a living donor.

E. He is best advised to avoid going on theextended-criteria donor waiting list be-cause the outcomes are poor when the re-cipient is older than 65 years.

13. An 80-year-old woman with a history of ESRDsecondary to drug-induced interstitial nephritiscomes to your transplant follow-up clinic. Shehad a living-donor transplant 18 months ago.Her clinical history is unremarkable apart fromher renal history. Her laboratory investigationsshow a serum creatinine of 1.5 g/dl, with anormal hemoglobin. Her immunosuppressantmedications include tacrolimus and low-dosagesteroids. She has just had a new grandchild andis updating her financial affairs and wants todiscuss her long-term prospects.

Which ONE of the following statements isCORRECT?

A. Her survival with a deceased-donor trans-plant in the United States is estimated at80% at 5 years.

B. She is more likely to experience anacute rejection compared with those aged18 to 45 years.

C. Her death-censored graft survival rate issimilar to individuals aged 18 to 45 years.

D. Her lack of comorbidities plays little partin predicting her survival rates.

14. An 87-year-old woman was started on HD 3months ago. At baseline, she had significantweight loss and anorexia, and her serum albu-min was 2.8 g/dl but recently has started tostabilize to between 2.8 and 3.2 g/dl. She hasimproved appetite, and her serum phosphate hasclimbed from a baseline of 3.1 to 5.4 mg/dl, witha serum calcium of 9.4 mg/dl. She has beenprescribed sevelamer but is taking only one800-mg tablet irregularly with her meals. Sheremains housebound and is unable to ambulatewithout assistance. She remains apathetic andfatigued and complains of reduced strength to

86 Nephrology Self-Assessment Program - Vol 10, No 1, January 2011

Page 98: 2011.01 - Geriatric Nephrology

participate in her own care. Although she liveswith her extended family, her daughter is themain caregiver and wants to know what toexpect for her mother’s health in the future.

Which ONE of the following is MOST clearlyassociated with increased risk for hospitaliza-tion and mortality in this patient?

A. Serum albumin of 3.2 g/dl

B. High serum phosphate-calcium product

C. Reduced physical activity, fatigue, and arecent history of unintentional weight loss

15. A 78-year-old man is routinely attending yourpredialysis clinic. He has a history of diabeticnephropathy with neuropathy and retinopathy. Hismedical history includes ischemic heart diseasewith previous four-vessel coronary artery bypassgraft surgery, a stroke resulting in mild left-sidedhemiparesis, and hypothyroidism. He lives withhis wife and adult son in a two-story home and isstill involved with managing the family business.

Which ONE of the following statements isMOST CORRECT regarding this patient’schoice of ESRD treatment?

A. He should be discouraged from choosingperitoneal dialysis because the data showearly increased mortality in comparisonwith HD.

B. He is poorly suited for home HD becausehe is at high risk for recurrent cardiovas-cular or cerebrovascular events.

C. He should be encouraged to use assisteddialysis programs (e.g., those that offerdaily home visits from a nurse) to pursuea home dialysis program.

D. He should be discouraged from choosingdialysis because elderly patients who haveneurologic impairment and are on dialysishave poor mental health component scores.

16. An 80-year-old man is admitted from home withpneumonia and delirium and treated with anti-biotics and fluids. His medical history is signif-icant for hypertension and chronic obstructivepulmonary disease. He remains confused, and acomputed tomography scan of the head is neg-ative. Three days into his course, he developsoliguric acute renal failure and fluid overload

and requires dialysis. The medical team hasbeen unable to contact the patient’s health careproxy, a brother who is traveling abroad. Aprogress note in the medical record from anadmission 4 years ago for a similar episode withacute renal failure resolved without dialysismentions that he did not want dialysis if heshould need it, but no reason was documented.

Which ONE of the following is the MOSTappropriate next step for this patient?

A. Dialysis should be delayed until the healthcare proxy can be contacted.

B. Two attending physicians should sign thedialysis consent, and dialysis should bestarted.

C. The medical team should request an ethicsconsultation.

D. A court-appointed guardian should be ur-gently obtained.

E. A hydromorphone drip should be started.

�17. An 82-year-old woman with mild cognitive im-pairment has been followed in the renal clinicfor 3 years. She first presented with stage 4CKD secondary to diabetic nephropathy withnephrotic syndrome, a BP of 160/100 mmHg, ahemoglobin of 8.5 g/dl, and an intact parathy-roid hormone of 226 pg/ml. With the addition oflosartan, epoetin alfa, and calcitriol, BP normal-ized, the hemoglobin rose to 10.1 g/dl, and theintact parathyroid hormone is now 140 pg/ml.However, her renal function has progressed tostage 5 CKD with an eGFR of 12 ml/min per1.73 m2, K 4.8 mEq/L, HCO3 16 mEq/L, hemo-globin 10.4 g/dl, and a urine protein:creatinineratio of 3000 mg/g. The patient and family havechosen nondialysis medical therapy.

Which ONE of the following statements isMOST CORRECT?

A. The patient should be referred to hospice.

B. The epoetin alfa should be discontinued.

C. Sodium bicarbonate 650 mg three timesdaily should be added.

D. A “Do Not Resuscitate” order should beobtained.

E. Lisinopril 10 mg/d should be started.

Nephrology Self-Assessment Program - Vol 10, No 1, January 2011 87

Page 99: 2011.01 - Geriatric Nephrology

18. A 71-year-old man with a history of rapidlyprogressive glomerulonephritis from ANCA�

systemic vasculitis has a functioning deceased-donor kidney transplant in the right iliac fossafor the past 9 months. He is maintained oncyclosporine (with stable blood levels in thetherapeutic range), mycophenolate mofetil (2g/d), low-dosage prednisone (20 mg/d). His se-rum creatinine has ranged between 1.4 and 1.6mg/dl. He presents to the emergency departmentwith the acute onset of fever, malaise, lowerabdominal pain, and moderate delirium. He hasabdominal tenderness, particularly in the rightlower iliac fossa, but no clinical guarding re-bound or masses. No other abnormality is noted.A white blood cell count is 14,100 mm3, andserum creatinine is 1.9 mg/dl. A urinalysisshows 1� protein, 1� blood, and 5 to 10 redblood cells and six white blood cells per high-power field in the urinary sediment. A computedtomography scan of the abdomen and a cyclo-sporine blood level are pending.

Which ONE of the following is the MOSTlikely explanation for these findings?

A. Acute allograft rejection, cellular type

B. Recurrent vasculitis

C. Sepsis, probably resulting from a perfo-rated colonic diverticulum

D. Gastrointestinal toxicity of mycophenolatemofetil

E. Cytomegalovirus gastroenteritis/colitis

19. A 71-year-old man is referred to you for thesudden onset of bloody urine and lower extremityedema. He has had a monogamous homosexualrelationship for the past 10 years. He was recentlyadmitted to the hospital for treatment of a sternalwound infection after an elective coronary arterybypass surgery. Culture of the wound revealedStaphylococcus aureus (methicillin resistant). Hewas treated with vancomycin for 3 weeks. At thistime, 1 month after his hospitalization, the woundis partially healed but still shows erythema andswelling. The physical examination shows 2�lower extremity edema and a BP of 165/94mmHg. No skin rashes are present. Serum creati-nine is 2.1 mg/dl (it was 1.0 mg/dl 2 weeks ago).Serum electrolytes are normal. A urinalysis shows

3� protein, 10 to 15 erythrocytes, and a fewerythrocyte casts per high-power field.

Which ONE of the following is the MOSTlikely cause of these clinical findings?

A. Hepatitis C cryoglobulinemia

B. IgA-dominant postinfectious glomerulonephritis

C. Systemic vasculitis

D. HIV nephropathy

E. Vancomycin nephrotoxicity

20. An 81-year-old man is referred to you by hisprimary care physician for consideration of di-alysis. The patient has been very active sociallyand intellectually but has had slowly progres-sive CKD that has been attributed to long-standing hypertension, although a renal bi-opsy has not been performed. He does nothave diabetes and has not had symptoms ofcongestive heart failure. He now complains ofuremic symptoms including weakness andpoor appetite. His BUN is 87 mg/dl, andserum creatinine is 6.2 mg/dl. After review ofhis physical examination and laboratory stud-ies, you determine that he is a candidate fordialysis treatment, but he questions you on theeffects of dialysis treatment on his lifestyle.

Which ONE of the following statementsabout dialysis therapy for this patient isMOST CORRECT?

A. Studies have shown that peritoneal dialysiswill produce a higher quality of life scorewhen assessed by the SF-36.

B. You tell him that on the basis of prospec-tive studies, he will feel as well physicallyand mentally as much younger patientswho have been on dialysis treatments foran extended period.

C. You tell him that he has a greater chanceof becoming clinically depressed thanyounger patients in his circumstances.

D. You tell him that he will feel less wellphysically than younger patients, but youdo not expect that he will have a diminu-tion in his mental acuity.

21. A 78-yr-old woman developed acute kidneyinjury after coronary artery bypass surgery. Her

88 Nephrology Self-Assessment Program - Vol 10, No 1, January 2011

Page 100: 2011.01 - Geriatric Nephrology

serum creatinine rose to 2.6 mg/dl for 5 days andthen gradually declined without the need forHD. During surgery, her mean arterial BP fell to48 mmHg for 30 minutes, but no other etiologicfactor was noted. She had an eGFR (by Modi-fication of Diet in Renal Disease equation) of 63ml/min per 1.73 m2 preoperatively and had along history of treated hypertension.

Which ONE of the following mechanismsMOST likely accounts for her failure totolerate renal hypoperfusion?

A. Failure to autoregulate renal blood flow

B. Increased renal nitric oxide generation

C. Enhanced activity of SIRT1 gene product

D. Preferential distribution of blood flow tothe renal medulla

22. Which ONE of the following drugs has beenshown to have effects MOST likely consistentwith an antiaging action?

A. Prednisone

B. Mycophenolate

C. Rapamycin

D. Growth hormone

E. Insulin

23. Which ONE of the following regimens hasbeen shown to slow the aging process in ex-perimental animals?

A. Daily aspirin treatment

B. Increased fruit and vegetable content ofthe diet to 50% of food intake

C. Forced aerobic exercise for 1 hour per day

D. Caloric restriction by 30%

24. You are called to see an 81-year-old man in theCoronary Care Unit with acute renal failure sec-ondary to decompensated congestive heart failure.He was admitted to a nursing home 6 months afterrepeated falls and inability to care for himself.Medical history is significant for stage 4 CKD,

ischemic cardiomyopathy, and moderate demen-tia. He responds to diuresis but at the expense ofconverting to stage 5 CKD. There are no acuteindications for dialysis yet, but dialysis discussionsare initiated as are discussions with the familyregarding long-term care. The family wants toknow the benefits of renal replacement therapy.

Which ONE of the following statements isMOST CORRECT?

A. Initiation of dialysis will slow any furtherfunctional decline.

B. Initiation of dialysis will improve his de-mentia.

C. His dementia will not be a factor affectinghis survival on dialysis.

D. After 6 months of dialysis, functional sta-tus will be preserved.

E. His survival may be similar with or with-out dialysis.

25. The adult children bring their mother, an 87-year-old woman with stage 5 CKD, for a follow-up visitto you after seeing a dialysis demonstration video.They relate that she is adamantly refusing anyconsideration of dialysis. The patient is coherentand appropriate but quiet and reserved. They relatethat 3 years ago she had a bad experience when herhusband died on dialysis.

Which ONE of the following is the BESTnext step?

A. Explain the pros and cons of nondialysistherapy and that she may choose that op-tion.

B. Ask for a psychiatric consultation to estab-lish decision-making capacity.

C. Explain the pros and cons of dialysis andthat she has a right to refuse dialysis.

D. Describe the symptoms of uremia and askthe patient and the family to come back in1 month.

Nephrology Self-Assessment Program - Vol 10, No 1, January 2011 89